You are on page 1of 136

FLORENTIN SMARANDACHE

PROBLEMS WITH
AND
WITHOUT …

PROBLEMS!

Translated from French

2011
FLORENTIN SMARANDACHE

PROBLEMS WITH
AND
WITHOUT …

PROBLEMS!

Translated from French into English

Original title:
“Problèmes avec et sans… problèmes!”, par Florentin Smarandache, Edition
Française, Traduit du Roumain par l’auteur et Sophie Mugnier, 1983,
SOCIETE MAROCAINE D’IMPRESSION ET D’EQUIPEMENT S.A.R.L.
SOMIPRESS, Fès, Marocco,
Siege Sociale: 72 Boulevard Hassan II, Tel: 238-68 – FES (v.n.).

1
This book can be ordered in a paper bound reprint from:

Books on Demand

ProQuest Information & Learning

(University of Microfilm International)

300 N. Zeeb Road

P.O. Box 1346, Ann Arbor

MI 48106-1346, USA

Tel.: 1-800-521-0600 (Customer Service)

http://wwwlib.umi.com/bod/basic

Copyright 2011 by Kappa & Omega

6744 W. Northview Ave.


Glendale, AZ 85303, USA

Peer-reviewers:
Prof. Ion Pătrașcu, Frații Buzești College, Craiova, Romania.
Prof. Gabriel Tica, Bailesti College, Bailesti, Jud. Dolj, Romania.
Dr. Fu Yuhua, 13-603, Liufangbeili, Liufang Street, Chaoyang district
Beijing, 100028, P. R. China.

ISBN-13: 9781599731421

Printed in the United States of America

2
PRE//FACE

This book is addressed to College honor students, researchers, and professors.


It contains 136 original problems published by the author in various scientific journals
around the world.
The problems could be used to preparing for courses, exams, and Olympiads in
mathematics.
Many of these have a generalized form.
For each problem we provide a detailed solution.

I was a professeur coopérant between 1982-1984, teaching mathematics in French language at


Lycée Sidi EL Hassan Lyoussi in Sefrou, Province de Fès, Morocco.

I used many of these problems for selecting and training, together with other Moroccan
professors, in Rabat city, of the Moroccan student team for the International Olympiad of
Mathematics in Paris, France, 1983.

The Author

3
Contents
• PRE//FACE ........................................................................................................................................ 3
• AMUSING PROBLEMS ...................................................................................................................... 9
1.1. .......................................................................................................................................................... 10
1.2. .......................................................................................................................................................... 12
1.3. .......................................................................................................................................................... 13
1.4. .......................................................................................................................................................... 15
1.5. .......................................................................................................................................................... 16
1.6. .......................................................................................................................................................... 17
1.7. .......................................................................................................................................................... 18
• ARITHMETIC ................................................................................................................................... 20
2.8. .......................................................................................................................................................... 21
2.9. .......................................................................................................................................................... 21
2.10. ........................................................................................................................................................ 21
2.11. ........................................................................................................................................................ 22
2.12. ........................................................................................................................................................ 22
2.13. ........................................................................................................................................................ 23
2.14. ........................................................................................................................................................ 23
2.15. ........................................................................................................................................................ 24
2.16. ........................................................................................................................................................ 24
2.17. ........................................................................................................................................................ 25
2.18. ........................................................................................................................................................ 26
2.19. ........................................................................................................................................................ 26
2.20 ......................................................................................................................................................... 27
2.21. ........................................................................................................................................................ 27
2.22. ........................................................................................................................................................ 28
2.23. ........................................................................................................................................................ 29
2.24. ........................................................................................................................................................ 30
2.25. ........................................................................................................................................................ 31
• MATHEMATICAL LOGIC .................................................................................................................. 34
3.26. ........................................................................................................................................................ 35
3.27. ........................................................................................................................................................ 36

4
• TRIGONOMETRY............................................................................................................................. 37
4.28. ........................................................................................................................................................ 38
4.29. ........................................................................................................................................................ 39
4.30. ........................................................................................................................................................ 40
• GEOMETRY ..................................................................................................................................... 42
5.31. ........................................................................................................................................................ 43
5.32. ........................................................................................................................................................ 43
5.33. ........................................................................................................................................................ 44
5.34. ........................................................................................................................................................ 45
5.35. ........................................................................................................................................................ 45
5.36. ........................................................................................................................................................ 46
5.37. ........................................................................................................................................................ 47
5.38. ........................................................................................................................................................ 48
5.39. ........................................................................................................................................................ 48
5.40. ........................................................................................................................................................ 49
5.41. ........................................................................................................................................................ 50
5.42. ........................................................................................................................................................ 51
5.43. ........................................................................................................................................................ 52
5.44. ........................................................................................................................................................ 53
5.45. ........................................................................................................................................................ 54
5.46. ........................................................................................................................................................ 54
5.47. ........................................................................................................................................................ 55
• CALCULUS....................................................................................................................................... 57
6.48. ........................................................................................................................................................ 58
6.49. ........................................................................................................................................................ 58
6.50. ........................................................................................................................................................ 59
6.51. ........................................................................................................................................................ 59
6.52. ........................................................................................................................................................ 60
6.53. ........................................................................................................................................................ 61
6.54. ........................................................................................................................................................ 61
6.55. ........................................................................................................................................................ 62

5
6.56. ........................................................................................................................................................ 63
6.57. ........................................................................................................................................................ 64
6.58. ........................................................................................................................................................ 64
6.59. ........................................................................................................................................................ 65
6.60. ........................................................................................................................................................ 65
6.61. ........................................................................................................................................................ 67
• ALGEBRA ........................................................................................................................................ 69
7.62. ........................................................................................................................................................ 70
7.63. ........................................................................................................................................................ 70
7.64. ........................................................................................................................................................ 71
7.65. ........................................................................................................................................................ 71
7.66. ........................................................................................................................................................ 72
7.67. ........................................................................................................................................................ 73
7.68. ........................................................................................................................................................ 73
7.69. ........................................................................................................................................................ 74
7.70. ........................................................................................................................................................ 75
7.71. ........................................................................................................................................................ 75
7.72. ........................................................................................................................................................ 76
7.73. ........................................................................................................................................................ 76
7.74. ........................................................................................................................................................ 77
7.75. ........................................................................................................................................................ 78
7.76. ........................................................................................................................................................ 78
7.77. ........................................................................................................................................................ 79
7.78. ........................................................................................................................................................ 80
7.79. ........................................................................................................................................................ 81
7.80. ........................................................................................................................................................ 81
7.81. ........................................................................................................................................................ 82
7.82. ........................................................................................................................................................ 83
7.83. ........................................................................................................................................................ 84
7.84. ........................................................................................................................................................ 85
7.86. ........................................................................................................................................................ 86
7.87. ........................................................................................................................................................ 86

6
7.88. ........................................................................................................................................................ 87
7.89. ........................................................................................................................................................ 89
7.90. ........................................................................................................................................................ 90
7.91. ........................................................................................................................................................ 90
7.92. ........................................................................................................................................................ 90
7.93. ........................................................................................................................................................ 91
7.94. ........................................................................................................................................................ 92
7.95. ........................................................................................................................................................ 93
7.96. ........................................................................................................................................................ 95
7.97. ........................................................................................................................................................ 96
7.98. ........................................................................................................................................................ 97
7.99. ........................................................................................................................................................ 98
7.100. ...................................................................................................................................................... 98
7.101. ...................................................................................................................................................... 99
7.102. .................................................................................................................................................... 100
7.103. .................................................................................................................................................... 101
7.104. .................................................................................................................................................... 102
7.105. .................................................................................................................................................... 103
7.106. .................................................................................................................................................... 104
7.107. .................................................................................................................................................... 104
7.108. .................................................................................................................................................... 105
7.109. .................................................................................................................................................... 105
7.110. .................................................................................................................................................... 106
7.111. .................................................................................................................................................... 106
7.112. .................................................................................................................................................... 107
7.113. .................................................................................................................................................... 108
7.114. .................................................................................................................................................... 108
7.115. .................................................................................................................................................... 109
7.116. .................................................................................................................................................... 109
7.117. .................................................................................................................................................... 110
7.118. .................................................................................................................................................... 111
7.119. .................................................................................................................................................... 112

7
7.120. .................................................................................................................................................... 112
7.121. .................................................................................................................................................... 113
7.122. .................................................................................................................................................... 114
7.123. .................................................................................................................................................... 115
7.124. .................................................................................................................................................... 116
7.125. .................................................................................................................................................... 117
7.126. .................................................................................................................................................... 118
7.127. .................................................................................................................................................... 119
7.128. .................................................................................................................................................... 120
7.129. .................................................................................................................................................... 121
7.130. .................................................................................................................................................... 122
7.131. .................................................................................................................................................... 122
7.132. .................................................................................................................................................... 123
7.133. .................................................................................................................................................... 125
7.134. .................................................................................................................................................... 130
7.135. .................................................................................................................................................... 132
7.136. .................................................................................................................................................... 133
• REFERENCES ............................................................................................................................ 134

8
AMUSING PROBLEMS

9
1.1.
Four teams of soccer: A, B, C and D participated in a tournament. The final ranking has
not been established, the stars will indicate the figures undecided (classification, teams, the
number of games played. Victories, null games, defeats, points marked, points missed):
1. A 3 *** 5-2 6
2. B * **2 3–3 *
3. C 3 *** 4-* *
4. D 3 *** 1–4 *
The teams have been separated by criteria known to the soccer rules and the ranking the
same.
a) Complete the grid.
b) Find the result of all disputed games.

Solution
a) The team A played 3 games, so A played against B . The same for C and D . Or B
B has to play 3 games. A has 6 points in 3 games, then A won all the games, therefore A has 3
victories, zero defeats. B has 2 defeats, therefore B is on the second place. It results that the
third game of B is a victory, because if it would be a null game, then C and D would have
6x2-(6+1) = 5 points, which will mean that at least one of them would have more points than B.
Therefore B has 2 points.
C and D have together 6x2-(6+2) = 4 points. Then C and D have both 2 points, because
otherwise it will result that at least one of them, C or D, will have more than B.
Therefore C has a victory, zero null games, and two defeats. The same for D. (C and D
can not get the 2 points from two null games, because A and B did not have any null matches),
C got (5+3+4+1)-(2+3+4) = 4 goals. The complete ranking is:
1. A 3 300 5–2 6
2. B 3 102 3–3 2
3. C 3 102 4–4 2
4. D 3 102 1–4 2
We know that in a ranking, the sum of marked goals by all the teams is equal to the sum
of the goals received by all the teams.
b) We determine the scenario of the games.
A has 3 victories, then A – B = 1, A – C = 1, A – D = 1
B and C have the same number of points, the same difference between the goals marked
and the goals received, the same number of victories, but B has a place superior than C, it results
that B – C =1, from where B – D = 2, then C – D = 1.
D marked only one goal and received 2 goals; the difference is 5 – 2 = 3. Then A took the
results: 1 – 0, 1 – 0, 3 – 2 or 1 – 0, 2 -1, 2 – 1. Because A – D = 1 and D marked its own goal
against B, it results that: A – D = 1 – 0, from which it results that C – D = 3 – 0.
The situation is:
1. A 2 200 4–2 4
2. B 2 101 3–2 2
3. C 2 002 1–4 0
with the scenario anterior.

10
A can have the results: 1 – 0, 3- 2, 2 – 1, 2 – 1. We observe that we cannot have the result
A – B = 3 – 2, because B marked only 2 goals. Then, it results the alternative 2 – 1, 2 – 1, from
which A – B = 2 -1, A – C = 2 -1 and we obtain B – C = 2 – 0. The exact results are:
A –B = 2 – 1, A – C = 2 – 1, A – D = 1 – 0, B – C = 2 – 0, B – D = 0 -1, C – D = 3 – 0.

11
1.2.
At the end of a soccer tournament of the teams A, B, C, D, the classment is the following:

1. A 3 201 2–1 4
2. B 3 201 2–1 4
3. C 3 111 4–4 3
4. D 3 012 3–5 1

The criteria for the teams rating was:


a) the number of accumulated points
b) the difference between the marked goals and the received goals
c) the number of victories
d) the direct victories against a team.
Find all the games results, knowing that for a victory a team gains 2 points, for a tight
game it gets 1 point, and for a defeat 0 (zero) points.
The first column represents the team order, second column the team, third column the number of
played games, fourth column the number of victories, fifth column the number of tight games,
sixth column thenumber of defeats, seventh column the numbers of marked goals, eighth column
the number of received goals, and last column the number of points.

Solution
Firstly we determine the exact estimation of the played games.
The teams A and B have the same number of points, the same difference between the
marked goals and the goals received, the same number of victories, but A is on the first place
while B is on the second place. From here A – B = 1 (that is, A gained a game). B has two
victories and one defeat, then B – C = 1 and B – D = 1. Then C – D = X (where X means an
equal game). A has a defeat, then A – C = 2. The exact estimations are:
A – B = 1, A – C = 2, A- D = 1, B – C = 1, B – D = 1, C – D = X
Now we determine the results.
Because A has 2 victories and marked only 2 goals, then its victories have been obtained
as 1 – 0, 1 – 0. We have then A – B = 1 – 0 and A – D = 1 – 0 , from where A – C = 0 – 1.
Similarly for B we have A – D = 1 – 0, from which A – C = 0, and therefore C – D = 3 –
3. The exact results are:
A – B = 1 – 0, A – C = 0 – 1, A – D = 1 – 0, B – C = 1 – 0, B – D = 1 – 0, C – D = 3 – 3.

12
1.3.
In the elaboration of a soccer ranking which follows, were made four errors, the order of
the teams remaining the same.

1. A 3 210 1–0 5
2. B 2 101 5–4 2
3 C 3 102 6–6 2
4. D 3 021 2–5 2

a) What are the errors?


b) Correcting these errors, find the results of all the games played.

Solution
a) Because A, C, D played 3 games its results that B also played 3 games (it is not possible to
have 2 games played by each team, because it would be more than 4 errors in the ranking). The
third game played by B cannot be a victory, because it would have in total 5 + 4 + 2 +2 = 13 ≠
12 points. (it is not possible to make other modifications on the points of A, C, or D, because we
would get more than 4 errors).
In the same way, the third game of B cannot be a defeat. Therefore B has a null game (
the third error). A has 2 victories and one single marked goal. The number of marked goals is 1 +
5 + 6 + 2 = 14 ≠ 15 = 0 + 4 + 6 + 5 which is the number of received goals by all the teams.
From which it results that A has marked 2 goals (the fourth error). (It is not possible to make
modifications on the received goals for A or for others for the same reason.
b) The correct ranking is

1. A 3 210 2–0 5
2. B 3 111 5–4 3
3 C 3 102 6–6 2
4. D 3 021 2–5 2

1) It is necessary to establish the exact forecasts.


D has 2 null games and A and B each have a null game.
Then A – D = X, B – D = X. The team A has also 2 victories. Then A – B = 1, A – C = 1. From
B – D = X and A – B = 1 it results that B – C = 1, because B has one victory. In the same way C
– D = 1.
The exact forecasts are:
A – B = 1, A – C = 1, A – D = X, B – C = 1, B – D = X, C – D = 1.
2) Now, it is sufficient to establish the exact results.
A has 2 victories, and 2 marked goals. Then A – B = 1 – 0, A – C = 1 – 0.
Because A did not receive any goals, we have A – D = 0 – 0.
Excluding the team A from the ranking ( with all its results), we obtain the following sub-
ranking:

2. B 2 110 5–3 3
3 C 2 101 6–5 2

13
4. D 2 011 2–5 1

with the known forecasts: B – C = 1, B – D = X, C – D = 1


B has one victory and no defeats, and the difference of goals is 5 – 3 = 2.
From which: B – C = 2 -0 or 3 -1, or 4 -2, or 5 – 3.
C has one victory and the difference of goals 6 – 5 = 1; but because B has won over C by 2 goals,
C wins therefore by 3 goals against D. From which C – D = 3 – 0 or 4 -1.

If we have C – D = 3 – 0, then we have B – C = 5 – 3, and B – D = 0 – 0. But this means that D


has zero marked goals. This is a contradiction. Therefore C – D = 4 – 1. And we have : B – C = 4
– 2, B – D = 1 – 1.
These last results verify the ranking
The results are:
A – B = 1 – 0, A – C = 1 – 0, A – D = 0 – 0, B – C = 4 – 2, B – D = 1 – 1, C – D = 4 – 1.
The problem has been uniquely resolved.
The problem is completely proved.

14
1.4.
For the preliminaries of the world soccer championship are disputed, back and forth, the
games of a group of 5 teams from which will be qualifies the first 2 teams.
Determine the minimum number of points for which a team can qualify, and also the
results that will enable the qualification.
Generalize the problem for the case of a group of n teams, from which must qualify the
first m (1 ≤ m ≤ n ).

Solution
We’ll resolve the exercise, directly, for the general case, the initial case will result as a
particularization.
In the group of n teams will be played 2 ⎡⎣(n −1) + (n − 2) + ... + (2 + 1)⎤⎦ = n (n −1)
games. The total number of points is 2n(n −1) . If m = 1 , the minimum number of points will be
2n(n −1) : 2 = 2(n −1) points (all teams have the same number of points, but that that will have
the largest difference between the goals marked and the goals received will qualify. If one team
has less than 2(n −1) points, then there will be another team which will have more that 2(n −1) ,
because the total number of points is equal to 2n(n −1) . If m = n , obvious, the minimum
number of points is zero.
In the case that 1 < m < n . The qualified team with the minimum number of points will
be the m th. When it is the minimum of points it means that the m − 1 first teams would have
obtained the maximum possible points. Then the h th team, 1 ≤ h ≤ m − 1 , would have 4(n − i )
points. The m − 1 teams would have 4(n −1) + 4(n − 2) + ... + 4(n − m + 1) = 2(m −1)(2n − m)
points. From the total number of points we remove the points of the first m − 1 teams and we find
2(n − m)(n − m + 1) , which represent the (n − m + 1) points of the remaining teams. Then
2( n − m )( n − m + 1)
= 2( n − m ) , which is the minimum number of points for a team to qualify.
n − m +1

15
1.5.
At a forecasting game regarding 13 soccer games, a person plays utilizing m doubles and n
triples, 0 ≤ m + n ≤ 13 , n , m ∈ N .
a) In the case that he’ll obtain a variant with 13 exact results, implicitly how many
variants of 12 and of 11 exact results he’ll obtain?
b) Also, if he gets 12 exact results, implicitly how many variants of 11 and of 10 exact
results will he obtain?

Solution
There are 13 games and for each there are three possibilities: 1, X or 2 ( that is, regarding
the first team, victory, null match or defeat). There are 12 possible variants (more than 1000000).
Having m the doubles and n the triples, it results that we have 13 − m − n solitaries, which
means the games for which we give only one answer. There are 2m ⋅ 2n variants in total.
a) We obtain m + 2n variant with 12 exact results.
If m ≥ 2 and n ≥ 2 , we have Cm2 + 4Cn2 + 2mn variants with 11 exact results; if m ≥ 2 and
2 2
n < 2 we have Cm + 2mn ; if m < 2 , n < 2 we have 2mn ; if m < 2 , n ≥ 2 we have 4Cn + 2mn
b) The case: when the solitaries are false. Then it results:
m + 2n variants with 11 exact results
Cm2 + 4Cn2 + 2mn variants with 10 exact results, or m ≥ 2 , n ≥ 2
4Cn2 + 2mn , if m < 2 , n ≥ 2 , variants have 10 exact results
Cm2 + 2mn , if m ≥ 2 and n < 2 variants with 10 exact results
2mn , if m < 2 , n < 2 variants with 10 exact results.

In the case that the double is false, we have


(m −1) + 2n variants with 10 exact results
Cm2 −1 + 4Cn2 + 2(m −1)n variants with 10 exact results, if m ≥ 3 , n ≥ 2
4Cn2 + 2(m −1)n variants with 10 exact results if m < 3 , n ≥ 2
Cm2 −1 + 2(m −1)n variants with 10 exact results if m ≥ 3 , n < 2
2(m −1)n variants with 10 exact results if m < 3 , n < 2 .

16
1.6.
At a tour of chess participated 10 players A1 , A2, ..., A10 - each chess players played with
each of the other players one game. For each victory one point is gained, for each null game half
of a point, and for each defeat zero points.
At the end of the tour, the ranking was

1. A1 9.5 points
2. A2 9 points
3. A3 6 points
4-5 A4 5 points
4-5 A5 5 points
6. A6 4 points
7-9. A7 2 points
7-9. A8 2 points
7-9. A9 2 points
10. A10 1 point

Show that in this ranking there are at least three errors.

Solution
The first error: A1 cannot accumulate more than the maximum of 9 points, because only
9 games were played, therefore there are no 9.5 points.
The second error: A2 , situated on the second place and the ranking, cannot accumulate
more than 8 points, not 9 points, because he cannot gain more than a maximum of 8 points (the
9th game, played against A1 was lost; against A1 the player A2 could not have a null match
because it would result that A2 should occupy the place 1 – 2, not 2).
The third error: in this tour there were played 9 + 8 + 7 +…+ 1 = 45 games, therefore the
total number of points of the ranking must be 45, because
9.5 + 9 + 6 + 2,5 + 4 +3,2 + 1 = 45.5 ≠ 45.

17
1.7.
Given a grid of crosswords (of n lines, m columns and p black boxes), such that there
are not two black cases that have a common side.

A B A

B B

B B
A B A

a) Prove that the number of the total words (horizontal and vertical) – we called word a
box that contains only one letter.
b) Find the difference between the number of horizontal words and the number of vertical
words.

Solution

a) We show that N = n + m + CNB + 2CNC , where


N = the number of the total words of the grille
CNB = the number of the black boxes in the B boxes
CNC = the number of the black boxes from the C boxes
We consider the grid divided in 3 zones.
10 the four corners of the grid (the A zone)
20 the border of the grid minus the four corners
30 the interior part of the grid (the C zone).
We assume that the grid at the beginning does not have any black boxes. Then, there are
n + m words.
- If we put a black box in the zone A, the number of the total words remains the same.
(Then the number of the black boxes from the zone A does not present any
importance)
- If we put a black box in the zone B, for example on the line 1 and column j ,
1 < j < m , the number of words being a unit, [because on the line 1 are formed now
two words (before there was only one word), and on column j there is also only one
word]. The situation is similar if we put a black box on the column 1 and line i ,

18
1 < i < n , ( we can reverse the grid: the horizontal to be the vertical and vice versa).
Then, for each black box from the zone B we add a word to the total number of the
words of the grid.
- If we place a black box in the zone C, for example on the line i , 1 < i < n , and the
column j , 1 < j < m , then the number of the words formed by two unites: as on the
line i , and on the column j are at this time two words, contrary to the previous
situation where it was a single word in each. Therefore, for each black case from the
zone C we add two words to the number of the words of the grid.
b) We divide the zone B in two parts:
- the zone BO = the horizontal part of B (the lines 1 and n )
- the zone BV = the vertical part of B (the columns 1 and m )
Then: NO − NV = n − m + CNBO − CNBV , where
NO = the number of the horizontal words
NV = the number of the vertical words
CNBO = the number of the black cases of BO
CNBV = the number of the black cases of BV

The proof of this proposition feats the precedent one, and we use the following
result:
- If there is no black boxes on the zone A, the difference NO − NV is equal to n − m
- If there is a black box on the zone A, the difference remains the same
- The same for the zone C
- If there is a black box on the zone BO , then the difference will be n − m + 1 , and if
the black box will be on the zone BV , then the difference will be n − m − 1 .

19
ARITHMETIC

20
2.8.
Determine the last digit of the numbers of the sequence of Fermat:
n
F n = 2 2 + 1 , with n ∈ N .

Solution
For n = 0 we have F0 = 3 , and for n = 1 we find F1 = 5 .
n n−2 n −2
For n ≥ 2 it results that F n = 22 + 1 = 24⋅2 + 1 = 162 + 1 = 16 K + 1 which contains as
last digit, 6+1=7, because the power of 16 ends in 6.

2.9.
Let p the product of the first n prime numbers.
Determine the set F = {α ∈ N | α ! = Mp} , Mp being the multiple of p .

Solution
Because α ! = Mp , 1 ≤ i ≤ n , we must have α ≥ Pi . Therefore α ≥ max {Pi } = Pn .
i

Pn ! = 1 ⋅ P1 ⋅ P2 ⋅ 4 ⋅ P3 ⋅⋅⋅ Pn−1 ⋅⋅⋅ Pn , from where Pn ! = Mp .


Pn is the smallest number which has this property, because if there exists α ' < Pn then
α '! ≠ Mp . If β > Pn then, of course β ! = Mp . And F = {Pn , Pn + 1, Pn + 2,...} .

2.10.
Find the smallest natural number such that its factorials are multiples of each of the
numbers 1970, 1980, 1990, and 2000.

Solution
The greatest prime number which divides one of the numbers from the above is 199.
Let α ∈ N the number which we’re seeking. Then α ! = M 1990 , i.e. multiple of 1990,
from where α = M 199. .
Then α ≥ 199 . We take α = 199 . 199! = M 10 because 10 < 199 . We also have 199! = M 197 .
Because (10,197) = 1 , it results that 199! = M 1970. .
199! = M 1990.
199! = M 36 and 199! = M 55 , but (36,55) = 1 ; from here 199! = M 36i55 = M 1980 .
199! = M 16 and 199! = M 125 and (166,125) = 1 ; then 199! = M 2000.
We suppose by absurd that α is not the smallest. Then, it exists α ' ≤ 199 such that
α '! = M 199 ; contradiction.

21
2.11.
Let A and B natural numbers. We consider M 1 = A + B, M 2 = A − B, M 3 = Ai B .
We note X m the numbers formed only by the last m digits of X .
a) Show that to find the last m digits of M 1 it is sufficient to find the last m digits of the
sum Am + Bm . The same question for M 2 and M 3 .
b) Generalization.
c) What it can be said about the last m digits of AB ?

Solution
a) We can write A = M 10m + Am and the same B = M10m + Bm . Then
M 1 = A + B = M 10m + ( Am + Bm )
The same:
M 2 = A − B = M 10m + ( Am − Bm )
M 3 = Ai B = M 10m + ( Am i Bm )
b) Generalization:
If E ( A1 , A2, ..., An ) is an arithmetic expression in which we have only the operations +, -,
and if A1 , A2, ..., An are natural numbers, then
Em ( A1 , A2, ..., An ) = E ( A1,m ,..., An ,m )
where Ai ,m represent the last m digits of Ai .
The proof results from a).
c) AB is a repeated multiplication. Therefore ( AB )m = AmB .

2.12.
Knowing what h hour and m minutes, 1 ≤ h ≤ 12 , 0 ≤ m < 60 , find after how long the
needles on the clock’s face would form an angle α , with 0 ≤ α < 360° .

Solution
Firstly we determine the angular speed for each needle on the clock’s face.
The big needle executes 360o in an hour; therefore VG = 6° / min .
The small needle executes 360o in 12 hours; therefore VS = 0.5° / min .
We compute the angle between the needles on the clock’s face at the h and m minutes.
The big needle would execute 6m o. The small needle would execute
(60h + m ) ⋅ 0.5 = 30h + 0.5m degrees.
We note by x (the minutes) the unknown of the problem.

22
The angle formed by the needles is 6m − 30h − 0.5m = 5.5m − 30h . (We consider the
angles positive, because in the problem it is not specified the direction of the angle).
A) The case in which 5.5m − 30h ≤ α .
a) 5.5m − 30h ≥ 0 ⇔ the big needle executes a distance (in degrees) greater than
the distance executed by the small needle. We have:
α + 30h − 5.5m
6 x − 0.5 x = α −(5.5m − 30h ) ⇒ x = .
5.5
b) 5.5m − 30h < 0 , the opposite situation. We have
α + 30h − 5.5m
6 x − 0.5 x = α −(5.5m − 30h ) ⇒ x = .
5.5
B) The case in which 5.5m − 30h > α .
a) 5.5m − 30h ≥ 0 . We have:
α + 360h + 30h − 5.5m
6 x − 0.5x = α + 360 −(5.5m − 30h ) ⇒ x = .
5.5
b) 5.5m − 30h < 0 . We have:
30h − 5.5m − α
6 x − 0.5 x = 5.5m − 30h − α = 5.5m − 30h − α ⇒ x = .
5.5

2.13.
Let a1 ,..., a2 n+1 integers and b1 ,..., b2 n+1 the same numbers but in a different order. Prove
that the expression E = (a1 ± b1 ) ⋅⋅⋅ (a2 n+1 ± b2 n+1 ) is an even number, where the signs +
and – are arbitrary in each parenthesis. (The generalization of problem A.7, page 105, of
D. Gerll and G. Gerard, “Les olympiades internationales de mathématiques”, Hachette,
1976).

Solution

We suppose that the expression E is odd. It resuls that each parenthesis is odd, therefore
each parenthesis contains an even number and the other an odd number.
We have then 2n + 1 even numbers. But, if in a parenthesis we find an ai0 even, then
there exists another parenthesis where we find a b j0 = ai0 , then b j0 is even. And the number of
evens is an odd number, which, obviously is different of 2n + 1 . This is a contradiction.

2.14.
Resolve the equation: X −Φ( X ) = 24 , knowing that Φ( X ) represents the number of
positive numbers, smaller than X and relatively prime in rapport to X .

23
Solution
Because Φ( X ) ∈ N , It results that X = 24 + Φ( X ) ∈ N * and X ≥ 24 . Let
X = P1α1 ⋅⋅⋅ Psαs , αi ∈ N * , Pi different prime numbers, i = 1, s .
Φ( X ) = P1α −1 ⋅⋅⋅ Psα −1 ⋅ ( P1 − 1) ⋅⋅⋅ ( Ps − 1) , Φ being the Euler’s function from the number
1 s

theory.
X −Φ( X ) = P1α1−1 ⋅⋅⋅ Psαs −1 ⋅ ⎡⎣ P1 ⋅⋅⋅ Ps −( P1 −1) ⋅⋅⋅( Ps −1)⎤⎦ − 24 = 23 ⋅ 31 ;
then, evidently, X has the form: X = 2α1 ⋅ 3α2 , where α1 , α2 ∈ N * . Then s = 2 .
We obtain X −Φ( X ) = P1α1−1 ⋅ P2α2 −1 ⋅ ⎡⎣ P1 ⋅ Ps −( P1 −1) ⋅( P2 −1)⎤⎦ , which means that
X − Φ( X ) = 2 α1−1 ⋅ 3α2 −1 ⋅[ 6 − 1, 2 ] = 2 3 ⋅ 31 , where X −Φ( X ) = 2α1−1 ⋅ 3α2 −1 = 21 ⋅ 31 , from
where α1 = α2 = 2 and in conclusion X = 22 ⋅ 32 = 36 .

2.15.
Let Φ(n) be Euler’s totient function. Prove that: Φ(n) is a prime number if and only if
n ∈ {0, ±3, ±4, ±6} .

Solution
The sufficiency:
Φ(0) = Φ(±3) = Φ(±4) = Φ(±6) = 2 , which is a prime number.
The necessity:
Φ(±1) = Φ(±2) = 1 which is not a prime number. Then n ≠ {±1, ±2} .
Let n = P1α1 ⋅⋅⋅ Psαs with P1 ,..., Ps different prime numbers. αi ∈ N * , i ∈ {1, 2,..., s} .
Φ(n) = P1α1−1 ( P1 −1) ⋅⋅⋅ Psαs −1 ( Ps −1) = M 2 for n ∉ {±1, ±2} because Pi − 1 = M 2
α j −1
where Pj = M2 .
Because Φ(n) is prime number, it results that Φ(n) = 2 . Then Pi −1 = 1 or Pi − 1 = 2 , or
3. Then Pi = 1 ⇒ α j = 2 , therefore n = 4,3,6 . And n ∈ {±3, ±4, ±6} .
But Φ(0) = 2 which is prime, then n ∈ {0, ±3, ±4, ±6} .

2.16.
Let m be an integer such that Φ(m) = M 4 (multiple of 4), where Φ represents Euler’s
indicator . Prove that it exists an even number of primitive solutions modulo m (an integer a is
called a primitive solution modulo m , if a Φ( m ) ≡ 1(mod m) and a K ≡
/ 1(mod m) for
1 ≤ K < Φ(m) ).

24
Solution
1) If there does not exists a primitive solution modulo m , then we have 0 solutions and 0
is an even number.
2) If there exist primitive solutions, let r one of them. We have:
( r, m) = 1 , r Φ ( m ) ≡ 1(mod m ) and r K ≡
/ 1(mod m)
For 1 ≤ K < Φ(m) . We’ll show that m − r is also a primitive solution modulo m .
A) Firstly, m − r ≠ r(mod m) , because contrarily it would result that 2r ≡ 0(mod m) ,
where 2r = t ⋅ m , with t ∈ Z . Because Φ(m) = M 4 , we have m ∈ {0, ±1, ±2} .
α ) m = 2 h , h ∈ Z − {0, ±1} . We have m | 2r ⇒ 2h | 2r ⇒ h | r ⇒ ( r, m) = h ≠ ±1 , which
is absurd.
β ) m = 2h + 1 , h ∈ Z − {−1,0} . We have m | 2r ⇒ m | r ⇒ ( r, m) = m ≠ ±1 , which is
absurd. Therefore m − r ≡ / r(mod m)
B) (m, m − r ) = d ⇒ d | m and d | m − r ⇒ d | r ⇒ d = ( r, m) = 1 , then (m − r, m) = 1 .
Φ( m )
(m − r ) ≡ 1(mod m) , in accordance with Euler’s theorem.
μ
We suppose, by absurd, that exists μ ∈ N * , μ < Φμ (m) with (m − r ) ≡ 1(mod m) . It results
μ
1 ≡ (m − r ) ≡ (−r ) ≡ (−1)μ r μ (mod m) . From where μ is odd (if not it results that
r μ ≡ 1(mod m ) and 1 ≤ μ < Φ(m) , in other words r will not be a primitive solution). Then
μ = 2 p + 1 , with p ∈ N and r μ ≡ −1(mod p ) , where r 2 μ ≡ 1(mod p ) . But μ < Φ(m) , which
implies that 2μ < 2Φ(m) . Because r is a primitive solution we have 2μ = Φ(m) , where
μ
Φ( m ) = 2 (2 p + 1) ≠ M 4 . Contradiction. And (m − r ) ≡
/ 1(mod m) for 1 ≤ μ < Φ(m) , therefore
m − r is also a primitive solution.

2.17.
Let m a natural number ≥ 3 , and a 1 ,..., a p all the positive numbers smaller than m and
different of m . Then a 1+a 2 +... + a p = Mm (multiple of m).

Solution
We prove that p = M 2 . We observe that if 0 < a < m and (a, m) = 1 , then we also have
0 < m − a < m and (m − a , m ) = 1 , because:
0 < a < m ⇒ −m < a − m < m − m ⇒ 0 < m − a < m , let d = (a − m, m) , it results d | m , from
where d | a , therefore d divides (a , m ) = 1 , and d = 1 . We have that for
∀a ∈ {a 1 ,..., a p } , ∃ m − a ∈ {a 1 ,..., a p } such that m − a ≠ a ; (in the contrary case, it would
have resulted that m = 2 a and (a , m ) = a ≠ 1 from where m = 2 , which is impossible). (1)
But a + ( m − a ) = m = Mm and, because of (1) we obtain the conclusion of the problem.

25
2.18.
Given three integer numbers a , b, c , such that a 2 + c 2 ≠ 0 and b 2 + c 2 ≠ 0 , prove that
(a, b, c) ⋅ c
∈Z
(a, c) ⋅(b, c)
(The notation ( x1 ,..., xn ) represents the largest common divisor of the numbers x1 ,..., xn )

Solution
Let d = (a, b, c) . This implies that a = a ' d , b = b ' d , c = c ' d and (a ', b ', c ') = 1 . Then
(a , c ) = (a ' d , c ' d ) = d ⋅ (a ', c ') = d ⋅ d13
(we note (a ', c ') = d 23 ); from where c ' = d 23 ⋅ β , β ∈ Z .
But (d13 , d 23 ) = ((a ', c ') ,(b ', c ')) = (a ', b ', c ') = 1
Because d13 ⋅ α = d 23 ⋅ β , (d13 , d 23 ) = 1 , and that all numbers are integers, it results that d 23
divides α , that is α = d 23α ' with α ' ∈ Z . Then c = d ⋅ d13 ⋅ d 23 ⋅ α ' , and that
(a, b, c) ⋅ c d ⋅ d ⋅ d13 ⋅ d 23 ⋅ α '
= = α' ∈ Z .
(a, c) ⋅(b, c) d ⋅ d13 ⋅ d ⋅ d 23
The conditions from the problem ensure the existence of the expression and that the denominator
is different of zero.

2.19.
Given ai , bi ∈ N , i = 1, n . Prove that:
n
(a 1 ,..., a n , b 1 ,..., b n ) ≥ ∏(a i , b i )
i =1

where (α, β ) represents the greatest common divisor of the numbers α and β .

Solution
We’ll apply the recurrence reasoning.
It is evident for i = 1 . We have to show that for i = 2 we have
(a 1 a 2 , b 1 b 2 ) ≥ (a 1 , b 1 ) ⋅ (a 2 , b 2 ) . Having a 1= a 11 d a 1 b1 , b 1= b 11 d a 1 b1 with (a 11 , b 11 ) = 1 and
having a 2 = a 21 d a2b2 , b 2 = b 21 d a 2 b2 with (a 21 , b 21 ) = 1 , then
(a 1 a 2 , b 1 b 2 ) = d a b ⋅ d a b ⋅ (a 11 a 21 , b 11 b 21 ) ≥ d a b ⋅ d a b = (a 1 , b 1 ) ⋅ (a 2 , b 2 ) .
1 1 2 2 1 1 2 2

We suppose that the inequality is true for the values of i which are smaller than n . It results
that:
⎛ n ⎞ n+1
(a 1...a n a n+1 , b 1...b n b n+1 ) ≥ (a 1...a n , b 1...b n ) ⋅(a n+1 , b n+1 ) ≥ ⎜⎜⎜∏(a i , b i )⎟⎟⎟ ⋅(a n+1 , b n+1 ) = ∏(a i , b i ) .
⎝ i=1 ⎠ i =1

26
2.20
If (a i , b i ) ∈ N 2 , i ∈ {1, 2,..., n} and [ α, β ] represent the smallest common multiple of the
n
numbers α and β , then ⎡⎣ a 1...a n a n+1 , b 1...b n ⎤⎦ ≤ ∏[ a i , b i ]
i =1

Solution
We’ll apply the recurrence reasoning.
It is evident for i = 1 . We have to show that for i = 2 we have
[ a 1 a 2 , b 1 b 2 ] ≤ [ a 1 , b 1 ]⋅[ a 2 , b 2 ] . Having a 1= a 11 d a 1 b1 , b 1= b 11 d a 1 b1 with (a 11 , b 11 ) = 1 and
having a 2 = a 21 d a2b2 , b 2 = b 21 d a 2 b2 with (a 21 , b 21 ) = 1 , then
[ a 1 a 2 , b 1 b 2 ] = d a b ⋅ d a b ⋅[ a 11 a 21 , b 11 b 21 ] ≤ d a b ⋅ d a b ⋅ a 11⋅a 21⋅b 11⋅b 21 = [ a 1 , b 1 ]⋅[ a 2 , b 2 ] .
1 1 2 2 1 1 2 2

We suppose that the property is true for the values of i ≤ n . It results that it is true also for
i = n + 1 because:
⎛ n ⎞
⎡ a 1...a n a n+1 , b 1...b n b n+1 ⎤ ≤ [ a 1...a n , b 1...b n ]⋅ ⎡ a n+1 , b n+1 ⎤ ≤ ⎜⎜∏[ a i , b i ]⎟⎟ ⋅ ⎡ a n+1 , b n+1 ⎤ . Then the
⎣ ⎦ ⎣ ⎦ ⎜⎝ ⎠⎟ ⎣ ⎦
i =1

problem is proved.

2.21.
Let m be a natural number, and 1 ≤ n ≤ 5 . Prove that if 9 m = a 1⋅⋅⋅a m , then
9 ⋅⋅⋅ 9m = 9 ⋅⋅⋅ 9 a1 0 ⋅⋅⋅ 0 a2 9 ⋅⋅⋅ 9 ⋅⋅⋅ a n , where there are n digits of 9 in the left-hand side of this
n−1 n−1 n−1
equation, with n ∈ N * .

Solution
When m = 1 , we have 91 = 9, 9 ⋅⋅⋅ 91 = 9 ⋅⋅⋅ 9 9 .
n n−1
When m = 2 , we have:
2
⎛ ⎟⎞
2 2 ⎜
9 = 81, 9 ⋅⋅⋅ 9 = ⎜100 ⋅⋅⋅ 0 − 1⎟⎟ = 10 ⋅⋅⋅ 0 − 2 0 ⋅⋅⋅ 0 + 1 = 10 ⋅⋅⋅ 0 0 ⋅⋅⋅ 01 − 2 0 ⋅⋅⋅ 0 = 9 ⋅⋅⋅ 980 ⋅⋅⋅ 01
n
⎜⎝ n ⎠ 2n n n n−1 n n−1 n−1

When m = 3 , we have:
3
⎛ ⎞⎟
3 3 ⎜
9 = 729, 9 ⋅⋅⋅ 9 = ⎜100 ⋅⋅⋅ 0 − 1⎟⎟ = 10 ⋅⋅⋅ 0 − 30 ⋅⋅⋅ 0 + 30 ⋅⋅⋅ 0 − 1 =
n ⎝⎜ n ⎠ 3n 2n n

= 10 ⋅⋅⋅ 0 0 ⋅⋅⋅ 0 30 ⋅⋅⋅ 0 − 30 ⋅⋅⋅ 0 0 ⋅⋅⋅ 01 = 9 ⋅⋅⋅ 9 7 0 ⋅⋅⋅ 0 2 9 ⋅⋅⋅ 9 9 .


n n−1 n n n n−1 n−1 n−1

When m = 4 , we have:

27
4
⎛ ⎞
9 = 6561, 9 ⋅⋅⋅ 9 = ⎜⎜100 ⋅⋅⋅ 0 − 1⎟⎟⎟ = 10 ⋅⋅⋅ 0 − 4 0 ⋅⋅⋅ 0 + 60 ⋅⋅⋅ 0 − 4 0 ⋅⋅⋅ 0 + 1 =
4 4

n
⎜⎝ n ⎠ 4n 3n 2n n

= 10 ⋅⋅⋅ 0 0 ⋅⋅⋅ 0 6 0 ⋅⋅⋅ 0 0 ⋅⋅⋅ 01 − 4 0 ⋅⋅⋅ 0 0 ⋅⋅⋅ 0 4 0 ⋅⋅⋅ 0 = 9 ⋅⋅⋅ 9 6 0 ⋅⋅⋅ 059 ⋅⋅⋅ 9 6 0 ⋅⋅⋅ 01 .
n n n n n n n n−1 n−1 n−1 n−1

When m = 5 , we have:
5
⎛ ⎞⎟
5 5 ⎜
9 = 59049, 9 ⋅⋅⋅ 9 = ⎜100 ⋅⋅⋅ 0 −1⎟⎟ = 10 ⋅⋅⋅ 0 − 50 ⋅⋅⋅ 0 + 100 ⋅⋅⋅ 0 −100 ⋅⋅⋅ 0 + 50 ⋅⋅⋅ 0 − 1 =
n
⎜⎝ n ⎠ 5n 4n 3n 2n n

= 9 ⋅⋅⋅ 9 50 ⋅⋅⋅ 0 9 9 ⋅⋅⋅ 9 0 0 ⋅⋅⋅ 0 4 9 ⋅⋅⋅ 9 9 .


n−1 n−1 n−1 n−1 n−1

Observation: For m ≥ 6 the formula is not true.

2.22.
⎪⎧ ⎪⎫
We consider the set A = ⎪⎨9 ⋅⋅⋅ 9 ⋅ m / m ∈ N *⎪⎬ and n ∈ N * , constant .
⎪⎩⎪ n ⎪⎭⎪
a) Compute the greatest number of 2n digits from the set A , which does not
contain the digit 9
b) Compute the smallest number 2n + K digits which does not contain the digit 9.
Discussion.

Solution
a) We’ll try to find the greatest m ∈ N * , m = b 1⋅⋅⋅b n , which multiplied by 9 ⋅⋅⋅ 9
n

gives us a product of 2n digits with all its digits different of 9.


⎛ ⎞
9 ⋅⋅⋅ 9 ⋅ m = ⎜⎜10 ⋅⋅⋅ 0 − 1⎟⎟⎟ ⋅ m = m 0 ⋅⋅⋅ 0 − m .
n
⎜⎝ n ⎠ n

We compute the difference : m = b 1⋅⋅⋅bn 0 ⋅⋅⋅ 0 − b 1⋅⋅⋅bn = ?


n

If there exist b j = 9, j ∈ {1,2,..., n −1} , then: it there is at least a non zero digit at the end
of b j , by subtraction it will result b j = 9 in the solution, if all the bh = 0 with h ∈ { j + 1,..., n}
by subtraction in the solution it will exist at least one digit equal to 9 in one of the places
j + 1,..., n .
The next case is 8 ⋅⋅⋅ 89 . By doing the difference (that is a multiplication 9 ⋅⋅⋅ 98 ⋅⋅⋅ 89 )
n n n

we will obtain the greatest number of 2n digits of the set A which does not contain the digit 9,
which is 8 ⋅⋅⋅ 81 ⋅⋅⋅1 .
n n

b) The number m would be n + K digits.


1) The case 1 ≤ K ≤ n . We prove that m = 10 ⋅⋅⋅ 01 ⋅⋅⋅12 is the smallest number
K −1 n−1

of n + K digits which will have the required property.

28
We cannot have any zero among the last digits of the number m because it would result,
by multiplication, at least one digit 9 in the product; the last digit non zero of m is different of 1
(for the same reason); the other digits of m can be equal to zero, only the first would have the
value of minimum 1 because K ≤ n , then K −1 ≤ n −1 .
9 ⋅⋅⋅ 9 ⋅10 ⋅⋅⋅ 01 ⋅⋅⋅12 = 1 ⋅⋅⋅11 ⋅⋅⋅18 ⋅⋅⋅ 8
n K −1 n−1 n−K +1 K −1 n

(here we wrote directly that 9 ⋅⋅⋅ 9 ⋅1 ⋅⋅⋅12 = 1 ⋅⋅⋅18 ⋅⋅⋅ 8 )


n n−1 n n

(We utilize the property that the smallest number of 2n digits of A , which does not contain the
digit 9 is 1 ⋅⋅⋅18 ⋅⋅⋅ 8 , and that the correspondent m is ⋅1 ⋅⋅⋅12 ).
n n n−1

2) The case K ≥ n + 1 . Momentarily, we cannot write m = 10 ⋅⋅⋅ 01 ⋅⋅⋅12 because


K −1 n−1

K − 1 > n − 1 and the result of the multiplication contains digits 9:


9 ⋅⋅⋅ 9 ⋅10 ⋅⋅⋅ 01 ⋅⋅⋅12 = 1 ⋅⋅⋅18 ⋅⋅⋅ 8 (1)
n K −1 n−1 n n

9 ⋅⋅⋅ 9 = 9 ⋅⋅⋅ 9 0 ⋅⋅⋅ 01 ⋅⋅⋅18 ⋅⋅⋅ 8


n n K −1 n n

We’re looking to find the smallest m ∈ N * , of n + K digits, which will have the
requested property. The last n digits of m will be also 1 ⋅⋅⋅12 . The first will be also 1.
n−1

Among the unknown digits we cannot have more than n − 1 consecutive zeros because of
(1). Because n is also small it is possible to attach n − 1 consecutive zeros after the first digit,
then a digit 1 (the minimum not null), other n − 1 consecutive zeros and again a digit 1, etc.
Therefore: m = 10 ⋅⋅⋅ 010 ⋅⋅⋅ 0 ⋅⋅⋅10 ⋅⋅⋅ 010 ⋅⋅⋅ 01 ⋅⋅⋅12
n−1 n−1 n−1 p n−1
K digits

⎡K⎤
Then, the number will be 9 ⋅⋅⋅ 9 ⋅ m = 10 ⋅⋅⋅ 01⋅⋅⋅101 ⋅⋅⋅18 ⋅⋅⋅ 8 with p = K − n ⋅ ⎢ ⎥ −1 , where
n K −1 n− p−1 p n ⎣⎢ n ⎦⎥
[ X ] represents the integer part of X .
This is the multiplication:
9 ⋅⋅⋅ 9 ⋅10 ⋅⋅⋅ 010 ⋅⋅⋅ 0 ⋅⋅⋅10 ⋅⋅⋅ 010 ⋅⋅⋅ 01 ⋅⋅⋅12 = 1 ⋅⋅⋅101 ⋅⋅⋅18 ⋅⋅⋅ 89 ⋅⋅⋅ 9 9 ⋅⋅⋅ 9 9 ⋅⋅⋅ 9 ⋅⋅⋅ 9 ⋅⋅⋅ 9 =
n n−1 n−1 n−1 p n−1 n− p p n p n− p n n
K

= 10 ⋅⋅⋅ 0 ⋅⋅⋅ 0 ⋅⋅⋅ 0 0 ⋅⋅⋅ 01 ⋅⋅⋅101 ⋅⋅⋅18 ⋅⋅⋅ 8


n n p n− p−1 p n
K

2.23.
If x, y ∈ N , then there exist z ∈ N such that 10 x ⋅10 y = 10 z .
Generalize this result for the case of any number of zeros between 1 and x and between
1 and y .

29
Solution
Let’s consider x = a 1...a n , 0 ≤ ai ≤ 9 , i ∈ {1, 2,..., n} , n ∈ N * and y = b 1...b m ,
0 ≤ b j ≤ 9 , j ∈ {1, 2,..., m} , m ∈ N * .
We do the multiplication:

10a 1...a n ×
10b 1...b m

n + 2 digits ⋅ ⋅⋅⋅ the multiplication by bm


..................................................................................
n + 2 digits ⋅ ⋅⋅⋅ the multiplication by b1
n + 2 digits 1 0 ⋅ ⋅ the multiplication by 1

=1 ⋅ ⋅ ⋅
We noted by “ ⋅ ” a natural digit between 0 and 9.
Then, the first digit of the product is 1.
Generalization: if x, y ∈ N , then there is a z ∈ N such that 10 ⋅⋅⋅ 0 x ⋅10 ⋅⋅⋅ 0 y = 10 ⋅⋅⋅ 0 z ,
s digits t digits u digits

Where we have u = inf( s, t ) −1 .


The proof of this generalization follows the proof of the first part of this problem.

2.24.
⎛ b⎞
We consider a numeric base b , and p a simple divisor such that ⎜⎜ p, ⎟⎟⎟ = 1 . Then
⎜⎝ p ⎠⎟
∀n ∈ N *, ∃ An = a 1...a n written in base b which is divisible by p n , with
ai ∈ {1, 2,..., p } ,1 ≤ i ≤ n .

Solution
We will apply the recurrence reasoning for n ∈ N * .
For n = 1 we have ∃ A1 = p which is divisible by p1 . (We observe that, because p | b ,
⎛ b⎞
it results that b = Kp, K ∈ Z ; 1 = ⎜⎜ p, ⎟⎟⎟ = ( p, K ) , also, all the digits of the numbers in the base
⎜⎝ p ⎟⎠
b belong to the set M b = {0,1, 2,..., p , p + 1,...., n −1} , and these are represented by one single
symbol (for example, if b > 10 , then the digits 10,11,… are noted by A, B,... ). Then
p ∈ M p = {1,2,..., p } and it is formed by a digit in base b ; ( p | b ⇒ p ≤ b = b ).

30
We suppose that the property is true for n , that is ∃ An = a 1...a n , written in the base b ,
which is divisible by p n , where ai ∈ M p , 1 ≤ i ≤ n . We’ll show that it is true for n + 1 .
Let An+1 = xa 1...a n with x ∈ M p written in base b . We determine a x for which An+1 is
divisible by p n+1 (it is sufficient to prove that there is such an x ).
n
An+1 = x ⋅ b n + a 1...a n = x ⋅ K n ⋅ p n + An = p n ( K n ⋅ x + t ) , where An = tp , t ∈ Z (from
the recurrence hypothesis).
∃ x ∈ M p such that K n x + t ≡ 0(mod p ) ⇔ K n ⋅ x ≡ −t (mod p ) .
⎛ b⎞
Because ⎜⎜ p, ⎟⎟⎟ = 1 = ( p, K ) we have ( p, K n ) = 1 . Then there exists the inverse of the element
⎜⎝ p ⎟⎠
−1
K n in rapport to the module p . The above congruence becomes: x ≡ −t ( K n ) (mod p) and we
chose the smallest x not null, that is: x ∈ M p .
(There exists x ∈ M p , because M p constitute a complete systems of residues modulo p.)

2.25.
m
⋅⋅ m ... m
Let n, m ∈ N * . We note an( m) = mm with n digits m , and bn( m ) = m...m n . For each
n
(m) (m)
n and m compare an with bn . Discussion. (All numbers are written in base 10.)

Solution
In the precedent conditions, we have:
Lemma 1. ∀n ∈ N *, ∀m ≥ 2 m 4 n > m...m
n
Proof: We use the recurrence method for n ∈ N * .
The case n = 1 implies m 4 > m which is true. We assume that the property is true for n
and we’ll prove it for n + 1 :
m 4( n+1) = m 4 n ⋅ m 4 > m...m ⋅ m 4 ≥ m...m ⋅16 = m...m 0 + m...m ⋅ 6 > m...m 0 + m = m...mm .
n n n+1 n n+1 n+1
(m) ( n+1)
Lemma 2. ∀n ≥ 3, ∀m ∈ N *, b n >4 m...m
n+1
2
Proof: m...m = m...m ⋅ m...m > m...mm because n ≥ 3 .
n n n n+1

m...m >n + 1 ≥ 4 because n ≥ 3 .


n
m ... m 4
bn( m ) = m...m n > m...m > 4( n+1) ⋅ m...mm .
n n n+1

31
Lemma 3: If there exists n0 ∈ N *, n0 ≥ 3 , such that an( 0m ) > bn(0m ) then ∀m ≥ 2, ∀n ≥ n0 we
have an( m ) > bn( m ) .
Proof: use the recurrence method for n ≥ n0 .
The case n = n0 is true by hypothesis. We assume that the property is true for n , and
we’ll prove it for n + 1 :
(m) (m) m ... m
an( m+1) = m an > m bn > m 4( n+1) ⋅ m...m > m...m n +1 = bn( m ) .
n+1 n+1

To prove these inequalities we will use the hypothesis from the recurrence from Lemma
2, respectively Lemma 1.
Lemma 4: ∀m ≥ 6, a3( m ) > b3( m ) .
Proof. Because m ≥ 6 and because of the results from the Lemmas 1 and 2, it results that:
m m = m 2 ⋅ m m−2 > 4 ⋅ 3 ⋅ m m−2 > 4 ⋅ 3 ⋅ 6 4 > 4 ⋅ 3 ⋅ mmm
a3( m ) = m m > m 4⋅3⋅mmm = (m 4⋅3 )
m mmm mmm
> mmm = b3( m ) .
m
⋅⋅ m ... m
We have: an( m ) = mm with n digits of m , bn( m ) = m...m n .
n

Case m = 1 . a1(1) = 1 = b1(1) .


1...1
an(1) = 1 < 1...1 n+1 = bn(1) , ∀n ≥ 2
n+1
(2)
Case m = 2 . a 2 = 22 < 2222 = b2(2)
a3(2) = 24 < 222222 = b3(2)
a4(2) = 216 < 22222222 = b4(2)
a5(2) = 265536 < 23⋅22222 < 2222222222 = b5(2)
Using Lemma 1, we obtain
265536 > 25 ⋅ 24⋅6 > 4 ⋅ 7 ⋅ 222222
Then
> 24⋅7⋅222222 = (24⋅7 )
65536 222222
a6(2) = 22 > 222222222222 = b6(2)
From the Lemma 3 it results that an(2) > bn(2) , ∀n ≥ 6 .
Case m = 3 a1(3) = 3 < 33 = b1(3)
a2(3) = 33 < 3333 = b2(3)
a3(3) = 327 < 333333 = b3(3)
Using Lemma 1 we obtain 327 > 33 ⋅ 34⋅4 ?16 ⋅ 3333
Then
a4(3) = 33 > 316⋅3333 = (34⋅4 )
27 3333
> 33333333 = b4(3)
From Lemma 3 it results an(3) > bn(3) , ∀n ≥ 4
Case m = 4 . a1(4) = 4 < 44 = b1(4)

32
a2(4) = 44 < 4444 = b2(4)
4
a3(4) = 44 = 4256 < 444444 = b3(4)
From Lemma 1 it results 4256 > 42 ⋅ 44⋅4 > 4 ⋅ 4 ⋅ 4444 .
Then a4(4) = 44 > 44⋅4⋅4444 = (44⋅4 )
256 4444
> 44444444 = b4(4) .
From Lemma 3 it results:
∀n ≥ 4, an(4) > bn(4)
Case m = 5 . a1(5) = 5 < 55 = b1(5)
a2(5) = 55 < 5555 = b2(5)
a3(5) = 55 = 53125 = (55 )
5 625
= 3125625 > 555555 = b3(5) .
From Lemma 3 we have ∀n ≥ 3, an(5) > bn(5) .
Case m = 6 . a1( m ) = m < mm = b1( m )
mm
a2( m ) = m m < mm = b2( m )
From Lemma 4 it results: a3( m ) > b3( m )
And from Lemma 3 we have an( m ) > bn( m ) , ∀n ≥ 3 and the problem is solved

33
MATHEMATICAL LOGIC

34
3.26.
Consider P, Qi 1 ≤ i ≤ n , logical propositions. Prove that the logical proposition
n n
" V ( P ∧ Qi ) ⇒ Λ ( P ∨ Qi )"
i =1 i =1
is always true

Solution
A logical proposition " A ⇒ B " is false only when A = 1 (true) and B = 0 (false). We’ll
prove that this situation does not exist.
n
If " V ( P ∧ Qi )" = 1 , then ∃i 0 ∈ {1,..., n} such that P ∧ Qi = 1 , that is P = 1 and Qi0 = 1 .
i =1
Then:
P ∨ Qi = 1, ∀i ∈ {1,..., n} since P = 1 ,
Then
n
" Λ ( P ∨ Qi ) = 1" ≠ 0
i =1
( ∧ means “and”, ∨ means “or”).

35
3.27.
Show that if the logical propositions " A1 ⇒ A2 " and " B1 ⇒ B2 " are true, then the logical
propositions " A1 ∧ B1 ⇒ A2 ∧ B2 " and " A1 ∨ B1 ⇒ A2 ∨ B2 " are also true.

Solution

A1 A2 B1 B2 A1 ⇒ A B1 ⇒ B2 A1 ∧ B1 A2 ∧ B2 A1 ∧ B1 A1 ∨ B1 A2 ∨ B2 A1 ∨ B1
⇒ ⇒
A2 ∧ B2 A2 ∨ B2

0 0 0 0 1 1 0 0 1 0 0 1
0 0 0 1 0 1 0 0 1 0 1 1
0 0 1 0 1 0 0 0 1 1 0 0
1 0 0 0 1 0 0 1 1 1 0 0
0 0 1 1 1 1 0 0 1 1 1 1
0 1 1 0 1 0 0 0 1 1 1 1
1 1 0 0 1 1 0 0 1 1 1 1
0 1 0 1 1 1 0 1 1 0 1 1
1 0 1 0 0 0 1 0 0 1 0 0
1 0 0 1 0 1 0 0 1 1 1 1
0 1 1 1 1 1 0 1 1 1 1 1
1 1 1 0 1 0 1 0 0 1 1 1
1 1 0 1 1 1 0 1 1 1 1 1
1 0 1 1 0 1 1 0 0 1 0 0
1 1 1 1 1 1 1 1 1 1 1 1
1 2 3 4 5 6 7 8 9 10 11 12

We note with “1” the true and with “0” the false. Immediately we observe that
" A1 ⇒ A2 " and " B1 ⇒ B2 " are true in the same time, it results that " A1 ∧ B1 ⇒ A2 ∧ B2 " and
" A1 ∨ B1 ⇒ A2 ∨ B2 " are true in the same time.

36
TRIGONOMETRY

37
4.28.
Prove the following formulae of transformation of the products of functions in sums:
1
1) cos α1 cos α2 ⋅⋅⋅ cos αn = n−1 ∑ cos (ε1α1 + ... + εn αn )
2 ε1 ,...,εn ∈τn
2)
p
(−1)
a) sin α2 ⋅⋅⋅ sin α2 p = 2 p−1 ∑ (−1) cos (ε1α1 + ... + ε2 p α2 p )
k

2 ε1 ,...,ε2 p ∈τ2 p
p
(−1)
∑ (−1) cos (ε1α1 + ... + ε2 p+1α2 p+1 )
k
b) sin α2 ⋅⋅⋅ sin α2 p+1 = 2p
2 ε1 ,...,ε2 p +1∈τ2 p +1

where
⎡m⎤
⎢ ⎥
⎢⎣ 2 ⎥⎦
τ m = ∪ (ε1 ,..., εm ) / εi1 = εi2 = ... = εik = −1 and ε j = 1 for
k =0

j ∉ {i1 ,..., ik } − {(ε1 ,..., εm ) / (−ε1 ,..., −εm ) ∈ τ m } .

Solution
The set τ m contains all the m -lepts (ε1 ,..., εm ) which have the components εi = ±1
arranged in all possible orders, but such that (ε1 ,..., εm ) ∈ τ m , then (−ε1 ,..., −εm ) ∈ τ m . Therefore
τ m has in total (Cm0 + Cm1 + ... + Cmm ) : 2 = 2m−1 elements, by Cmk , 0 ≤ k ≤ m we represent the
numbers of m-lepts such that the k components are equal to -1, and the rest of m − k are equal
to +1.
1) We’ll make the prove using the recurrence method on n .
The case n = 1 is evident. We suppose that the equality is true for n , and then prove it
for n + 1 :
1
(cos α1 − cos αn ) cos αn+1 = n−1 ∑ cos (ε1α1 + ... + εn αn ) cos αn+1 =
2 (ε1 ,...,εn )∈τn
1
= n ∑ cos ⎡⎢⎣(ε1α1 + ... + εn αn + αn+1 ) + cos (ε1α1 + ... + εn αn + εn+1 − αn+1 )⎤⎥⎦ =
2 (ε1 ,...,εn )∈τn
1
= n ∑ cos (ε1α1 + ... + εn+1αn+1 )
2 (ε1 ,...,εn )∈τn
2)
a) We apply the recurrence rational for p ∈ N * .
1
(−1)
If p=1 we have sin α1 sin α2 = [ cos(α1 + α2 ) − cos(−α1 + α2 ) ] which is true.
2
We suppose that the equality is true for p , we we’ll prove for p + 1 :
(sin α1....sin α2 p )sin α2 p+1 sin α2 p+2 =

38
p
(−1)
= 2 p−1 ∑ (−1)k cos (ε1α1 + ... + ε2 p α2 p )sin α2 p+1 cos α2 p+2 =
2 (ε ,...,ε )∈τ
1 jp 2p

p−1
(−1)
= ∑ (−1) k cos (ε1α1 + ... + ε2 p α2 p ) ⎡⎣cos(α2 p+1 + α2 p+2 ) − cos(−α2 p+1 + α2 p+2 ) ⎤⎦ =
22 p (ε1 ,...,ε2 p )∈τ2 p
p+1
(−1)
∑ (−1) k [ cos (ε1α1 + ... + ε2 p α2 p + α2 p+1 + α2 p+2 ) +
2 2 p+1 (ε1 ,...,ε2 p )∈τ2 p
+ cos (ε1α1 + ... + ε2 pα2 p − α2 p+1 − α2 p+2 ) −
− cos( ε1α1 + ... + ε2 p α2 p + α2 p+1 − α2 p+2 ) − cos(ε1α1 + ... + ε2 p α2 p + α2 p+1 − α2 p+2 ) ] =
p+1
(−1)
=
2 2 p+1 ∑ (−1) k cos(ε1α1 + ... + ε2 p+2α2 p+2 ) .
(ε1 ,...,ε2 p+2 )∈τ2 p+2
(We can prove easily the relations:
τ m+1 = {(ε1 ,..., εm , −1),(ε1 ,..., εm ,1) | (ε1 ,..., εm ) ∈ τ m } and
τ m+2 = {(ε1 ,..., εm , −1, −1),(ε1 ,..., εm , −1,1),(ε1 ,..., εm ,1, −1),(ε1 ,..., εm ,1,1) such that (ε1 ,..., εm ) ∈ τ m }
(We can also generalize).
b) The first method: by recurrence for p ∈ N * (similarly with above reasoning.
The second method:
p
(−1)
(sin α1....sin α2 p ) sin α2 p+1 = 2 p−1 ∑ (−1) k cos (ε1α1 + ... + ε2 p α2 p )sin α2 p+1 =
2 (ε1 ,...,ε2 p )∈τ2 p
p
(−1)
= 2p ∑ (−1) k [sin (ε1α1 + ... + ε2 p α2 p + α2 p+1 ) + sin (ε1α1 + ... + ε2 p α2 p − α2 p+1 ) ] =
2 (ε1 ,...,ε2 p )∈τ2 p
p
(−1)
= 2p ∑ (−1) k sin (ε1α1 + ... + ε2 p+1α2 p+1 ) .
2 (ε1 ,...,ε2 p+1 )∈τ2 p+1

4.29.
Let P( x ) = 2 x 2 − 1 . Prove that for n > 2 we have:
n−1
sin 2n x = 2n−12 x = ∏ p(... p( p (cos x ))...)
i=1 i times

Solution
a) We will prove by recurrence for n ∈ N * we have:
sin 2n x = 2n sin x cos x cos 21 x cos 22 x...cos 2n−1 x (1)
In the case n = 1 the property is evident
We suppose the equality true for n , and we prove that it is true also for n + 1 .

39
sin 2 ⋅ 2n... = 2sin 2n x cos 2n x...2n+1 sin x cos x...cos 2n−1 x cos 2n x
(we used the hypothesis of the recurrence)
b) We’ll prove by recurrence that for i ∈ N * we have:
cos 2i x = p(... p )( p(cos x ))...) (2)
2
In the case i = 1 we have cos 2 x = 2 cos x − 1 = p(cos x ) .
If we suppose the equality true for i we’ll prove that it is true also for i + 1 :
cos 2 ⋅ 2i x = 2 cos2 2i x − 1 = p(cos 2i x ) = p 2 x cos 2i x = p( p(... p )( p cos x ))...)
i times
i
Substituting (2) in (1) for all cos 2 x , it will result the equality that we need.

4.30.
Let s, n ∈ N * , K i , Pi rational with 1 ≤ i ≤ n , and the continuous functions fi , gi : R s → R
for 1 ≤ i ≤ n .
a) Find a method for solving the equation:
sin K1 f1 ( x1 ,..., xs ) cosP1 g1 ( x1 ,..., xs ) + ... + sin Kn f n ( x1 ,..., xs )...cosPn gn ( x1 ,..., xs ) = n
b) Find the necessary and sufficient condition that the equation from a) is equivalent to
the following system of equations:
⎧⎪sin K1 f1 ( x1 ,..., xs ) + ... + sin Kn f n ( x1 ,..., xs ) = n
⎪⎨
⎪⎪cos P1 g1 ( x1 ,..., xs ) + ... + cos Pn g n ( x1 ,..., xs ) = n

Solution
a) The right side of the equation is a sum of n terms, each belonging to [− 1,1] . Then
each term must be equal to 1, because if not we have s < n . Then the equation is equivalent to
the system:
sin Ki fi ( x1 ,..., xs ) cosPi gi ( x1 ,..., xs ) = 1 ,
which is equivalent to

⎪ sin Ki f i ( x1 ,..., xs ) = 1

⎨ Pi (1’)

⎪ cos g ( x ,..., x ) = 1
⎩ i 1 s

or

⎪ sin Ki f i ( x1 ,..., xs ) = −1

⎨ Pi (1”)

⎪ cos g ( x ,..., x ) = − 1
⎩ i 1 s

with i ∈ {1,..., n} ;
which are resolved normally, we obtain then an algebraic system
b) The system from b) is also equivalent with the system from (1’). Because the equations
from a) are equivalent to the system from b), therefore with (1’), we must eliminate the case (1”).

40
ri u
Then, if Ki = , Pi = i , ri , ti , ui , vi are integers, 1 ≤ i ≤ n , then for ∀i ∈ {1,..., n} , there exist
ti vi
at least one integer in {ri , ti , ui , vi } which is even.

41
GEOMETRY

42
5.31.
We design the projections M i of a point M on the sides Ai Ai+1 of a polygon A1... An .
2 2 2 2 2
Show that M 1 A1 + ... + M n An = M 1 A2 + ... + M n−1 An + M n A1 .

Solution
2 2 2 2
For all i we have: M i Ai − M i Ai+1 = MAi − MAi+1 . From which:

∑( M A − M i Ai+1 ) = ∑ ( MAi − MAi+1 ) = 0 .


2 2 2 2
i i
i i

5.32.
On a line we have the following points A1 , A2 ,..., An in this order.
⎡n⎤ ⎡ n + 1⎤
Let n1 = ⎢ ⎥ and n2 = ⎢ ⎥ . Prove that
⎣⎢ 2 ⎦⎥ ⎣⎢ 2 ⎦⎥
n2 n1

∑ i =1
Ai An+1 = ∑ Aj An+1− j
j =1

Solution
a) n = 2 K ⇒ n1 = n2 = K
We make the notation: Ai Ai+1 = xi , 1 ≤ i ≤ n − 1 (1)
Our relation becomes:
k k

∑i =1
Ai Ak +1 = ∑ Ai A2 k +1−i
i =1
From (1) we have:
k k

∑ ( xi + xi+1 + ... + xi+k ) = ∑ ( xi + xi+1 + ... + x2k−1 )


i =1 i =1
The left side is equal to:
x1 + x2 + x3 + ... + xk +
x2 + x3 + ... + xk + xk +1
.............................................
xk + xk +1 + ... + x2 k−1
which is equal to
x1 + 2 x2 + 3x3 + ... + kxk + ( k − 1) xk +1 + ( k − 2) xk +2 + ... + x2 k−1 (2)
The side from the right is equal to

43
x1 + x2 + x3 + ... + xk + xk +1 + x2 k−2 + x2 k−1 +
x2 + x3 + ... + xk + xk +1 + ... + x2 k−2 +
..................................................................
Which ia also equal to (2)
b)
n = 2k + 1 ⇒ n1 = k
n2 + k + 1
The proof is the same.

5.33.
Let ABC an arbitrary triangle and O the center of its inscribed triangle. On one of the
sides, BC , we take n points A1 ,..., An , in this order, such that the lines AA1 ,..., AAn divide the
angle BAC in n + 1 equal parts. In a similar mode we proceed for the sides CA and AB on
which we take the points B1 ,..., Bn respectively C1 ,..., Cn .
Prove that the point O belongs to the geometrical figure determined by the intersection
of the lines AAi , BBi , and CCi , i ∈ {1, 2,..., n} .

Solution

n + 1 ⎡ n + 1⎤
a) If =⎢ ⎥ = i then AAi , BBi , and CCi are the bisectrics of the angles A, B,
2 ⎢⎣ 2 ⎥⎦
and C because these divide the angles into two equal parts. Then O is thir intersections.
n +1
b) If ≠ i , then AAi , BBi , and CCi are not anymore bisectrics. These intersect each
2
other in pairs forming a triangle that is in the interior of the triangle ABC . We obtain the small
triangle from the figure (1).

Ci
F
E
O Bi
B
Ai
D
C

Fig. (1)

44
Let AD, BE , CF be the bisectrics of the angles A, B, C . These can be on the left side of
the lines AAi , BBi , and CCi ( as we look at the sides starting from A to Ai ) or to the right. In
figure (1) we have the case where the lines are on the left side. We will have the same proof for
other cases).
Because AD is on the left side of AAi and O ∈ AD , it results that O ∈ AAC i . Because
BE is on the left side of BBi and that O ∈ BE we have that O ∈ BBi A . The same, O ∈ CCi B .
Then O ∈ AAC i ∩ BBi A ∩ CCi B .

5.34.
Given n lines that intersect two by two and are not on the same plane three by three,
prove that these lines pass through the same point.

Solution
We’ll consider the case n = 3 .
The lines d1 and d 2 intersect in M ; d 3 intersects d1 in M ' and d 2 in M " . If M ' ≠ M
and M " ≠ M , then the three lines are on the same plane, which is absurd. Therefore
M '≡ M"≡M .
The case n > 3 is reduced to the previous case.
Among the n lines we choose arbitrary three, that satisfy the conclusion. Among these
three lines we take two arbitrary ones and one among the n − 3 left lines. We obtain three lines
that pass through the same point, which is also M .
The rational continues the same until we finish with all the lines.

5.35.
Let n points A1 ,..., An in a plane, n ≥ m ≥ 3 , such that m of these points form a regular
polygon. Prove that n = m .

Solution
1) Case m > 3 . Let m − 1 points in this plane. We add a new point and we construct a
regular polygon with m sides.
A3

A1 A2

A4

45
Each regular polygon can be inscribed in a circle. We consider for beginning that the m − 1
points are placed on the circumference of a circle. Evidently, the other point (which has been
added) belongs to the same circle, and it is well determined, because the circle is divided in equal
arcs. But, through m −1 ≥ 3 points passes only one circle. Then to m − 1 points we can add one
single point to form a regular polygon (of m sides).

More, the number of points that have the property from above cannot be larger than m ;
but, also, it cannot be less than m either because we cannot form a regular polygon with m
sides. From here we have that n = m
2) The case m = 3 . Then m − 1 = 2 . Taking two distinct points, to form an equilateral
triangle, we can find: let one point in a semi-plane, let one point on another semi-plane (the
semi-planes determined by the line that unite the two points and divide the plane in two parts).
If A1 A2 A3 and A1 A2 A4 are equilateral, then A1 A3 A4 will not be equilateral.
The proof, now becomes similar to the case 1.

5.36.
We consider a polygon (which has at least 4 sides) circumscribed to a circle, and D the
set of the diagonals and of the lines that connect the contact points of two non adjacent sides.
Then D contains at least three concurrent lines.

Solution
Let n be the number of sides. If n = 4 , then the two diagonals and the two lines that
connect the points of contact of the two non adjoined sides are concurrent (in conformity to the
Newton’s theorem)

P Aj+1

Aj-1 B1
Ai

B2
``` O
B4 Ai-1
R,
B3 Aj+1

Aj

The case n > 4 will resume to anterior case: we consider the arbitrary polygon A1... An
(see the figure) circumscribed to a circle and we select two segments Ai , Aj , i ≠ j , such that
Aj Aj−1 ∩ Ai Ai+1 = P , Aj Aj+1 ∩ Ai Ai−1 = R

46
Let Bh , h ∈ {1, 2,3, 4} be the contact points of the quadrilateral PAj RAi with the circle in
the center O .
Due to Newton’s theorem, the lines Ai Aj , B1B3 , B2 B4 are concurrent.

5.37.
In a triangle ABC let AA ', BB ', CC ' be the ceviene that intersect in the point P .
Compute the minimum value of the expressions:
AP BP CP
E ( P) = + +
PA ' PB ' PC '
and
AP BP CP
F ( P) = ⋅ ⋅
PA ' PB ' PC '
where
A ' ∈ [ BC ], B ' ∈ [CA], C ' ∈ [ AB ] .

Solution
We apply the Van Aubel theorem three times for triangle ABC , and we have:
AP AC ' AB '
(1) = +
PA ' CB ' B 'C
BP BA ' BC '
(2) = +
PB ' A' C C'A
CP CA ' CB '
(3) = +
PC ' A' B B' A
If we add these three relations and if we make the following notations
AC ' AB ' BA '
= X >0 , = Y > 0, = Z >0
CB ' B 'C A' C
then we obtain:
⎛ 1⎞ ⎛ 1⎞ ⎛ 1⎞
E ( P ) = ⎜⎜ X + ⎟⎟⎟ + ⎜⎜Y + ⎟⎟⎟ + ⎜⎜ Z + ⎟⎟⎟ ≥ 2 + 2 + 2 = 6 .
⎜⎝ X⎠ ⎝ ⎜ Y⎠ ⎝ ⎜ Z⎠
The minimum value will be obtained when X=Y=Z=1, that is when P will be the center
of gravity of the triangle.
Multiplying the three relations will find that
⎛ 1⎞ ⎛ 1⎞ ⎛ 1 ⎞ YZ X
F ( P ) = ⎜⎜ X + ⎟⎟⎟ + ⎜⎜Y + ⎟⎟⎟ + ⎜⎜ Z + ⎟⎟⎟ + + ≥8
⎜⎝ X⎠ ⎝ ⎜ Y⎠ ⎝ ⎜ Z ⎠ X YZ

47
5.38.
If the points A1 , B1 , C1 divide the sides BC , CA , respectively AB of a triangle in the
rapport k, determine the minimum of the following expressions:
2 2 2
AA1 + BB1 + CC1

Solution
We suppose that k > 0 , because we work with distances.
BA1 = k BC ; CB1 = k CA ; AC1 = k AB .
We apply three times the Stewart theorem in the triangle ABC , with the segments
AA1 , BB1 respectively CC1 :
2 2 2 2 2 3
AB ⋅ BC (1− k ) + AC ⋅ AC ⋅ k − AA1 ⋅ BC = BC (1− k )k
where:
2 2 2 2
(1) AA1 = (1− k ) AB + k AC − (1− k )k BC
2 2 2 2
(2) BB1 = (1− k ) BC + k BA − (1− k )k AC
2 2 2 2
(3)CC1 = (1− k ) CA + k CB − (1− k )k AB
By adding these three equalities we find:
AA1 + BB1 + CC1 = (k 2 − k + 1) AB + BC + CA
2 2 2
( 2 2 2
),
1
Which take the minimum value when k = , that is the case when the three lines from
2
the problem hypothesis are the medians of the triangle.
3
The minimum is
4
2
( 2
AB + BC + CA .
2
)

5.39.
In the triangle ABC we construct the concurrent lines AA1 , BB1 , CC1 such that
AB12 + B1C 2 + C1 A2 = AB12 + BC12 + C1 A2 and one of them is the median
Prove that the other two lines are the same medians when the triangle ABC is isosceles.

Solution
Suppose that AA1 is the median, without diminishing the problem’s generality, then
A1 B = A1 C , and the relation from the hypothesis becomes:
B1C 2 + C1 A2 = AB12 + BC12 (1)
From the concurrency of the lines AA1 , BB1 , CC1 and from the Menelaus’ theorem it results that

48
AB1 AC1
= (2)
B1C C1B
AB1
We make the notation: = k , k > 0 , from which we have:
B1C
B1C 2 + k 2C1B 2 = k 2 BC12 + BC12 .
Consequently (k 2 −1)C1B 2 − BC12 = 0 , and from here we find k = 1 , or C1B = B1C .
If k=1 then AB1 = B1C and AC1 = C1B then consequently BB1 , CC1 are medians .
If C1B = B1C from (2) it results that AB1 = AC1 , consequently AB = AC , and the
triangle ABC is isosceles.

5.40.
In a triangle we construct the ceviane AA1 , BB1 , CC1 that intersect in a point P . Prove
that
PA PB PC AB ⋅ BC ⋅ CA
⋅ ⋅ =
PA1 PB1 PC1 A1B ⋅ B1C ⋅ C1 A

Solution

A
B1
C1

C
B A1

In the triangle ABC we apply the Ceva theorem


AC1 ⋅ BA1 ⋅ CB1 = − AB1 ⋅ CA1 ⋅ BC1 (1)
In the triangle AA1B cut by the transversal AA1 , we apply also the Menelaus’ theorem:
AC1 ⋅ BC ⋅ A1P = AP ⋅ A1C ⋅ BC1 (2)
In the triangle BB1C cut by the transversal AA1 , we apply also the Menelaus’ theorem:
BA1 ⋅ CA ⋅ B1 P = BP ⋅ B1 A ⋅ CA1 (3)
We apply one more time the Menelaus’ theorem in the triangle CC1 A cut by the transversal BB1 :
AB ⋅ C1P ⋅ CB1 = AB1 ⋅ CP ⋅ C1B (4)
We divide each relation (2), (3) and (4) by the relation (1), and we obtain:

49
PA BC B1 A
= ⋅ (5)
PA1 BA1 B1C
PB CA C1B
= ⋅ (6)
PB1 CB1 C1 A
PC AB A1C
= ⋅ (7)
PC1 AC1 A1B
We’ll multiply (5) by (^) and by (7), and we obtain
PA PB PC AB ⋅ BC ⋅ CA AB1 ⋅ BC1 ⋅ CA1
⋅ ⋅ = ⋅ ,
PA1 PB1 PC1 A1B ⋅ B1C ⋅ C1 A A1B ⋅ B1C ⋅ C1 A
but the last fraction is equal to 1 in conformity to the Ceva’s theorem.

5.41.
Let the triangle ABC which has all the angles acute and we consider A ' B ' C ' the
triangle formed by the legs of its heights.
In which conditions the following expression is maximum?
A' B ' ⋅ B 'C ' = B 'C ' ⋅ C ' A' + C ' A' ⋅ A' B '

Solution
We have
ABC ∼ A ' B ' C ' ∼ AB ' C ' ∼ A ' BC ' (1)
We note:
BA ' = x , CB ' = y , AC ' = z
It results that:

A
b-y
z B
C

y
c-z

B x A’ a-x C

A ' C = a − x, B ' A = b − y , C ' B = c − z

50
BAC = B ' A ' C = BA ' C '
ABC = AB ' C ' = A ' B ' C
BCA = BC ' A ' = B ' C ' A
Qualities it results relation (1).
A' C ' x
A ' BC ' ∼ A ' B ' C ⇒ = (2)
a−x A' B '
A' C ' c− z
A ' BC ' ∼ AB ' C ' ⇒ = (3)
z B 'C '
B 'C ' b− y
AB ' C ' ∼ A ' B ' C ⇒ = (4)
y A' B '
From the relations(2), (3) and (4) we conclude that the sum of the products from the
hypothesis is equal to:
1 a b c
x ( a − x ) + y (b − y ) + z ( c − z ) = ( a 2 + b2 + c 2 ) − ( x − ) 2 − ( y − ) 2 − ( z − )2
4 2 2 2
a b c
which reaches its maximum when x = , y = , z = that is when the heights’ legs fall in the
2 2 2
middle of the sides, therefore the ABC is equilateral. The maximum of the expression is
1 2
4
( a + b2 + c 2 ) .

5.42.
Let’s consider A1 ,..., An n distinct points on the circumference of a circle with the center
O and radius R .
180°
Sow that there exist two points Ai and Aj such that OAi + OAj ≥ 2 R cos .
n

Solution

An-1

Aj
An R

O α M

R
A1 Ai

51
A2
Because
S = AOA 1 2 + A2OA3 + ... + An−1OAn + AnOA1 = 360° and

∀i ∈ {1, 2,..., n} , AOA


i i +1 > 0° ,

360°
It results that there exist at least one angle AOA
i j ≤ (if not it results that )
n
360°
S> ⋅ n = 360° ).
n
OAi + OA j = OM ⇒ OAi + OA j = OM .
The quadrilateral OAi M + Aj is a rhomb.
While α is very small, OM is very big.
360°
Because α ≤ , it results that
n
α 180°
OM = 2 R cos ≥ 2 R cos .
2 n

5.43.
Determine the maximum number of points which can be found on the circumference of a
circle, such that the distance between two arbitrary points is greater or equal to the circle’s
radius.

Solution
The side of a regular hexagon inscribed in a circle has the same dimension as the radius
of the respective circle. Therefore there are at least 7 points on a circle that have the property
from the hypothesis, one point in the center of the circle and 6 points on the circumference such
that the six points constitute the vertexes of the regular hexagon inscribed in the circle.
The selected 7 points are taken in an optimal way. For example, if we want to construct
the set of point that have the property from the problem’s hypothesis it would not be at all
optimal of taking the first point different from the center of the circle, and not on the
circumference.

52
Therefore in the geometric figure included here, taking C1 in the interior of the circle and
different from the center, then in the shaded portion (which is a circle of the same radius like the
initial circle and with the center in C1 , intersects the other one) it is not possible to take any other
point. Therefore the best is to have the shaded portion as small as possible. It will result that C1
must be on the circumference. From this will result that the other points will be: 5 on the
circumference such that the 6 points will constitute a regular hexagon and the other in the center
of the circle. Therefore we have constructed 7 points.

5.44.
How many points we can find in a sphere (and on its surface), such that the distance
between any two of them to be greater or equal with the radius.

Solution
We consider the large circles of the sphere, determined by the plane AOA
1 4 , where O is

the center of the sphere.

A5 A4

A6
O A3
A2

A1 N

On its circumference we take the points A1 , A2 ,..., A6 such that we get a regular hexagon
- therefore, the distance Ai Aj ≥ than the ray of the sphere, for which i ≠ j . We construct a
plane A 2 A5 MNO perpendicular on the plane AOA 1 4 which cuts the sphere by the circle A 2 A5 MN .

On its circumference we take also 6 points which constitute a regular hexagon. Then, we
construct the third big circle of the sphere, determined by A 3 , A6 , M , N . The same, on the
circumference of this last circle we take 6 points, which are the vertexes of a regular hexagon,
among which are the points A 3 , A6 . Etc.
We have in total 6+4+0=10 points, and if we add the center of the sphere we obtain 11
points which keep the property from the hypothesis.
This method of constructing the points is the optimal one. If we start the construction of
the points, for example taking a point A which does not belong to surface of the sphere, then the

53
sphere of center A and having the same ray will intersect a large zone of the initial sphere, but
with the condition is that this zone is the smallest possible. Then A belongs to the surface. And
the demonstration continues in the same way.

5.45.
Given n distinct points in a plane, connected two by two through a line:
a) What is the maximum number of lines which we can construct with these points?
b) If only m points 1 ≤ m ≤ n , are collinear, how many distinct lines are there?
(n − 2)(n + 1)
c) Prove that we cannot have lines, regardless of how we arrange the n
2
distinct points.

Solution
Let A1 ,..., An the n distinct points.
a) If they are three to three non-collineart, then we can form all possible lines
n (n −1)
Ai Aj with i < j and (i , j ) ∈ {1,..., n} , therefore lines.
2
2
b) If A1 ,..., Am are collinear, then the lines Ah Ak with h < k and (h, k ) ∈ {1,..., n} are the
same, and constitute a single line. Then it remains:
n (n − 1) m (m − 1) n2 − m2 − n + m + 2
− +1 =
2 2 2
distinct lines.
(n − 2)(n + 1) n (n −1)
c) = −1 .
2 2
n (n −1)
If the n points are three to three non-collinear, we saw that we have: distinct
2
m (m −1)
lines. If m points are collinear, we have −1 lines less. But
2
m (m − 1) m (m − 1)
−1 ⇔ −1 − m − 4 = 0 which does not have a natural solution.
2 2
For example, if we have 3 collinear points, we eliminate 2 lines from the total of
n (n −1)
, but not a line as it should.
2

5.46.
Given n distinct points in a plane, three to three non collinear A1 ,..., An , find the locus of
the points M ≠ A i , 1 ≤ i ≤ n , such that it doesn’t matter which line that passes through M and

54
which doesn’t contain any point Ai (1 ≤ i ≤ n ) divides the plane in two semi-planes that contain
⎡n⎤ ⎡ n + 1⎤
the ⎢ ⎥ and the other ⎢ ⎥ points.
⎢⎣ 2 ⎥⎦ ⎢⎣ 2 ⎥⎦

Solution
We note Π the plane that contains the points A1 ,..., An and let α be the locus we are
looking for.
1) If n = 1 , then obvious α = Π− { A1 } .
2) If n = 2 , α = [ A1 , A2 ]− { A1 , A2 } , where [ A1 , A2 ] represents the segment of line which
unite the points A1 , A2 .
3) n > 2 .
a) n = 2 k . Let d1 the line that passes through A1 and through a point As1 ,
2 ≤ s1 ≤ n , such that on a side and the other side of the line d1 we find to be k − 1 points
⎡ 2k ⎤ ⎡ 2k + 1 ⎤
A 2 ,..., As1−1 , As1+1 ,..., An . We have ⎢ ⎥ = ⎢ ⎥ = k . Obviously α is included in ⎡⎢⎣ A1, As1 ⎤⎥⎦ . We
⎢⎣ 2 ⎥⎦ ⎢⎣ 2 ⎥⎦
n
proceed the same for all the points Ai , 1 ≤ i ≤ n , and we find that α = ∩ ⎡⎣⎢ Ai , As1 ⎤⎦⎥ . Then, if all
i =1
these segments intersect in one point, then that point would be α l if not α = Φ .
⎡n⎤ ⎡ n + 1⎤
b) n = 2k + 1 . We have ⎢ ⎥ = k and ⎢ ⎥ = k + 1 . For A1 we construct the
⎢⎣ 2 ⎥⎦ ⎢⎣ 2 ⎥⎦
triangle A1 Au1 Av1 , where Au1 is such that the line A1 Au1 divides the plane in two semi-planes, one
containing k − 1 , the other k points among the points Ai ; in the same time that Av1 is such that
the line A1 Av1 divides also the plane in two semi-planes, one containing k and the other k − 1
points among the points Ai . Evidently α =⊆ A1 Au1 Av1 . We continue the rational for all the points
n
Ai , 1 ≤ i ≤ n , and we find that α = ∩ Ai Au1 Av1 .
i=1

5.47.
Prove that a sphere cannot be included in the union of two spheres whose rays are strictly
smaller than that of the sphere itself.

Solution
Let’s S be the sphere, C the big circle, r the ray of the sphere (implicitly r is the ray
of the circle C ).
By reduction ad absurdum, let S1 and S2 the spheres that comprise this sphere and such
that they are strictly inferior to S .

55
We note C1 (respectively C2 ) the ray of the sphere S1 (respectively S2 ), [implicitly r1
(respectively r2 ), is the ray of circle C1 (respectively C2 )].
We intersect S with S1 .
a) S ∩ S1 = {P} (one common point)
Let O the center of the sphere S . We construct a plane Π that contains the ray OP .
Π ∩ S = C , C ∩ S1 = {P} , then S2 ⊃ C − {P} , that means r2 ≥ r . Contradiction.
b) S ∩ S1 = CSS 1 (a circle). It results that its ray rC SS ≤ min {r1 , r} = r1 < r .
1

Then there will exist a big circle C of the sphere S which has the property that
C ∩ S1 = Φ . Then S2 ⊃ C , and r2 ≥ r , which is absurd.
c) The case S1 ⊂ C and the surface of S1 does not intersect the surface of S , then it will
exist a big circle C of the sphere S such that C ∩ S1 = Φ . Then S2 ⊃ C , and then r2 ≥ r , which
is also absurd.
d) The same prove when S ∩ S1 = Φ and S1 ⊄ S .

56
CALCULUS

57
6.48.
Let’s consider the numbers k1 ,..., k p that form an arithmetic sequence. Prove that if
a1 ,..., an form an arithmetic sequence (respectively geometric) then ak1 ,..., akm constitute an
arithmetic sequence (respectively geometric).

Solution
ki = k1 + (i −1) r1 , 1 ≤ i ≤ p and r1 is the ratio of this arithmetic sequence.
a) When a1 ,..., an form an arithmetic sequence, then

aki = ak1 + (ki − k1 ) r1 = ak1 + rr1 (i − 1) ,


where 1 ≤ i ≤ p and r is the ratio of the sequence a1 ,..., an . Then ak1 ,..., ak p constitute also an
arithmetic sequence of ratio rr1 .
b) When a1 ,..., an constitute a geometric sequence, with the ratio q , then
i−1
aki = ak1 qki −k1 = ak1 ( q r1 ) , where 1 ≤ i ≤ p . Then ak1 ,..., ak p constitute a geometric
sequence with the ratio q r1 .

6.49.
Let xn and yn natural sequences such that xn = ayn , ∀n ∈ N * , and a ≠ 1 . From the
arithmetic progression b1 , b2 ,... we will eliminate the terms of rank xn , n ∈ N * . Prove that
among the remaining terms there exists an arithmetic sub-progression.

Solution
We observe that if the naturals numbers i1 ,..., is constitute an arithmetic progression then
bi1 ,..., bis has the same property, since
2bi j = 2 ⎡⎣b1 + (i j − 1) r ⎤⎦ = 2b1 + (2i j − 2) r = 2b1 + (i j−1 + i j+1 − 2) r = (b1 + (i j−1 − 1) r ) + (b1 + (i j+1 − 1) r ) = bi j−1 + bi
2bi j = 2 ⎡⎣b1 + (i j −1)r ⎤⎦ = 2b1 + (2i j − 2) r =
= 2b1 + (i j−1 + i j+1 − 2) r = (b1 + (i j−1 − 1) r ) + (b1 + (i j+1 − 1) r ) = bi j−1 + bi j+1
(We used 2i j = i j−1 + i j+1 ). Then we can replace b1 , b2 ,... by 1, 2,....
We must construct an arithmetic progression a1 , a2 ,.. such that ai+1 ≠ x n , ∀ (i , n ) ∈ N × N * .
A) The case a = 0 is trivial
B) Let a ≠ 0 .
ai+1 = a1 + ir , where r is the ratio. Then a1 = ?, r = ? such that
a1 + ir ≠ ayn , ∀ (i , n ) ∈ N × N * (1)

58
ayn − a1 1
From here we have i ≠ . Because i ∈ N , we put the condition: (ayn − a1 ) ∈ Z .
r r
We take r = a ≥ 2 (because a ≠ 0, a ≠ 1 and xn and yn are the natural sequences, then a ∈ N
ay − a + 1 1
or a1 = a −1 . It results that i ≠ n = yn −1 + ∈ N and the relation (1) is verified.
a a

6.50.
n
Prove that ( f1 .... f n ) ' = ∏ f i ' fi+1 ... f n .
i=1

Solution
For i = 1 we have an immediate result.
We suppose that the equality is true for i = n − 1 . Then
( f1 .... f n ) ' = ( f1 ( f 2 ... f n )) ' = ( f1 ' ( f 2 ... f n )) ⋅( f 2 ... f n ) ' =
n n
= ( f1 ' f 2 ... f n ) ⋅ ∏ f i ' f i+1 ... f n = ∏ f i ' f i+1 ... f n .
i =2 i=1

6.51.
Given the sets Φ ≠ A ⊂ M 1 ⊂ M 2 , where M 1 is everywhere dense in M 2 , and inf A ≅ in
M 2 . Then there exist an inf A in M 1 if and only if α ≤ M 1 . The same question for sup A .

Solution
The sufficiency.
If inf A = α in M 2 and α ∈ M 1 ⊂ M 2 , then evidently inf A = α in M 1 .
The necessity:
Let α ' = inf A in M 1 . Then α ' ∈ M 1 . We know that inf A = α in M 2 by hypothesis.
1) If α ' > α , then inf A = α ' ≠ α in M 2 . Contradiction.
2) If α ' < α , because M 1 is everywhere dense in M 2 , it results that there exists γ ∈ M 1
such that α ' < γ < α . If γ ∈ A , then α is not equal to inf A in M 2 (contradiction); then γ ∈ A .
If there exist γ ' ∈ (α ', γ ) such that γ ' ∈ A , then α ' ≠ inf A in M 1 . Contradiction.
Therefore:
3) α ' = α , that is α ∈ M 1 .
The proof for sup A is done the same way.

59
6.52.
Show that, given a natural number n > 0 and a natural number T > 0 , there exists a
T
function f : R → R with a period T and such that f has the period . In this case, if f is
n
continue prove that f is null in at least n − 1 points in the interval of length T .

Solution
Let’s consider n ∈ N *, T ∈ R+* and the function f : R → R , of a period T , which has
the following graphic representation.:

T T T
2 3 ……………. ( n −1)
n n n
(1)
T
O T
n

where all n semi-circles of (1) are equal among them.


The function f : R → R will have the following representation:

………………………….
(2)
T T T T
O 2 3 ………… ( n −1) T x
n n n n

T
Then its period is
n
Evidently, there exist an infinity of such functions, because we can replace the semi-
circles of (1) by other curves as long as the property from the hypothesis will be satisfied.
We must prove the second point:
The case n = 1 is banal. Let’s consider n > 1 . Let k the number of points for which f
is null in the interval of length T . But k is non-null, because otherwise it would result that

60
f = f or f = − f on R , that means that f has the period T . Therefore k ≥ 1 .
f ( x) = 0 ⇔ f ( x) = 0 .
(3) Because f ( x ) is null more than one times (we have k ≥ 1 ), it results that f is null in an
T
interval of length or in the interior of that interval or at one extremity of the interval. But in
n
T
an interval of length T there are n intervals of length . Therefore f will be zero on at least
n
n − 1 times in an interval of length T , and therefore knowing (3) it results the last question of
the problem.

6.53.
Let’s consider the positive functions f1 ,..., f n on an interval I such that they vary on the
same direction on this interval.
Then f1 ⋅ ... ⋅ f n varies on the same direction on the interval I .

Solution
We consider that all functions f i are increasing. (Analog prove if all f i are decreasing)
We will use the recurrence rational.
For i = 2 . Let x1 < x2 .
( f1 f 2 )( x2 ) − ( f1 f 2 )( x1 ) f 2 ( x2 ) − f 2 ( x1 ) f ( x ) − f1 ( x1 )
= f1 ( x1 ) + f 2 ( x2 ) 1 2 ≥0
x2 − x1 x2 − x1 x2 − x1
Therefore, f1 f 2 is a an increasing function on I .
We suppose that f1 ⋅ ... ⋅ f n−1 is increasing, then f1 ⋅ ... ⋅ f n−1 ⋅ f n is increasing, because we
can note f1 ⋅ ... ⋅ f n−1 = g which is positive and increasing by the recurrence’s hypothesis, and
f1 ⋅ ... ⋅ f n−1 ⋅ f n = gf n which is increasing concludes the proof for i = 2 .

6.54.

Let n a natural number and not null.


a) Determine the functions f : R → R , odd, derivable 2n times, such that the derivative
order 2n are non negative.
2)Determine the functions g : R → R , even, derivable 2n − 1 times, such that the
derivative of the order 2n − 1 are not negative.

61
Solution
( 2 n) 2 n+1 (2 n)
a) f ( x) = − f (−x ), ∀x ∈ R . It results that f ( x ) = (−1) f (−x ) .
( )
2n ( )
2n ( ) 2n
But f ( x ) ≥ 0 and f (−x) ≤ 0 , which implies that f ( x ) = 0 on R .
By integration 2n times, we obtain:
f ( x ) = a2 n−1 x 2 n−1 + a2 n−2 x 2 n−2 + ... + a1 x + a0
with ai ∈ R , 0 ≤ i ≤ 2n −1 .
Since f is odd, it results:

a2 n−1 x 2 n−1 + a2 n−2 x 2 n−2 + ... + a1 x + a0 = a2 n−1 x 2 n−1 − a2 n−2 x 2 n−2 + ... + (−1)i+1 ai xi + a1 x1 − a0 .
We obtain: a2 n−2 = a2 n−4 = ... = a0 . Therefore f ( x ) = a2 n−1 x 2 n−1 + a2 n−3 x 2 n−3 + ... + a1 x
b) g ( x) = g (−x ) , ∀x ∈ R . It results that g (2 n−1) ( x ) = (−1)(2 n−1) g (2 n−1) (−x ) , which
implies that g (2 n−1) ( x ) = 0 on R .
By integrating 2n times, we have g ( x) = b2 n−2 x 2 n−2 + b2 n−3 x 2 n−3 + ... + b1 x + b0 with
bi ∈ R, 0 ≤ i ≤ 2n − 2 .
Because g is even, it results that:
g ( x ) = b2 n−2 x 2 n−2 + b2 n−3 x 2 n−3 + ... + b1 x + b0 =
= b2 n−2 x 2 n−2 − b2 n−3 x 2 n−3 + ... + (−1)i bi x i + ... + (−1)b1 x − b0 .
We obtain: b2 n−3 = b2 n−5 = ... = b1 = 0 , then g ( x ) = b2 n−2 x 2 n−2 + b2 n−4 x 2 n−4 + ... + b2 x 2 + b0 .

6.55.
A function f : R → R admits a symmetry center if and only if it exist two real constants
a, b such that the function g ( x ) = f ( x + a ) − b is odd.
In these conditions, the symmetry center is for coordinates (a , b ) .

Solution
The necessity.
Let C (α, β ) the center of symmetry. We raise a = α, b = β . We execute a translation of
axes, by moving the origin in C (a, b) . The formulae of the change of the reference system
CXY to CX ' Y ' are
⎧⎪⎪ x ' = x − a ⎧⎪⎪ x = x '+ a
⎨ ⇔ ⎨
⎪⎪⎩ y ' = y − b ⎪⎪⎩ y = y '+ b
Then y = f ( x ) becomes y '+ b = f ( x '+ a ) m where y ' = f ( x + a ) − b . We make the notation
g ( x ' = f ( x '+ a ) − b, g : R → R . The function g admits a center of symmetry, which is the
same with the axes origin. Then g is odd.
The sufficiency.

62
g being odd, it results that g admits the axes origin as center of symmetry.
We execute the axes’ translation, moving the origin in C "(−a, −b) .
The formulae of the change of the reference system CXY to CX " Y " are
⎧⎪⎪ x " = x + a ⎧⎪⎪ x = x "− a
⎨ ⇔ ⎨
⎪⎪⎩ y " = y + b ⎪⎪⎩ y = y "− b
Then y = g ( x ) = f ( x + a ) − b becomes y "− b = f ( x "− a + a ) − b , that is y " = f ( x ") .
Because g admits the symmetry center O (o, o) in the reference system CXY , this implies then
that f admits the symmetry center O(a, b) in the reference system O " X " Y " .

6.56.
In an system of orthogonal axes, the function f : R → R has an axis of symmetry if and
only if there exists a real constant such that the function g ( x) = f ( x + a) , is even.
In this condition, the symmetry axis has the equation x = a .

Solution
The necessity.
Let x = α be the symmetry axis of function f . We set x = a . We execute a translation
of axes, by moving the origin in O '(a,0) . The equations of change from the (OXY ) to
(O ' X ' Y ') are:
⎧⎪⎪ x ' = x − a ⎧⎪⎪ x = x '+ a
⎨ ⇔ ⎨
⎪⎪⎩ y ' = y ⎪⎪⎩ y = y '
Then y = f ( x ) becomes y ' = f ( x '+ a ) = g ( x '), g : R → R . f admits as axis of
symmetry the line x = a . Il results that g admits as axis of symmetry the line x ' = 0 (in
(O ' X ' Y ') ), that is the axis O ' Y ' .
Therefore g is even.
The sufficiency:
Because g is even, we have that g admits the axis OY as axis of symmetry.
We’ll execute a translation of axes, by moving the origin in O "(−a,0) . The movement of
(OXY ) in (O " X "Y ") is done by:
⎧⎪⎪ x " = x + a ⎧⎪⎪ x = x "− a
⎨ ⇔ ⎨
⎪⎪⎩ y ' = y ⎪⎪⎩ y = y "
Then y = g ( x ) = f ( x + a ) becomes y " = f ( x ") . g admits as axis of symmetry the line
x = 0 , from which it results that f admits as axis of symmetry the line x " = a .

63
6.57.
We consider the continuous functions A, B, C : I → R , where I is an interval on R ,
with A( x ) ≤ C ( x ) ≤ B( x ) , ∀x ∈ I . Let’s consider x1 , x2 ∈ I , x1 < x2 and A( x1 ) = f ( x1 ) ,
B ( x2 ) = f ( x2 ) , where f is a continuous function on [ x1 , x2 ] .
Prove that there exist x3 ∈ [ x1 , x2 ] such that C ( x3 ) = f ( x3 ) .

Solution
We’ll use the absurd method.
By reduction ad absurdum we suppose that the conclusion is not true, then ∀x ∈ [ x1 , x2 ]
we have that C ( x) ≠ f ( x) , or C ( x ) − f ( x ) ≠ 0 . C and f being continuous on [ x1 , x2 ] , it results
that C − f is continuous on [ x1 , x2 ] . Then: ∀x ∈ [ x1 , x2 ] , C ( x ) − f ( x ) > 0 , or ∀x ∈ [ x1 , x2 ] ,
C ( x) − f ( x) < 0 .
We consider the first situation (the second will be similar).
Because A( x ) ≤ C ( x ) ≤ B( x ) on I , we have: A( x) − f ( x) ≤ C ( x) − f ( x) ≤ B( x) − f ( x )
on [ x1 , x2 ] . Therefore C ( x2 ) − f ( x2 ) ≤ B( x2 ) − f ( x2 ) = 0 , which is a contradiction.
Then we have the conclusion tha there exist x3 ∈ [ x1 , x2 ] such that C ( x3 ) = f ( x3 ) is true.

6.58.
Find the real numbers a , b, c, such that
a (2 x 3 − x 2 ) + b ( x 3 + 5 x 2 − 1) + c (3 x 3 − x 2 )
lim =1
x→∞ a (5 x 4 − x ) + b (−x 4 ) + c (4 x 4 + 1) + 2 x 2 + 5 x

Solution
We can write:
(2a + b − 3c) x 3 + (−a + 5b − c ) x 2 − b
lim =1 (1)
x→∞ (5a − b + 4 c ) x 4 + 2 x 2 + (−a + 5) x + c

If 5a − b + 4c ≠ 0 then the limit (1) is equal to 0 ≠ 1 .


Then 5a − b + 4c = 0 . It results that 2a + b − 3c = 0 , because if not, the limit (1) would
be equal to ±α ≠ 1 . Then:
(−a + 5b − c ) x 2 − b
lim = 1,
x→∞ 2 x 2 + (−a + 5) x + c

−a + 5b − c
from where = 1.
2
and the real numbers a , b, c verify the system:

64
⎧⎪5a − b + 4c
⎪⎪
⎨2a + b − 3c = 0
⎪⎪
⎪⎪⎩−a + 5b − c = 2
When resolved we fund the values:
2 46 14
a= , b= , c= .
109 109 109

6.59.
Given the natural numbers ai , b j between 0 and 9, with ai ≠ 9 ≠ bm and a1 + 1 = x1 ,
9 − ai = yi , i = 2, n , and 9 − b j = z j , j = 1, m . Compute:
⎡ ⎛ n p m ⎞⎤
lim ⎢0, x1 − ⎜⎜∑ 0,0...0 yi + ∑∑ 0, 0........0 z j ⎟⎟⎟⎥⎥
⎢ ⎜
p→∞
⎣⎢
⎜⎝ i=2 i k =0 j =1 n−1+ j+k −m ⎠⎟⎦⎥

Solution
We note
n
α = ∑ 0,0...0 yi = 0,0 y2 ... yn
i =2 i
and
p m
β ( p ) = ∑∑ 0, 0........0 z j = 0,0....0 z1.... zm .... z1.... zm
......
k =0 j =1 n−1+ j+k −m n 1 p

If we take t = bm + 1 , we have:
γ p = 0, x1 − (α + β ( p )) = 0, a1...an b1...bm ... b1...bm b1...bm t (1)
1 p−1 p

We show that γ p → 0, a1...an (b1...bm ) = γ 0 .


p→∞

Let ε > 0 , ∃p0 = p (ε) ∈ N , p0 being the smallest natural number which has the property :
lg ε ⋅10n
p0 > − ,
m
such that
n
∀p ≥ p0 , p ∈ N , γ p − γ 0 = 10−n− pm < 10−n ⋅10lg ε⋅10 = ε ,
and it results (1).

6.60.
We consider the functions f1 , f 2 : R → R such that lim f1 ( x ) = ∞ and lim f 2 ( x ) = a .
p→∞ x→∞

65
⎛ ⎡ 1 ⎤ ⎞⎟
Show that lim ⎜⎜⎜ f1 ( x ) ⋅ ⎢ ⎥⎟ exists and compute this limit , where α represents the
x→∞ ⎜
⎝ ⎢ f 2 ( x ) ⎥ ⎟⎠⎟
⎣ ⎦
integer part of α .

Solution
Then
lim f1 ( x ) = +∞ or −∞ (1)
x→∞
⎡ 1 ⎤
We note f ( x ) = f1 ( x ) ⎢ ⎥;
⎢ f2 ( x) ⎥
⎣ ⎦
Discussion:
A) If a = 0 , then
1 ⎡ 1 ⎤
lim = +∞ or −∞ or it doesn’t exist and the same lim ⎢ ⎥ = +∞ or
x→∞ f ( x ) x→∞ ⎢ f ( x ) ⎥
2 ⎣ 2 ⎦
−∞ or it doesn’t exist.. From here lim f ( x ) = +∞ or −∞ (conform to (1) and A) or we
x→∞
cannot conclude anything.
⎡1⎤
B) a ≤ 1 and a ≠ 0 then ⎢ ⎥ ≠ 0 . From where, also, lim f ( x ) = +∞ or −∞ (conform
⎢⎣ a ⎥⎦ x→∞

⎡1⎤
(1) and B), that is conform to (1) and the sign of ⎢ ⎥ .
⎢⎣ a ⎥⎦
⎡1⎤
C) a > 1 ⇒ ⎢ ⎥ = 0 . We have f 2 ( x ) → a > 1 . Then it exists α > 0 such that a = 1 + α .
⎢⎣ a ⎥⎦ x→∞

Let Vα = (a − α, a + α ) a neighborhood of a ; 1 ∉ Vα . Let a sequence xn → ∞ , then f 2 ( xn ) → a


(conform to the limit’s definition.). f 2 ( xn ) is a real sequence which tends to a . It results that
outside of Vα we find at most a limited number of terms of the sequence f 2 ( xn ) if and only if it
exists at most a limited number of terms that have the property f 2 ( xi ) ≤ 1 . Therefore, the
majority if terms are found in Vα , therefore it exists nα ∈ N * such that ∀i > nα , f 2 ( xi ) > 1 .
⎛ ⎡ 1 ⎤ ⎞⎟
Therefore the sequence ⎜⎜⎜ ⎢ ⎥⎟ is the following:
⎜⎝ ⎢⎣ f 2 ( xn ) ⎥⎦ ⎟⎠⎟
n∈ N *

⎡ 1 ⎤ ⎡ ⎤
⎢ ⎥ ,..., ⎢ 1 ⎥ ,0,0,0,..
⎢ f 2 ( x1 ) ⎥ ⎢ f (x )⎥
⎣ ⎦ ⎣⎢ 2 nα ⎦⎥
Therefore , with the exception of a limited number of null terms, this sequence is the null
sequence.
It results that the sequence ( f 2 ( xn ))n∈ N * is the following:
⎡ 1 ⎤ ⎡ ⎤
f1 ( x1 ) ⋅ ⎢ ⎥ ,..., f1 ( xn ) ⋅ ⎢ 1 ⎥ ,0,0,0,...
⎢ f 2 ( x1 ) ⎥ α ⎢ f (x )⎥
⎣ ⎦ ⎢⎣ 2 nα ⎥⎦

66
Then, also, with the exception of a limited number of null terms, this new sequence is the
constant, null sequence.
In conclusion: lim f ( x ) = 0
x→∞

6.61.
Compute without using l’Hospital theorem)
n ri
f i ( x ) − ri f i ( x0 )
lim ∏ s
x→ x0
i =1
i
f i ( x ) − si f i ( x0 )
with the conditions that the anterior solutions exist. Generalization.

Solution
We multiply each fraction by the conjugate of the numerator and denominator.
Let 1 ≤ i ≤ n , we note:
A = ri f i ( x )
B = ri f i ( x0 )
C = si f i ( x )
D = si f i ( x0 )
We have:
A − B ( A − B )( A + A B + ... + A B + B )(C + A D + ... + C D + D )
ri −1 ri −2 1 1 ri −2 ri −1 si −1 si −2 1 1 si −2 si −1

= =
C − D (C − D )( Ari −1 + Ari −2 B1 + ... + A1B ri −2 + B ri −1 )(C si −1 + C si −2 D1 + ... + C 1C si −2 + D si −1 )

s ( )
si −1

=
ri
A −B ri

(C si −1 + C si −2 D1 + ... + D si −1 )
→ ⋅ i
si
f i ( x0 )
=
si si −ri
⋅( f i ( x0 )) si ri
C si − D si ( Ari −1 + Ari −2 B1 + ... + B ri −1 ) r ( x→ x0
i
ri
f (x ))
ri −1
ri
i 0

From where the limit from the hypothesis will be equal to:
n
si si −ri


i =1 i r
⋅ ( f i ( x 0 ) ) si ri

Generalization:
⎧⎪ n si si −ri
n ri
f i ( x ) − f i ( x0 ) ⎪∏
ri ⎪ ⋅ ( f ( x ) ) si ri , if n = m

∏ ⎪⎪ i=0 ri i 0

i =1
si
f ( x ) − si f i ( x0 ) ⎪⎪
lim m r i = ⎨0 if n > m
x→ x0 i
f i ( x ) − ri f i ( x0 ) ⎪⎪

j =1
sj
f i ( x ) − s j f i ( x0 ) ⎪⎪⎪
⎪does not exist if n < m
⎪⎩
in the conditions in which the anterior solutions exist.
If n < m the limit does not exist because the denominator is equal to zero, then the numerator is
not null; from where the two lateral limits are different.

67
Remark:
If there exist at least a constant function fi0 , 1 ≤ i0 ≤ min {m, n} , then the limit does not
exist.
The same thing for the limit from the problem’s enounce.

68
ALGEBRA

69
7.62.
Compare the sets:
⎧⎪⎪ n ⎫⎪
A = ⎨ X | X = ∑ ai Ki + a, with ( K1 ,.., K n ) ∈ Z n , and a1 ,..., an are integer constants such that (a1 ,..., an ) = 1⎪⎬
⎪⎩⎪ i =1 ⎪⎭⎪

⎪ m ⎫⎪
⎪X | X = ⎪
B =⎨
⎪ ∑ b j K j + b, with ( K1 ,.., K m ) ∈ Z m , and b1 ,..., bm ,b are integer constants⎬


⎩ j =1 ⎭⎪

Solution
First of all we prove that A = Z .
It is obvious that A ⊆ Z , and also Z ⊆ A because
n
∀Z ∈ Z , ∃( K10 ,..., Kn0 ) ∈ Z n : ∑ ai Ki0 + a = Z
1

Because the greatest common divisor (a1 ,..., an ) = 1 and 1 divides Z − a .


We see immediately that B ⊆ Z .
If (b1 ,..., bm ) =1 then B = Z = A , otherwise B ⊂ A and B ≠ A .

7.63.
Given a prime number p and M a set of p consecutive natural numbers, prove that
M cannot be divided in two disjoint subsets M 1 , M 2 , with M 1 ∪ M 2 = M , as the product of the
numbers in M 1 are equal to the product of the numbers in M 2 .

Solution
Because M contains p consecutive natural numbers, then M constitute a complete
system of residues modulo p . Then:
∃n0 ∈ M : n0 = Mp ∀n ∈ M − {n0 } , n ≠ Mp
We consider that n0 ∈ M 1 (the same proof for the contraire cae).
The product of the numbers in M 1 is divisible by p , but the product of numbers from M 2 is not
divisible by p , because p is a prime number and because it does not exist any element in M 2
which is a multiple of p . From here, the product from M 1 cannot be equal to the product from
M2 .

70
7.64.
Let’s consider M a set which contains m natural numbers, m ≥ 2 , and n ∈ *, n<m.
⎡ m −1⎤
We note: K = ⎢ ⎥ + 1 , where [ X ] represents the integer part of X .
⎢⎣ n ⎥⎦
Prove that there exists a subset M ' of M , such that:
a) M ' contains at least K elements
b) The difference between two random elements of M ' is a multiple of n .

Solution
The set M has the form: M = {a1 ,..., am } , where all ai ∈ . ∀i ∈ {1,..., m} , ai = Mn+ri
with ri ∈ {0,1, 2,..., n − 1} .
Let m = q ⋅ n + r, q ∈ , 0 ≤ r ≤ n −1 . Because n < m , it results that q ∈ * . We
construct the set M ' ⊆ M . Because the condition b) are achieved it must that M ' contains only
elements from M which divided by n will give the same residue.
The residues obtained from the division by n are: 0,1,…,n-1. We have n equivalence classes
modulo n .
The problem is reduced to prove that there exists a class which contains at least K
elements. M ' will be exactly this class.
⎡ m −1⎤ ⎡ q : n −1⎤
1) The case when r = 0 . Then K = ⎢ ⎥ +1 = ⎢ ⎥ +1 = q .
⎣⎢ n ⎦⎥ ⎣⎢ n ⎦⎥
If m = q : n elements of M are equally distributed in the n classes, then each class contains q
elements, and the problem is solved.
In the contrary case, there exists at least a class which contains at least q elements and
also, in this case the problem is resolved.
⎡ m −1⎤ ⎡ qn + r −1⎤
2) The case when r ≠ 0 . Then K = ⎢ ⎥ +1 = ⎢ ⎥ +1 = q +1 .
⎢⎣ n ⎥⎦ ⎢⎣ n ⎥⎦
The m = qn + r elements of M are distributed randomly in the n classes, it results that
there exist at least a class which contains at least q + 1 elements (if not it would result that there
would be a maximum of qn elements < n ), and the problem is completely solved.

7.65.
Let’s consider the homogeneous polynomials Pn ( x, y ) and Qn ( x, y ) of n degree in x, y
a a P (a , b ) P (a , b )
. If 1 = 2 then n 1 1 = n 2 2
b1 b2 Qn (a1 , b1 ) Qn (a2 , b2 )

Solution
Let

71
Pn ( x, y ) = α0 x n + α1 x n−1 y1 + ... + αn−1 xy n−1 + αn y n
and
Qn ( x, y ) = β0 x n + β1 x n−1 y1 + ... + βn−1 xy n−1 + βn y n
a1 a2 a b a n b n a n−i bi α a n−i bi
= ⇒ 1 = 1 ⇒ 1n = 1n = 1n−1 1i = i 1n−1 1i
b1 b2 a2 b2 a2 b2 a2 b2 αi a2 b2
0≤i ≤n .
Since the sum of the numerators divided by the sum of the denominators is equal with
each rapport, we have:
α0a1n + α1a1n−1b11 + ... + αn b1n a1n
=
α0a2n + α1a1n−1b21 + ... + αn b2n a2n
The same way, we obtain:
β0a1n + β1a1n−1b11 + ... + βn b1n a1n
= ,
β0a2n + β1a1n−1b21 + ... + βn b2n a2n
Then:
Pn (a1 , b1 ) Qn (a1 , b1 )
=
Pn ( a2 , b2 ) Qn ( a2 , b2 )
Therefore, the conclusion.

7.66.
Let’s consider a natural number

p ≥ 2 and a sequence such that a1 = 1, an+1 = pan + 1, ∀n ∈ * . Prove that ∀K ∈ *,


K can be uniquely expressed as follows: K = t1an1 + ... + t1an1 with 1 ≤ ti ≤ p −1 for
i ∈ {1, 2,..., l − 1} and 1 ≤ ti ≤ p and n1 > ... > n1 .

Solution
p n −1
We deduct immediately that an = which is a sequence of natural numbers, strictly
p −1
ascending, unlimited. Then: ∃n1 ∈ * such that an1 ≤ k < an1+1 = pan1 + 1 .
From here k can be written uniquely as follows:
k = t1an1 + r1 ≤ pan1 , with 0 ≤ r < an1 .
k ⎡a p⎤
If r1 = 0 , it results that t1 = and 1 ≤ t1 ≤ ⎢⎢ n1 ⎥⎥ = p
an1 ⎢⎣ an1 ⎥⎦
If r1 ≠ 0 , it results that there exists n2 ∈ * such that an2 ≤ r1 < an2 +1 = pan1 + 1 ⇒ r1 = t2an2 + r2

72
k − r1 pan1 −1
an2 ≤ r1 < an1 ⇒ n2 < n1 ; t1 = ≤ < p;
an1 an1
an1
Then 1 ≤ t1 ≤ p −1 .
And k is written uniquely k = t1an1 + t2an2 + r2 , and the process continue. After a limited
number of steps we reach r1 = 0 . We will have the same prove for ni > ni+1 , i ∈ {1,...,1 − 1} and
1 ≤ ti ≤ p −1 , i ∈ {1,...,1−1} and 1 ≤ t1 ≤ p , and the problem is solved.

7.67.
If a1 ,..., an , b are positive real numbers with b ≤ a1 + ... + an and α ∉ {−a1 ,..., −an , −b} ,
b a1 an
then ≤ + ... + .
α + b α + a1 α + an

Solution
We will use the recurrence rational for n ∈ * .
b a α (b − a1 )
For n = 1 , if b ≤ a1 , we have − 1 ≤0 ⇔ ≤ 0 ; which is true,
α + b α + a1 (α + b)(α + a1 )
and taking into account the hypothesis/
We suppose that the inequality is true for all the values smaller or equal to n .
We will prove it for n + 1 : b ≤ (a1 + ... + an ) + an+1 and in conformity to the hypothesis
b (a1 + ... + an ) an+1
of the recurrence we have: ≤ + , but a1 + ... + an ≤ a1 + ... + an ,
α + b α + (a1 + ... + an ) α + an+1
then if we apply one more time the hypothesis of the recurrence, we obtain:
b a1 a2 an an+1
≤ + + ... + +
α + b α + a1 α + a2 α + an α + an+1

7.68.
Let’s consider the expression: E ( x, y ) = Ax 2 + Bxy + Cy 2 + Dx + Ey + F , ( x , y ) ∈ 2
,
with A, B, C , D, E , F real, and A2 + C 2 ≠ 0 . Find a necessary and sufficient condition for
E ( x, y ) to admit an extreme.

Solution
Because A2 + C 2 ≠ 0 , it results that at least A or C is not null, let it be A ; (the result
will be similar if C ≠ 0 ).

73
We suppose that E ( x, y ) admits an extreme.
⎛ B D ⎞
E ( x, y ) = A⎜⎜ x 2 + xy + x ⎟⎟⎟ + Cy 2 + Ey + F =
⎜⎝ A A ⎠
2
⎛ B D⎞ B 2 2 D 2 2 BD
= A⎜⎜⎜ x + y + ⎟⎟⎟ − y − − y + Cy 2 + Ey + F =
⎝ 2A 2 A⎠ 4 A 4A 4A
4 AC − B 2 2 4 AE − 2 BD 2 ⎜⎛ D 2 ⎞⎟
2
⎛ B D ⎞⎟
= A⎜⎜ x + y+ ⎟ + y + y + ⎜ F − ⎟
⎝⎜ 2A 2 A ⎠⎟ 4A 4A ⎝⎜ 4 A ⎠⎟⎟
4 AC − B 2
≠ 0 , because if not E ( x, y ) would not have extreme.
4A
We have
⎞⎟ 4 AC − B 2 ⎡⎢ 2 ⎛ 2 AE − BD ⎞⎟ ⎤⎥
2 2
⎛ B D 2 AE − BD
E ( x, y ) = A⎜⎜⎜ x + y + ⎟⎟ + ⋅⎢y +2 y + ⎜⎜⎜ ⎟ −
⎝ 2A 2 A⎠ 4A ⎢⎣ 4 AC − B 2 ⎝ 4 AC − B 2 ⎟⎠ ⎥⎥

4 AC − B ⎛⎜ 2 AE − BD ⎞⎟ ⎜
2 2
⎛ 2⎞
D ⎟
− ⋅⎜ ⎟ + ⎜⎜ F − ⎟=
4A ⎜ 2 ⎟
⎝ 4 AC − B ⎠ ⎝ 4 A ⎟⎠⎟
2


2 B D ⎟⎞
2
4 AC − B 2 ⎛⎜ 2 AE − BD ⎞⎟
2
D2 (2 AE − BD )
= A ⎜⎜ x + y+ ⎟⎟ + ⎜ y + ⎟ + F − − (1)
⎝ 2A 2 A⎠ 4A ⎝⎜ 4 AC − B ⎠⎟
2
4 A 4 A(4 AC − B 2 )
4 AC − B 2
Evidently E ( x, y ) admits only one extreme if A and have the same sign, that is if
4A
4 AC − B 2
A > 0 , from where 4 AC − B 2 > 0 , which constitute a necessary condition for E ( x, y )
4A
to admit un extreme (as long as the two are positive and have a minimum, if not a maximum).
But 4 AC − B 2 > 0 constitute also a sufficient condition, because E ( x, y ) can be written as (1)
4 AC − B 2
and A and have the same sign.
4A

7.69.
Let’s consider the integers A = a1...a2 n + a2 n ...a1 and B = a1...an − an ...a1 written in base
b∈ * −{1} . Show that A is divisible by b + 1 and B is divisible by b − 1 .

Solution
We’ll compute the criteria of divisibility by b + 1 and by b − 1 in base b .
bi ≡ (−1)i ( mod b + 1) and bi ≡ 1( mod b − 1) , i ∈ .
Because
A ≡ (a2 n − a2 n−1 + ... + (−1)k ak + ... + a2 − a1 ) +

74
+(a1 − a2 + ... + (−1)k ak + ... + a2 n−1 − a2 n ) ≡ 0( mod b + 1) .
Similarly:
B ≡ (an − an−1 + ... + a1 ) − (a1 + a2 + ... + an ) ≡ 0 ( mod b + 1) .

7.70.
Let’s consider a natural number a and p a non- null integer. Determine the number of
⎪⎧ ⎪⎫
elements of the set M = ⎪⎨a , aa,..., a...a ⎪⎬ which are divisible by p . Discussion.
⎪⎩⎪ n ⎪ ⎭⎪

Solution
Let a be written in base 10 in the form: a = a1...as , with ai ∈ , 1 ≤ i ≤ s, s ∈ * .An
element α ∈ M is α = a...a with 1 ≤ n ≤ m , n integer.
n

10 sn − 1
α = a ⋅10( n−1) s + a ⋅10( n−2) s + ... + a ⋅101s + a = a (1 + 10s + ... + 10( n−1) s ) = a .
10s − 1
We must find the n for which α divides p .
Let d = (a , p ) . Then a = da1 and p = dp1 and (a1 , p1 ) = 1 . Then we must determine n
10sn − 1
such that da1 s | dp1 .
10 − 1
1) If ( p,10) = 1 , let’s consider δ the oreder of the class of residues of 10 in rapport to
mod p1 (10s −1) . We have also: 10δ − 1 ≡ 0( mod p1 (10s −1)) , then 10δ − 1 ≡ 0 ( mod (10 s − 1))
⎡ m ⎤ ⎡ ms ⎤
and it results that δ = ks , k ∈ * . Then it exists exactly ⎢ ⎥ = ⎢ ⎥ elements in M which are
⎣⎢ k ⎦⎥ ⎣⎢ δ ⎦⎥
divisible by p .

7.71.
Prove that ∀k ∈ * −{1} there exist an infinity of natural numbers whose property is that
they admit exactly k positive divisors.

Solution
Any natural number A is written in the following form:
A = p1α1 ... psαs , (obviously A ∉ {0,1} ) where pi are prime numbers, i ∈ {1,..., s} and
pi ≠ p j for i ≠ j , αi ∈ * with i ∈ {1,..., s} , s ∈ * (which is the canonic form of a number,
and which is unique).
Re note by d ( A) the number of positive divisors of A .

75
Then
n
d ( A) = ∏(αi + 1) (1)
i=1
n

∏(α + 1) = k
s
Our problem is reduced to prove that the equation i has solutions in ( *)
i =1

(the unknown are α1 ,...αs and s ). We take s = 1 ∈ * and α1 = k − 1 ∈ * and we obtain


k − 1 + 1 = k . From all the numbers n = p k −1 , with a random prime number, has exactly k
positive divisors (we have an infinity of numbers n because we have an infinity of prime
numbers p ).
We can see that equation (1) has other solutions . For example, if k = k1 ,...kt with all
ki ∈ * −{1} , we have the infinite solution, s = t , α1 = k1 −1,..., αt = kt −1 from where
n = p1 ,..., pt and where all p j are the different prime numbers.

7.72.
Knowing that ai , i ∈ {1, 2,..., n} , satisfying the conditions of existence for all n
logarithms, solve the equation:
loga1 loga2 ...logan x = b

Solution
loga1 (loga2 loga3 ...logan x ) = b ⇔ loga2 loga3 ...logan x = a1
Where
log a2 (log a3 ...log an x ) = a1b ⇔ log a3 log a4 ...log an x = a2a1
b

Where
log a3 (log a4 ...log an x ) = a2a1 ⇔ log a4 log a5 ...log an x = a3a2
b a1 b

……………………………………………………………………….
a1b
⇔ log an−1 log an x = an. −2
a1b
⇔ log an x = an. −1
a1b
⇔ x = an.
Which is the solution of the problem.

7.73.
If a, b ∈ *
+ , b ≠ 1 and ∀α ∈ , a ≠ bα , then log b a ∉ , and reciprocal.

76
Solution
First of all we observe that this problem is the result of several particular problems; for
example “show that log 2110 is not a rational number”, etc.
The proof will be done using the absurd rational.
m
We suppose that logb a ∈ , with least common divisor (m, n ) = 1 .
n
m
It results that b n = a . By raining this equality to the n th power, we have bm = a n , with m, n ∈
, a n ≠ bnα , where nα ∈ (because a > 0 , b > 0 ),
Then, if we cannot write it as a rational power of b ,we would not be able to write a n any
more as a rational power of b .
And, it results that a n ≠ bm , ∀m ∈ . Contradiction.
Therefore log a ∈ .
Reciprocal:
If log b a ∉ , then obviously a ≠ bα , ∀α ∈ , because if not it will result that
logb bαα = α ∈ .
And the problem is completely proved.

7.74.
Let s ≠ 0 a natural number. Determine the natural numbers n that verify the propriety
s
n divides n . (we note [ x ] the integer part of x ).

Solution
s
∀n ∈ ∃p ∈ : p s ≤ n < ( p + 1) = p s + Cs1 p s−1 + ... + Css−1 p1 + 1 .
From here n can be written: n = p s + k with 0 ≤ k < Cs1 p s−1 + ... + Css−1 p1 + 1 and
k∈ . It results ⎡⎢ s n ⎤⎥ = p . Because ⎡⎢ s n ⎤⎥ divides n , we obtain that p divides k . Then
⎣ ⎦ ⎣ ⎦
, and 0 ≤ k ≤ ( p + 1) − ( p s + 1) from where:
s
k = α : p , with α ∈
( p + 1) − ( p s + 1)
s

0≤α≤ ∈
p
Therefore the natural numbers that have the property from the hypothesis are:
( p + 1) − ( p s + 1)
s
s
n = p + α p , with the property α ∈ and 0 ≤ α ≤ , p ∈ * , and also the
p
trivial solution n = 0 , because 0 divides 0.

77
7.75.
Let p a natural number, p ≥ 2 and the function:
⎡x⎤ ⎡ x ⎤
β p ( x ) = ⎢ ⎥ + ⎢ 2 ⎥ + ... , x ∈ + .
⎢⎣ p ⎥⎦ ⎢⎣ p ⎥⎦
Show that if x = (an ...a1a0 ) then:
p

1 ⎡
β p ( x) = x − (a0 + a1 + ... + an )⎤⎦
p −1 ⎣

Solution
x = (an ...a1a0 ) = an p n + ... + a1 p1 + a0
p

⎡x⎤ ⎡a ⎤
⎢ ⎥ = an p n−1 + an−1 p n−2 + ... + a1 p1 + a1 because ⎢ 0 ⎥ = 0
⎢⎣ p ⎥⎦ ⎣⎢ p ⎥⎦
⎡ x⎤
⎢ 2 ⎥ = an p n−2 + an−1 p n−3 + ... + a2 because 0 ≤ a1 p1 + a0 < p 2
⎢⎣ p ⎥⎦
…………………………………………………………………………..
⎡ x⎤
⎢ n ⎥ = an
⎢⎣ p ⎥⎦
⎡ x ⎤
⎢ m ⎥ = 0 for all m natural and m ≥ n + 1 .
⎢⎣ p ⎥⎦
Therefore
β p ( x ) = an (1 + ... + p n−1 ) + an−1 (1 + ... + p n−2 ) + ... + a2 (1 + p) + a1 =
= an (1 + ... + p n−1 ) + an−1 (1 + ... + p n−2 ) + ... + a2 (1 + p) + a1 =
p n −1 p −1 p −1
= an + ... + a2 + a1 =
p −1 p −1 p −1
1 1 ⎡
= (an p n + ... + a1 p1 − an − ... − a1 ) = x − (a 0 +a 1+... + an )⎤⎦ .
p −1 p −1 ⎣

7.76.
Prove the inequality:
⎛ 2 2⎞ ⎛ 2 2⎞ ⎛ 2 2⎞
⎜⎜ a1 + a2 ⎟⎟ + ⎜⎜ a2 + a3 ⎟⎟ + ... + ⎜⎜ an + a1 ⎟⎟ ≥
⎜⎝ a 2 a 2 ⎟⎟⎠ ⎜⎝ a 2 a 2 ⎠⎟⎟ 2⎟
⎝⎜ a1 an ⎟⎠
2
2 1 3 2
2 2 2
⎛a a ⎞ ⎛a a ⎞ ⎛a a ⎞
≥ ⎜⎜ 1 + 2 ⎟⎟⎟ + ⎜⎜ 2 + 3 ⎟⎟⎟ + ... + ⎜⎜ n + 1 ⎟⎟⎟
⎜⎝ a
2 a1 ⎟⎠ ⎜⎝ a3 a2 ⎟⎠ ⎜⎝ a1 an ⎟⎠
where

78
a1 ,..., an ∈ \ {0} , n ≥ 2

Solution
⎛ a 2 a 2 ⎞ ⎛ a b ⎞2
It is sufficient to prove that: ⎜⎜ 12 + 22 ⎟⎟⎟ ≥ ⎜⎜ + ⎟⎟⎟ , where a , b ∈ \ {0} .
⎜⎝ a2 a1 ⎟⎠ ⎝⎜ b a ⎠
After computing the powers, passing all the terms to the right side and executing all the
reductions of all similar terms it results that:
a 4 b4 a 2 b2
+ − − ≥0
b4 a 4 b2 a 2
a2
We note 2 = u . We have
b
1 1
u2 + 2 − u − ≥ 0 .
u u
2
⎛ ⎞ ⎛ ⎞ 2
⎜⎜u + 1 ⎟⎟ − ⎜⎜u + 1 ⎟⎟ − 2 ≥ 0 , u = a 2 > 0
⎝⎜ u ⎟⎠ ⎜⎝ u ⎟⎠ b
1
We note t = u + ≥ 2 ; we have t 2 − t − 2 ≥ 0 , that is (t + 1)(t − 2) ≥ 0 , inequality which
u
is true for t ≥ 2 .
Therefore each parenthesis from the right side, squared, is greater or equal to the
correspondent parenthesis in the left side squared.

7.77.
Show that if a1 + a2 + ... + an = a1...an , (ai ≠ 0, i ∈ {1, 2,..., n}) , then
⎛1 1 1⎞ ⎛1 1 1 1⎞ ⎛1 1 1 ⎞⎟
a1 ⎜⎜ + + ... + ⎟⎟⎟ + a2 ⎜⎜ + + ... + + ⎟⎟⎟ + ... + an ⎜⎜ + + ... + ⎟+ n =
⎜⎝ a2 a3 an ⎟⎠ ⎜⎝ a3 an an a1 ⎟⎠ ⎝⎜ a1 a2 an−1 ⎠⎟⎟
n
= ∑ a1...ai−1ai+1...an .
i =1

Solution
The left side of the equality can be written:
⎛a ⎞ ⎛ ⎞ ⎛ ⎞
⎜⎜ 1 + a1 + ... + a1 ⎟⎟ + ⎜⎜ a2 + a2 + ... + a2 + a2 ⎟⎟ + ... + ⎜⎜ an + an + ... + an ⎟⎟ + n =
⎜⎝ a2 a3 ⎟
an ⎟⎠ ⎜⎝ a3 an an a1 ⎟⎠ ⎟ ⎜⎝ a1 a2 an−1 ⎠⎟⎟
⎛a a a ⎞ ⎛a a a ⎞ ⎛a a a ⎞
= ⎜⎜ 2 + 3 + ... + n ⎟⎟⎟ + ⎜⎜ 1 + 3 + ... + n ⎟⎟⎟ + ... + ⎜⎜ 1 + 2 + ... + n−1 ⎟⎟⎟ + n =
⎜⎝ a a a ⎟⎠ ⎜⎝ a a a ⎟⎠ ⎜⎝ a a a ⎟⎠
1 1 1 2 2 2 n n n

a ...a − a1 a1...an − a2 a ...a − an


= 1 n + + ... + 1 n +n =
a1 a2 an

79
n n
= ∑ a1...ai−1ai+1...an − n + n =∑ a1...ai−1ai+1...an .
i=1 i =1

7.78.
Let the integer numbers a, b, ai , b j with i ∈ {1, 2,..., n} and j ∈ {1, 2,..., m} . Show that if
xi , y j (with i ∈ {1, 2,..., n} and j ∈ {1, 2,..., m} ) are integer numbers, the expression:
n

∑a x +ai i
E ( x1 ,..., xn , y1 ,..., ym ) = i =1
m

∑b y
j =1
j j +b

has integer values if and only if the greatest common divisor of the numbers a1 ,..., an , b1 ,..., bn , b
divides a .

Solution
The expression E ( x1 ,..., xn , y1 ,..., ym ) will have integer values if and only if the equality
E ( x1 ,..., xn , y1 ,..., ym ) = t admits integer solutions, where the unknown are x1 ,..., xn , y1 ,..., ym , t .
This equation is equivalent to:
n m

∑ a x −∑ b y t − bt = −a
i =1
i i
j =1
j j (1)
n m
⇔ ∑ ai xi −∑ b j t j − bt = −a
i =1 j =1

with
t j = y j , t, j ∈ {1,..., m} . (2)
Necessity
If this equation admits solutions in , then it results that the greatest common divisor of
the numbers a1 ,..., an , b1 ,..., bm ,b divide a .
Sufficiency
1) The case b ≠ 0 . Because the greatest common divisor of a1 ,..., an , b1 ,..., bm ,b divide b ,
it results that the equation (2) admits solutions in , then it exist t ∈ such that
E ( x1 ,..., xn , y1 ,..., ym ) = t .
2) The case b = 0 . We note d1 = (a1 ,..., an ) and d 2 = (b1 ,..., bm ) . From which
D = (a1 ,..., an , b1 ,..., bm , b) = (d1 , d 2 ) divides a . The equation (1) becomes:
n ⎛ m ⎞
⎜ b ' y ⎟⎟ ⋅ t = −a

i =1
a x
i i − d 2⎜ ∑
⎜⎜⎝ j=1 j j ⎟⎟

(3)

where b j = d 2b ' j , 1 ≤ j ≤ m .
Because (b '1 ,..., b ' m ) = 1 , it results that there exist ( y10 ,..., ym0 ) ∈ m
such that

80
m

∑b'
j =1
j y 0j = 1 . From here we have:
n

∑ a x − d t = −a
i =1
i i 2 (4)

Because D = (d1 , d 2 ) = (a1 ,..., an , d 2 ) divides a , it results that exists t in such that
E ( x1 ,..., xn , y1 ,..., ym ) = t .

7.79.
Let ai ∈ , i ∈ {1, 2,..., n} and the function f : → defined by
k
f ( x) = ∑ ∏( x − ax ) , k natural odd number.
1≤i1≤..≤ik ≤n j =1
j

Show that whatever are the values a1 ,..., an the function does not have the same sign on
the whole real axis.

Solution
Let m = min {ai } and M = max {ai } . We have m ≤ M (the sign + does not bother us).
i i
k
For x ∈ (−∞, m ) we have x − ai < 0, x − ai < 0, ∀i ∈ {1, 2,..., n} and ∏( x − a ) < 0
j =1
ij

because k is odd and all factors of the product are negative.


Then f ( x ) < 0, ∀x ∈ (−∞, m )
The same: f ( x ) > 0, x ∈ ( M , ∞) .
And the signs are different and ∀a1 ,..., an in the function is both negative and positive.

7.80.
If α1 ,..., αn are natural numbers not null, show that
n n
1+ n∑ ∑ αi1 ...αik = ∏(αh + 1)
k =1 1≤i1≤..≤ik ≤n h=1

Solution
Let’s consider the natural number a = p1α1 ... pnαn , where p1 ,..., pn are distinct numbers
two by two.

81
n
We’ll determine the number of positive divisors D . We know that: D = ∏(αh + 1) .
h=1
Then, we apply another method to compute D : we write all the divisors ( k represents the
number of different prime factors, of divisor d ):
k = 1 . We have α1 + ... + αn are the divisors of p11 ,..., pnα1 ,..., pn1 ,..., pn1 ,..., pnn ;
k = 2 . We have ∑
1≤i1≤i2 ≤n
αi1 αi2 divisors:

⎧⎪ p1 p12 ,..., p1 p2α2


⎪⎪
⎪⎨................... (are α1α2 divisors)
⎪⎪
⎪⎪⎩ p1α1 p12 ,..., p1α1 p2α2
…………………………………………………..

⎪ pn−1 pn1 ,..., pn−1 pnαn



⎨................... (are αn−1αn divisors)


⎪ αn−1 1 αn−1 αn
⎩ pn−1 pn ,..., pn−1 pn

………………………………………………….
k = l . We follow the same process, we have ∑ αi1 αi2 ...αil divisors.
1≤i1≤..≤il ≤n
n
Because k can take the l values until n , it results that D = 1 + ∑ ∑ αi1 αi2 ...αik ( we
k =1 1≤i1≤..≤ik ≤n

added 1 because, if k = 0 we have the divisor 1 for a ).

7.81.
Determine the natural number of n digits such that the following expression
x1... xn
m−1
is maximum, knowing that i0 = 0, i1 ,..., im−2 , im−1 = n are fixed, and
∑ xih +1 xih +2 ... xih+1
h =0

that all the numbers are written in base b .

Solution
xih +1 xih +2 ... xih+1 = ah+1 , 0 ≤ h ≤ m − 1 . In this case the rapport becomes:
a1bn−i1 + a2bn−i1−i2 + ... + am
R=
a1 + a2 + ... + am
m m−1

j ∑ a jc j ∑ a (c j j − 1)
We note c j = b − ∑ ih , 0 ≤ j ≤ m . Then R =
n 1
m
= 1+ 1
m
h=1
∑aj 1
∑a
1
j

82
Which is maximum when am = 0 (because c m = 1 ).
m−1 m−1

∑ a j (c j −1) ∑ a (c j j − 1 − cm−1 + 1)
Then Rmax = 1 + 1
m−1
= 1 + (c m−1−1) + 1
m−1
=
∑a
1
j ∑a
1
j

m−2

∑ a (c j j − cm−1 )
= cm−1 + 1
m−1
,
∑aj 1
m−2

∑ a (c j j − cm−1 )
which is maximum when am−1 = 0 . Then Rmax = cm−1 + 1
m−1
and the process
∑a 1
j

continues.
After a limited number of steps, we have:
a (c − c ) + a2 (c2 − c3 ) a (c − c )
Rmax = c3 + 1 1 3 = c3 + (c2 − c3 ) + 1 1 2
a1 + a2 a1 + a2
a1 (c1 − c2 )
Which is maxim when a2 = 0 . Therefore, Rmax = c2 + = c1 = bn−i1 .
a1
And the numbers we
Re a looking for are: x1 x2 ... xi1 0...0 written in base b .
n−i1

7.82.
We consider all residue modulo m classes C1 , C2 ,..., Ck , prime with m , and
ai ∈ Ci , 1 ≤ i ≤ k . Prove that m | ∑
1≤i1<..<is ≤k
ai1 ...ais

Solution
It is obvious k = ϕ(m) , where ϕ is the Euler’s indicator
1) m = 0 ⇒ ϕ(m) = 2 ⇒ s = 1
C1 = {−1} ; C2 = {+1} and then a1 = −1, a2 = +1 , the sum S = −1 + 1 and 0
divides 0.
2) m = ±1 ⇒ ϕ(m) = 1 and ±1| s , for any S ∈ .
3) m ≥ 2 . We note S j = ∑ ai1 ...ai j , 1 ≤ j ≤ k
1≤i1<..<i j ≤k
k k
(A) We construct f ( x ) = ∏( x − ai ) = ∑ (−1)
k− j
x k − j s j taking s0 = 1 .
i =1 j =0

83
Propriety 1: If (a, m) = 1 , then (m − a , m ) = 1 .
Property 2: If m ≠ ±1 , then ϕ ( m ) = M 2 (Their proof is banal)
Then k = 2k1 , k1 ∈ , and the set {a1 ,,..., ak1 ak1+1 ,..., a2 k1 } ≡ {a1 ,,..., ak1 -ak1 ,..., −a1 } and
k1
(B) f ( x ) = ∏(( x − ai )( x + ai ))(mod m )
i =1
If we compare (A) and (B), we observe that for s odd, 0 < s ≤ k , we have
k −s
(−1) Ss ≡ 0(mod m) , which is equivalent to m | Ss .
(We used the property 3: If a ∈ Ci0 , then −a ∈ C j0 , i0 ≠ j0 . Then the set {a1 ,,..., ak1 -ak1 ,..., −a1 }
contains exactly an element of each of the 2k1 classes of prime residues of m modulo m .

7.83.
1 n n −1 1 ⎡ n ⎤
Let ϕ a permutation on the set {1, 2,..., n} . Then ⋅ ∑ ϕ( h ) − h ≤ + ⎢ ⎥.
n h=1 2 n ⎣⎢ 2 ⎦⎥

Solution
⎛1 2 ... n − 1 n ⎞⎟
For the permutation ω = ⎜⎜ ⎟ we have:
⎝⎜n n − 1 ... 2 1 ⎠⎟⎟
n

∑ ω(h) − h = 2 ⎡⎣(n −1) + (n − 3) + (n − 5) + ...⎤⎦ =


h=1
⎡n⎤
⎢ ⎥
⎢⎣ 2 ⎥⎦
⎡ n ⎤⎛ ⎡ n ⎤ ⎞ n (n − 1) ⎡ n ⎤
= 2∑ (n − 2k + 1).... = 2 ⎢ ⎥ ⎜⎜n − ⎢ ⎥ ⎟⎟⎟ = +⎢ ⎥.
k =1 ⎣⎢ 2 ⎦⎥ ⎜⎝ ⎣⎢ 2 ⎦⎥ ⎠ 2 ⎣⎢ 2 ⎦⎥
We prove now using the recurrence method for n ∈ , n ≥ 2 , that the sum:
n ⎛
⎡ n ⎤⎞
S = ∑ ⎜⎜n − ⎢ ⎥ ⎟⎟⎟ gets the maximum value when ϕ = ω .

h=1 ⎝ ⎢⎣ 2 ⎥⎦ ⎠
For n = 2 and 3 we can verify it very easy .
We suppose that the property is true for the values less than n + 2 . We’ll prove that it is
true for n + 2 :
⎛1 2 ... n + 1 n + 2⎞⎟
ω = ⎜⎜ ⎟.
⎜⎝n + 2 n + 1 ... 2 1 ⎟⎟⎠
⎛ 2 ... n + 1⎞⎟
Knowing that ω ' = ⎜⎜ ⎟ is a permutation of n elements and for which S will have
⎜⎝n + 1 ... 2 ⎟⎟⎠
⎛ 1 ... n ⎞⎟
the same value as for the permutation ω " = ⎜⎜ ⎟ , in other words the maximum ( ω "
⎜⎝ n ... 1 ⎟⎟⎠

84
was obtained from ω ' by reducing each element by one unit) in conformity to the hypothesis of
the recurrence.
⎛ 1 ... n + 2⎞⎟
The permutation of two elements η = ⎜⎜ ⎟ gives the maximum value for S (in
⎜⎝n + 2 ... 1 ⎟⎠⎟
conformity with the hypothesis of the recurrence). But ω is obtained starting with ω ' and η :

⎪ω '( h ), if h ∉ {1, n + 2}
ω(h) = ⎪


⎩η ( h ), otherwise

7.84.
Let p an integer number p ≥ 2 and ai( k ) ∈ , where i ∈ {1, 2,..., n} , k ∈ {1, 2,..., m} .
1 1
⎛ n ⎛ m ⎞ ⎞p
2 m ⎛ n
2 ⎞p
⎜⎜ ⎜ (k ) ⎟ ⎟
Then ⎜∑ ⎜∑ ai ⎟ ⎟⎟ ≤ ∑ ⎜⎜∑ (ai( k ) ) ⎟⎟ .

⎜⎝ i=1 ⎜⎝ k =1 ⎟⎠ ⎟ ⎜
k =1 ⎝ i =1
⎟⎠

Solution
First of all we prove that
1 1 1
⎛ n (1) ⎞ p ⎛ n (1) 2 ⎞ p ⎛ n (2) 2 ⎞ p
⎜⎜⎜∑ (ai + ai ) ⎟⎟⎟ ≤ ⎜⎜⎜∑ (ai ) ⎟⎟⎟ + ⎜⎜⎜∑ (ai ) ⎟⎟⎟
(2) 2
(1)
⎝ i=1 ⎠ ⎝ i=1 ⎠ ⎝ i=1 ⎠
We can compute the power of p of this inequality because both sides are positive.
We have:
n n n n n n−1

∑(ai(1) ) + ∑(ai(2) ) + 2∑ ai(1)ai(2) ≤ ∑(ai(1) ) + ∑(ai(2) ) + ∑ ckpα p=k β p=k


2 2 2 2

i =1 i =1 i=1 i =1 i=1 k =1
1 1
⎛ n ⎞ ⎛ n p p ⎞
where α = ⎜⎜∑ (a ) ⎟⎟⎟ and β = ⎜⎜∑ (a ) ⎟⎟⎟⎠
(1) 2 (2) 2
.
⎜⎝ i=1 ⎠ ⎜⎝ i=1
i i

(A) If p = 2k , then
k
⎛ n 1 ⎞k ⎛ 2
⎞⎟
⎜ ⎛ ⎞
1
⎛ ⎞ ⎟ ⎜
⎜ ⎛ ⎞
p
⎟⎟
n
⎟ n
C pk α k β k ≥ 2 (αβ ) = 2 ⎜⎜⎜⎜⎜ ∑ (ai(1) ) ⎟⎟ ⎜⎜ ∑ (ai(2) ) ⎟⎟ ⎟⎟⎟ ≥ 2 ⎜⎜⎜⎜⎜ ∑ ai(1) ai(2) ⎟⎟ ⎟⎟⎟ ≥
k 2 p 2 p

⎜⎜⎜⎝ i=1 ⎟⎠ ⎝⎜ i=1 ⎟⎠ ⎟ ⎜⎜⎝⎜ i=1 ⎟⎠ ⎟


⎝ ⎠⎟ ⎜⎝ ⎟
⎠⎟
n
≥ 2∑ ai(1)ai(2)
i=1
We used the inequality Cauchy-Buniakouski-Schwarts.
(B) Let p = 2k + 1 .
a) α ≤ β . It results:
1 1 1 1 1
k+ k+ k+
k +1 k +1 k +1 k +1
= C pk +1 (αβ )
k k k 2 2
C p αβ =C p α β β β ≥C p β 2
α 2 2 ≥

85
1
k+
⎛ 2
⎞⎟ 2
⎜⎜⎛ n ⎞
p
⎟⎟ n
≥ 2 ⎜⎜⎜⎜⎜∑ ai(1) ai(2) ⎟⎟⎟ ⎟⎟⎟ ≥ 2∑ ai(1) ai(2)
⎜⎜⎜⎝ i=1 ⎠ ⎟⎟ i =1
⎜⎝ ⎟⎠
b) α > β . It results that
1 1 1 1 n
k+ k+ k+
≥ 2 ∑ ai(1) ai(2) .
k+
C pk α k +1β k = C pk α 2
> C pk α 2
β 2
> 2 (αβ ) 2
i =1
Now, from (1), using the absurd reasoning, we obtain what we’re looking for.
The case m = 2 is verified.
We suppose that the property is true for the values smaller than m .
We prove for m .
1 1 1 1 1
⎛ n ⎛ m ⎞ ⎞ p ⎛⎜⎛ n (1)
2
⎞ ⎞ p ⎛ n (1) 2 ⎟⎞ p ⎛⎜ n ⎛ m ( k ) ⎟⎞ ⎟⎞ p
2 2 m ⎛ n ⎞p
(k ) ⎟ ⎟ (k ) ⎟ ⎟
n
⎜⎜ ⎜ ⎟⎟ ≤ ∑ ⎜⎜∑ (ai( k ) )2 ⎟⎟

⎜ ⎜⎝ ⎜ ∑ a ⎟ ⎟

⎟⎠ ⎟⎟
= ⎜


⎜ ∑ a + ∑ a ⎟ ⎟

⎟⎠ ⎟⎟
≤ ⎜
⎜ ∑ ( a ) ⎟ + ⎜ ∑ ⎜
⎜ ∑
⎟⎠ ⎜⎜ i=1 ⎜⎝ k =2
a ⎟
⎟⎠ ⎟⎟ ⎟⎠
⎝⎜⎜⎝ i=1 ⎜⎝ i=1 ⎜
i i i i i
⎜⎝ i=1 k =1 ⎠ i=1 ⎠ ⎝ ⎠ k =1 ⎝ i =1

7.86.
⎡ n −1⎤ ⎡ n ⎤
Prove the inequality: n ! > 2n−1 ⎢ ⎥!⎢ ⎥!
⎢⎣ 2 ⎥⎦ ⎢⎣ 2 ⎥⎦

Solution
a) n = 2 k
n ! = (2 ⋅ 4 ⋅ 6 ⋅⋅2k ) ⋅ ⎡⎣1⋅ 3 ⋅ 5 ⋅⋅(2k −1)⎤⎦ = 2k ⋅[1⋅ 3 ⋅ 5 ⋅⋅k ]⋅ ⎡⎣1⋅ 3 ⋅ 5 ⋅⋅(2k −1)⎤⎦ =
= 2 ⋅ k ! ⎡⎣1⋅ 3 ⋅ 5 ⋅⋅(2k −1)⎤⎦ > 2k ⋅ k !2k−1 ⋅(k −1)! = 22 k−1 ⋅(k −1)!⋅ k !
b) n = 2k + 1
n ! = 2 ⋅ 4 ⋅ 6 ⋅⋅2 k ⋅1 ⋅ 3 ⋅ 5 ⋅⋅(2 k − 1) ⋅ (2 k + 1) = 2 k ⋅ k !⋅1 ⋅ 3 ⋅ 5 ⋅⋅(2 k + 1) >
> 2k ⋅ k !2 ⋅ 4 ⋅⋅2k = 2k ⋅ k !⋅ 2k ⋅ k ! = 22 k ⋅ k ! k !
⎡ n −1⎤ ⎡ n ⎤
From these two results we conclude that: n ! > 2n−1 ⋅ ⎢ ⎥!⎢ ⎥!
⎢⎣ 2 ⎥⎦ ⎢⎣ 2 ⎥⎦

7.87.
Prove the inequality:
m 2i −1 ⎛
⎡ n − j ⎤ ⎞⎟ 2 −1 ⎡ n − h ⎤
m+1

∑∑ ⎜
⎜⎜ ⎢ i ⎥ −1⎟⎟ ⋅ ∏ ⎢ m+1 ⎥ ! n ! > 2
i=0 j=0 ⎝ ⎢⎣ 2 ⎥⎦ ⎠ h=0 ⎢⎣ 2 ⎥⎦

Solution
86
⎡ n ⎤ ⎡ n −1⎤
(1) n !! > 2n−1 ⋅ ⎢ ⎥ !⋅ ⎢ ⎥!
⎢⎣ 2 ⎥⎦ ⎢⎣ 2 ⎥⎦
(See the anterior problems)
We can easily prove that
⎡⎡n−a⎤⎤
⎢⎢ p ⎥⎥
⎢ ⎢⎣ 2 ⎥⎦ ⎥ ⎡ n − a ⎤
(2) ⎢ ⎥=⎢ ⎥
⎢ 2 ⎥ ⎢⎣ 2 p+1 ⎥⎦
⎢ ⎥
⎢⎣ ⎥⎦
with 0 ≤ a < 2 p
⎡⎡n−a ⎤ ⎤
⎢ ⎢ p ⎥ −1⎥
⎢ ⎢⎣ 2 ⎥⎦ ⎥ ⎡ n − a − 2 p ⎤
(3) ⎢ ⎥=⎢ ⎥
⎢ 2 ⎥ ⎢⎣ 2 p+1 ⎥⎦
⎢ ⎥
⎢⎣ ⎥⎦
with 0 ≤ a < 2 p ,
(For this we put n = 2 p+1 ⋅ k + α and 0 ≤ α < 2 p+1 .
Then we use the recurrence method to prove the inequality from enounce. We consider this
inequality as a mathematical proposition which depends of m , then P(m) . We apply the
recurrence on m .
For m = 0 we obtain the inequality (1) which is true. We suppose that P(m) is true. We
must prove that P(m + 1) is true.
From (1), (2), and (3) it results
⎡ n−h ⎤
⎡n−h⎤ ⎢ m+1 ⎥−1 ⎡ n − h ⎤ ⎡ n − h − 2 m+1 ⎤
(4) if 0 ≤ h < 2 m−1 then ⎢ m+1 ⎥ ! > 2 ⎣⎢ 2 ⎦⎥ ⋅ ⎢ m+2 ⎥ ! ⎢ ⎥!
⎢⎣ 2 ⎥⎦ ⎢⎣ 2 ⎥⎦ ⎢⎣ 2 m +2 ⎥

For each h natural, 0 ≤ h < 2 n−1 we rapport (4) in P(m) , and then we execute all computations.
We’ll find exactly P(m + 1) .
Remark: To generalize this inequality we replace 2 by a random natural p , 2 ≤ p < n , and we
follow a similar method. We’ll find that the writing is more complicated and the inequality will
be less fine.

7.88.
Having aij ∈ [0,1] , pij > 0, 1 ≤ i ≤ n, 1 ≤ j ≤ m; n, m ∈ * . Prove that
n m n ⎛ m m ⎞
⎜ a pij − pij ⎟
∏ ∑ ∏ ∑ ∑
n
ij ⎟ ≤ m + ( 2 k − m )
pij
a + ⎜ a ⎟ n

i =1 j =1
ij ⎜
i =1 ⎜
⎝ j=1
ij
j =k +1
⎟⎠
Where k is a natural number

Solution:

87
We note
n m n
P1 = ∏ ∑ aij ij = ∏(aip1i 1 + aip2i 2 + ... + aimpim ) = (a11p11 + a12p12 + ... + a1pm1m )...(anp1n1 + anp2n 2 + ... + anm )
p pnm

i =1 j =1 i =1

If all the multiplications are done, one can see P1 is an algebraic sum which contains mn
terms and each of them has the sign +.
It is noted
n ⎛ k m ⎞ n
P2 = ∏⎜⎜⎜ ∑ aij ij − ∑ aij ij ⎟⎟⎟ = ∏(aip1i 1 + ... + aikpik − aikpik++11 − ... − aimpim ) =
p p

i =1 ⎜
⎝ j=1 j =k +1
⎟⎠ i=1
= (a11p11 + ... + a1pk1k − a1pk1+k +11 − ... − a1pm1m )...(anp1n1 + ... + ankpnk − ankpnk++11 − ... − anm
pnm
)
Also, when all the multiplications are done it is obtained an algebraic sum which contains
n
m terms, some have the sign +, others -. The terms of P2 are equal two by two to the terms of
P1 in absolute value. We note P = P1 + P2
In P all the negative terms of p will be reduced, because each has a positive
corresponding. (The null terms of P2 which have the sign – will be reduced with the null terms
of P1 which have the sign + and which have the same absolute value.
Therefore, without affecting the generality, it is considered that the null terms of P1 and
P2 are positive or negative, in function of the sign + or – which are found in front of them.)
Thus, P will be equal to two times the sum of all the positive terms of P2 . A positive
p
term of P2 has the following form: ai1 ij11j1 ....aimimjmjm ∈ [ 0,1] .
p

It results that P P is inferior or equal to two times the number of positive terms of P2 (the
equality will be true when all aij = 1 ).
Let’s consider the sequence
b1 = k , bn+1 = (2k − m ) bn + (m − k ) m n
By the recurrence method it is possible to prove that bn will be calculated exactly the
number of positive terms of P2 .
Because we are only interested in the sign of the terms, we convine to designate by + a
positive term, and by – a negative term.
The case n = 1 implies P21 = +.... + .−... − , therefore the number of the positive terms is
k m−k

k and b1 = k . It is supposed that the property is true for n , and we have to prove it for n + 1 .
For n , we have
⎛ ⎞⎟

P2( ) = ⎜⎜+.... + . −... −⎟⎟⎟
n

⎜⎝ bn m n −bn ⎟⎠
where bn represents the number of positive terms of P2(n ) . For n + 1 we have
⎛ ⎞⎟ ⎛ ⎞

P2( ) = ⎜⎜+.... + . −... −⎟⎟⎟ ⋅ ⎜⎜+.... + . −... −⎟⎟⎟
n +1

⎜⎝ bn ⎜ ⎟
mn −bn ⎟ ⎠⎝ k m−k ⎠

The number of positive terms, in this case, will be:


88
k ⋅ bn + (m − k )(mn − bn ) = (k − m + k ) bn + (m − k ) bmn = (2k − m) bn + (m − k ) mn = bn+1
But bn is a linear recurrent and homogenous sequence.
1
From which it results that bn =
−2
(
n
) n
m n + (2k − m) , therefore P ≤ mn + (2k − m) .

7.89.
Let’s consider a polynomial P( x) of r < n − 1 degree which for the distinct numbers
x1 ,..., xn takes the values y1 ,..., yn . For 1 ≤ i ≤ n we consider the equations
x n−1 − si ,1 x n−2 + si ,2 x n−3 + ... + (−1) si ,n−1 = 0
which have the solutions x1 ,..., xi−1 , xi+1 ,..., xn
Then
n n
1

i =1
yi si ,h ∏
j =1 xi − x j
=0
j ≠i

with 0 ≤ h ≤ n − r − 2 , where by convention is noted si ,0 = 1 .

Solution
The polynomial which, for the distinct numbers x1 ,..., xn takes the values, respectively,
y1 ,..., yn is
( x − x2 )( x − x3 )...( x − xn ) ( x − x2 )( x − x3 )...( x − xn )
P( x ) = y1 + y2 + ... +
( x1 − x2 )( x1 − x3 )...( x1 − xn ) ( x2 − x2 )( x2 − x3 )...( x2 − xn )
( x − x1 )( x − x2 )...( x − xn−1 )
+ y2 ,
( xn − x1 )( xn − x2 )...( xn − xn−1 )
and this is the one with the smallest degree having this property (according to Lagrange).
The degree of P( x ) = r < n −1 , this implies that the coefficients of x n−1 , x n−2 ,..., x r+1 are
null. But the coefficients of xk , with r + 1 ≤ k ≤ n − 1 , are exactly the expressions:
n n
1
(−1)n−k−1 ∑ si ,n−k−1∏
i =1 j =1 xi − x j
j ≠i

When r + 1 ≤ k ≤ n − 1 , we have 0 ≤ n − k − 1 ≤ n − r − 2 and it is noted h = n − k − 1 .


si ,h is the sum of all the products of h factors ( h ≠ 0 ), which it is formed with the numbers
x1 ,..., xi−1 , xi+1 ,..., nn (that is the h -th sum of the relation of Viète, applied to the equation from the
problem).

89
7.90.
n
Let’s consider the polynomial with integer coefficients P( x ) = ∑ ai x i . Prove that for
i =0

⎛ p⎞ ⎛ p⎞ 1
p, q ∈ , if P ⎜⎜ ⎟⎟⎟ ≠ 0 , then P ⎜⎜ ⎟⎟⎟ > m , m ∈ *, m ≥ n .
⎜⎝ q ⎟⎠ ⎜⎝ q ⎟⎠ q

Solution
⎛ p⎞ 1
P ⎜⎜ ⎟⎟⎟ = an p n + an−1 p n−1q1 + ... + a1 p1q n−1 + a0 q n n ≥
⎜⎝ q ⎟⎠ q
1 1
≥ m
an p n + an−1 p n−1q + ... + a1 p1q n−1 + a0 q n ≥ m
q q
because: m ≥ n implies that
1 1
n
≥ m
q q
and an p n + an−1 p n−1q + ... + a1 p1qn−1 + a0 qn ∈ − {0} and its absolute value is ≥ 1 .

7.91.

s
Prove that Cns11 Cns22 ....Cn pp = Cnk1+...n p
s1+...+s p =k

Solution
We have
n1 n2 np n1+... n p
(1 + x) (1 + x) ...(1 + x) = (1 + x)
The coefficient x k from the right side is Cnk1+...n p .


s
The coefficient x k from the left side is Cns11 Cns22 ....Cn pp
s1 +...+s p =k

From this observation it results the equality.

7.92.
* and a j ∈ , j = i, m . If a 2j kn + a(j
2 k −1)n
Let’s consider k , m, n ∈ + .... + a nj + 1 = 0 for j = i, m ,
compute:
pn
1 ⎛ a ...a ⎞⎟
⎜⎜ i1 ih ⎟
∑ ∑
pn
E (a1 ,..., am ) = (a1 ,..., am ) + pn
+ ⎜ a ...a ⎟⎟⎟
(a1 ,..., am ) 1≤h≤m−1 (i 1 ,...,i j )∈Chm ⎜⎝ ih+1 im ⎠
knowing that:
{
C hm = (i 1 ,..., i p ) ∈ {1, 2,..., m} / is ≠ it for s ≠ t
h
}
90
Solution
We’ll note a nj = y j , j = 1, m . Because y j ≠ 1 , by multiplying eache equality from the
enounce, respectively by y j −1 we obtain:
0 = ( y j −1)( y 2j k + y 2j k−1 + ... + y j + 1) = y 2j k +1 −1 .
Then, because j ∈ i, m we have
p jπ p jπ
y j = cos + i sin , p j = 1, 2,...2k .
2k + 1 1 2k + 1
+ ... +
2k + 1 2k + 1
We will prove by recurrence in function of m ∈ * , that:
⎛ 1 ⎞ ⎛ 1 ⎞
E (a1 ,..., am ) = ⎜⎜ a1pn + pn ⎟⎟⎟...⎜⎜ ampn + pn ⎟⎟⎟ .
⎜⎝ a ⎠⎟ ⎜⎝
1 a ⎠⎟
m

1
For m = 1 we have E (a1 ) = a1pn + . We assume the property true for m , we must prove for
a1pn
⎛ 1 ⎞
m + 1 , that E (a1 ,..., am , am+1 ) = E (a1 ,..., am )⎜⎜⎜ampn+1 + pn ⎟⎟⎟. .
⎜⎝ a ⎟⎠
m+1

Then
p1π p π
E (a1 ,..., am , am+1 ) = 2 m cos p...cos m p ,
2k + 1 2k + 1
m
( p1 ,...., pm ) = {1,2,...2k }

7.93.
In a plane we consider the set of points whose coordinates are integers. Having the natural
numbers n, m, p with p ≥ 4 . Prove that there exists a polygon with p sides which has n points
on its border and m points in the interior.
Generalize in the space.

Solution:

……………………………………………………
……………………………………………………
A ………………………………………………..B’
C….……..………………………………………D
…………………………………………………….
……………A………………………B…………….
……………………………………………………..
The proof is done by construction.

91
We draw the segment [ AB ] which contains exactly n points. On the line located above of A '
and AB , we draw the segment [CD ] which contains exactly m points.
It is possible to designate the segments [ AA ' ] and [ BB ' ] (see the figure nearby) such that
they do not pass through any point, and also A ', B ' are between the line of CD and the line
situated above this; and the segment CD (that is the m points) is located in the interior of the
quadrilateral AA ' B ' B .
p ≥ 4 ⇒ p − 3 ≥ 1 . Because the polygon has p sides, by uniting the points A ', B ' with a line
polygonal which contains p − 3 sides and which is situated between the line of CD and the line
above this, without touching any point.

Generalization.
3
In the Euclidean space , one considers the set of points whose coordinates are
integers. If n, m, p are natural numbers, p ≥ 5 , then there exists a polyhedral with p faces which
contains n points on its border and m points in the interior.
The proof is also done by construction: one considers the segments [ AB ] and [CD ] with
their previous properties, but [ AA ' ] and [ BB ' ] are replaced by planes which keep the same
properties. Then one constructs two planes which pass through the previous planes, keeping of
course the required conditions.
At the end, the polygonal line A ' B ' will be replaced by a series of p − 4 planes which will be
constructed in a similar way.

7.94.
Determine the set A defined by:
a) 102 ∈ A
b) if x ∈ A then 1× 2 ∈ A
c) the elements of A are only obtained by the utilization of the rules a) and b) for a limited
number of times.

Solution:
We show that A = M
where

⎪ ⎫

M =⎪ ⎨1...1 2...2 / n ∈ *⎪


⎪ n
⎩ n ⎪


First of all, we show that A ⊃ M
One utilizes the reasoning by recurrence for n ∈ * , to show that 1...1 2...2 ∈ A . For n = 1
n n
one has 102 ∈ A according to the rule a). One assumes that the property is true for n , then
1...1 2...2 ∈ A and it will result that also 11...1 2...22 ∈ A according to the rule b)
n n n n

92
We show that A ⊂ M .
Let’s consider x ∈ A . If one only applies the rule a) it will result that x = 102 ∈ M . One cannot
apply the rule a) but one time. Now the rule b) cannot be applied if the rule a was applied. If one
applies b) one time, one obtains x = 11022 . By recurrence one proves that if one applies the rule
b) n times, then
x = 1...1 2...2 ∈ A
n+1 n+1

but
1...1 2...2 ∈ M
n+1 n+1

Therefore A ⊂ M , from which A = M .

7.95.
One constructs the set B such that
a) the elements 0, 9 and 1 belong to B
b) if x, y ∈ B then x − y and xy ∈ B
c) all the elements of B are obtained by the utilization of the rules a) and b) for a limited number
of times.

Find B.

Solution:
We prove that B = M , where
M = {0, x1... x p | 0 ≤ xi ≤ 9, i ∈ {1,... p} , p ∈ *, p < +∞} ∪ {1}
“ ⊃”
First of all, we show that {0;0,1;0, 2;…0,9;1} ⊂ B
0,9 et 0,9 ∈ B ⇒ 0,9 − 0,9 = 0 ∈ B
0,9 et 1 ∈ B ⇒ 0,9 − 1 = 0,1 ∈ B
0,9 et 0,1 ∈ B ⇒ 0,9 − 0,1 = 0,8 ∈ B
0,8 et 0,1 ∈ B ⇒ 0,8 − 0,1 = 0,7 ∈ B
……………………………………..
0,3 et 0,1 ∈ B ⇒ 0,3 − 0,1 = 0, 2 ∈ B
If y ∈ {0,1, 2,....,9} then 0,0....0 y = 0,1...0,10, y ∈ B
i i times
because one obtains by the utilization of the rules a) and b).
Let’s consider x ∈ M ; if x = 1 one has 1 ∈ B by the rule a); if x ≠ 1 one has 0 ≤ x < 1
if x = 0 one has 0 ∈ B . It remains therefore the case x = 0, x1... x p with
0 ≤ xi ≤ 9, i ∈ {1,..., p} , p ∈ *, p < +∞ and x ≠ 0 ;

93
without diminishing the generality one assumes x p ≠ 0 .
α) If xi ≠ 9 then 0, x1... x p = 0, w1 − 0,0 y2 − ... − 0,0...0 y p−1 0,0...0z p
p−1
p−2

with w1 = x1 + 1, y j = 9 − x j , − 2 ≤ j ≤ p −1 and z p = 10 − x p

Of course 1 ≤ w1 ≤ 9 since 0 ≤ x1 ≤ 8 ; one has 0 ≤ y j ≤ 9 , 2 ≤ j ≤ p −1 since


0 ≤ x j ≤ 9, 2 ≤ j ≤ p −1; 1 ≤ z p ≤ 9 1 ≤ z p ≤ 9 since 1 ≤ x p ≤ 9 .
One has: 0, w1 − 0,0 y2 + 0,00 y3 + 0,0...0 y p−1 + 0,0...0 z p = 0, w1 − 0,0 y2 ... y p−1 z p
p−2 p−1

Performing the subtraction:


0, w1 0...00 − 0,0 y2 ... y p−1 z p = 0, x1 x2 ... x p−1 x p
because 10 − z p = x p , w1 −1 = x1 and 9 − y j = x j , 2 ≤ j ≤ p −1
0, w1 and 0,0 y2 ∈ B ⇒ 0, w1 − 0,0 y2 ∈ B

0, w1 − 0,0 y2 and 0,0 y3 ∈ B ⇒ 0, w1 − 0,0 y2 − 0,0 y3 ∈ B


but the last absolute value is equal to 0, w1 − 0,0 y2 − 0,00 y3
By recurrence it results that x ∈ B , since
x = 0, w1 − 0,0 y2 − ... − 0,0...00 y p−1 − 0,0...00 z p =
p−2 p−1

= 1...1.,0, w1 − 0,0 y2 − 0,00 y3 − .... − 0,0...0 y p−1 − 0,0...0 z p


p−1 p−2 p−1

which is obtained by the utilization of the rules a) and b) a limited number of times.
β ) If x1 = 9 , one has 0,9...9 = 1− 0,0...01 ∈ B , ∀n ∈ *

n n−1

x = 0,9 x2 ... x p = 0, 9...9 − 0,0t2 ...t p


p

where t j = 9 − x j , 2 ≤ j ≤ p; 0 ≤ t j ≤ 9 since 0 ≤ x j ≤ 9 for 2 ≤ j ≤ p


To show that t = 0,0t2 ...t p ∈ B; 0 ≤ t j ≤ 9, 2 ≤ j < p

it is sufficient to see that the first decimal digit of t is zero, therefore different of 9, therefore
we’ll use the case α)
t∈B⇒ x∈B
"⊂"
0,9 and 1 ∈ M ⊂ [ 0,1]; {0;0,1;0,2;...0,9;1} ⊂ M . The operations of the rule b) applied a limited
number of times on the elements 0, 9 and 1 will also give elements of M , because: if α, β ∈ M
it results that αβ ∈ M and α − β ∈ M since 0 ≤ α − β ≤ 1 and 0 ≤ αβ ≤ 1 and α, β having a
limited number of decimals, then also αβ and α − β will have a limited number of decimals.

94
7.96.
⎧a
⎪ ⎫

Let’s consider the set I = ⎨ | a, b ∈ , b ≠ 0, (b, m) = 1⎬ which is called the ring of m-integers,

⎪b
⎩ ⎪


m ∈ , fix.
It is said that x ≡ y ( mod m ) in I , with x, y of I , if and only if there exists z ∈ I such that
z∈I .
We consider the element r of I ; find its class of equivalence modulo m in I .

Solution. 1)
The case r ∈
a) m ∉ {−1,0,1}
We note with r the class we are looking for, M m the set m , and


⎪ mk1 + h ⎫

M =⎪ ⎨ | k1 , k2 , h, p ∈ , 0 ≤ h, p < 1 / m,6( m, p ) = 1 and h − pr = M m ⎪


⎪ mk2 + p
⎩ ⎪


We show that r̂ = M
First of all we show that r̂ ⊃ M
Let’s consider x ∈ M . Then there exist k1 , k2 , h, p ∈ with the properties written above.
1) x ∈ I , that is x is a m-integer, because from (m, p ) = 1 and k2 ∈ it results that
(mk2 + pm) = 1 .
γ1
2) x ≡ r (mod m) in I, since there exists ∈ I , γ1 = k1 − rk2 + hm − rp , γ1 ∈
γ2
γ
because h − rp + M m and γ 2 = mk2 + p (therefore 1 is a m-integer because (γ 2 , m ) = 1 ,
γ2
γ mk + h − mk2 r − rp
such that: 1 m = 1 = x − r . Therefore x ∈ rˆ .
γ2 mk2 + p
We prove that r̂ ⊂ M
a
Let’s consider x ∈ rˆ . Therefore x = and b ≠ 0 as well as (b, m) = 1 such that x ≡ r ( mod m )
b
in I . But b can be written b = mk2 + p, 0 ≤ p < m , ( p, m ) = 1 and the same
a = mk1 + h, 0 ≤ h < m with k1 , k2 , h, p ∈ .
γ1
x ≡ r ( mod m ) in I , implies that m | x − r in I , therefore implies that there exists γ = ∈I
γ2
such that mγ = x − r and (γ 2 , m ) = 1 .

95
γ1 m (k1 − k2 r ) + h − rp
We consider γ irreducible. It results = . Because (γ1 , γ 2 ) = 1 and
γ2 m (mk2 + p )
(γ 2 , m ) = 1 it results: m | m (k1 − k2 r ) + h − rp from which m /| h - rp that is x ∈ M .
Remark 1.
For m = 0 we have I = and for m = ±1 we have I = .
b) m = 0 . Each class of equivalence contains only an element therefore r̂ = {r}
c) m = ±1 . It exists only one class of equivalence, therefore r̂ = I = .
2) The case r̂ = I − : a) m ∉ {−1,0,1}
Property 1.
r
We have m ∈ , I the ring of m-integers and 1 ∈ I . Then ∃a ∈ such that
r2
r1
∈ I ⇒ ( r2 , m) = 1 in I .
r2
Proof.
r1
∈ I then (r2 , m ) = 1 r1 , r2 ∈ .
r2
Let’s consider the Diophantine equation xr2 + ym = r (the unknowns being x and y ) which
admits integer solutions, since (r2 , m ) = 1 and 1 | r1 . Let’s consider x = x0 ∈ and y = y0 ∈ a
r1 ⎛r ⎞
particular solution. Taking a = x0 and γ1 = y0 . We have ≡ x0 ( mod m) since m | ⎜⎜ 1 − x0 ⎟⎟⎟
r2 ⎜⎝ r ⎠⎟
2

γ1 y y r
because there exists γ = = 0 ∈ I such that m 0 = 1 − x0 . Therefore x0 r2 + y0m = r1 .
γ2 r2 r2 r2
Remark 2.
r1
It exists an infinite number of integers α ∈ I such that = α ( mod m) in I , with m ≠ 0
r2
These numbers are, for example, particular solutions of the previous Diophantine equation.
Therefore, for r ∪ I − , ∃a ∈ such that r ≡ ar ( mod m ) in I and rˆ = aˆr which can be
determinate as in the case 1 a).
b) The sub-case m = 0 does not exist because it would result I = and therefore r ∈ .
c) m = ±1 . One has p = I = .

7.97.
Let’s consider the equation a1 x1m1 + ... + am ximn = b with ai , mi ∈ * for i ∈ {1,..., n} and
b∈ . Show that the equation has a limited number of natural numbers solutions.

Solution:
α) b > 0 . We note all ximi = yi . The initial equation becomes

96
a1 y1 + ... + an yn = b (1)
One can see that:
⎡b⎤
0 ≤ y1 ≤ ⎢ ⎥ (otherwise one has: a1 y1 > b )
⎢ a1 ⎥
⎣ ⎦
………………
⎡b⎤
0 ≤ yn ≤ ⎢ ⎥ (same explanation)
⎢ an ⎥
⎣ ⎦
___________________________
It results that: 0 ≤ the number of natural solutions of the equation (1)
n ⎛ ⎡ b ⎤⎞
(1) ≤ ∏⎜⎜⎜1 + ⎢ ⎥⎟⎟⎟ = M = finite number.
⎝ ⎣⎢ ai ⎦⎥ ⎟⎠
i =1 ⎜

xi = mi yi , i ∈ {1,..., n} (2)
Thus, if the number of solutions of the equation (1) is limited, then also, from (2) will result that
there exists a limited number of values for each x i .
β ) b = 0 . Then, the only natural solution is the banal solution.
γ ) b < 0 . The equation does not admit any natural solution.

7.98.
Let’s consider a1 , b > 0 for i ∈ {1,..., n} . Then, the equation a1x + ... + anx = b x , n ≥ 2
admits in addition a solution in the set of real numbers.

Solution:
The equation can have no solution, or it can have at least one solution.
If the equation admits at least one solution, let it be x0 ∈ one of those. Therefore
a1x0 + ... + anx0 = b x0 .
1) x0 > 0 . Let’s consider x > x0 . It results that x = x0 + t with t > 0 .
Let’s consider z = max {a1 ,..., an }
a1x + ... + anx = a1t a1x0 + ... + ant anx0 ≤ z t (a1x0 + ... + anx0 ) = z t b x0
If z ≥ b ⇒ z x0 ≥ b x0 ⇒ a1x0 + ... + anx0 > b x0 . From which z < b . Thus a1x + ... + anx < b x , ∀x > x0 .
Let’s consider x < x0 , It results that x = x0 − t with t > 0
a1x + ... + anx = a1−t a1x0 + ... + an−t anx0 > z−t (a1x0 + ... + anx0 ) = z−t b x0
Since z < b ⇒ z−t > b−t . Thus a1x + ... + anx > b x , ∀x < x0 from which x0 is the only solution of
the equation.
−x −x
⎛ 1 ⎞⎟ 0 ⎛ 1 ⎞⎟ 0 ⎛ 1 ⎞− x0
x0 x0 x0 ⎜ ⎜
2) x0 < 0 . One has: a1 + ... + an = b where ⎜ ⎟⎟ + ... + ⎜ ⎟⎟ = ⎜⎜ ⎟⎟⎟ ,
⎝⎜ a1 ⎠⎟ ⎝⎜ an ⎠⎟ ⎝⎜ b ⎠

97
with −x0 > 0 , therefore one has reduced this case to the first case.
3) The case x0 = 0 is not possible, because it would result a10 + ... + an0 = b0 where n = 1 , but
by hypotheses n ≥ 1 . Contradiction.

7.99.
Show that a congruence mod m , m ≠ 0 which contains unknowns, admits a limited
number of distinct solutions (two by two non-congruent)

Solution:
Each unknown cannot take but the values: 0,1, 2,..., m − 1 , that is at maximum m
values (a complete system of remainders modulo m ). If the equation – a congruence containing
n
n unknowns, then the maximum number of solutions will be m < ∞ .

Observation:
When m = 0 , the congruence becomes an equality (an equation) which can have an infinite
number of solutions, for example 0 x ≡ 0 ( mod 0) .

7.100.
Solve the linear congruence 2 x −1 ≡ 1− 6 y (mod 4)

Solution:
The congruence can be written: 2 x + 6 y − 2 ≡ 0(mod 4) . From which 2 x + 6 y − 2 = 4k
with k ∈ From where x + 3 y − 2k −1 = 0
(Remark: one cannot divide the congruence by 2 at beginning (one would obtain
x + 3 y ≡ 0(mod 2) ), because solutions will be lost).
The module of the congruence gives the rest 4. One has: x = −3 y + 2k + 1
where x ≡ y + 2k + 1(mod 4) .
2
One takes ( y , k ) ∈ {0,1, 2,3} , therefore all the possibilities.
But it is sufficient to give to k the values 0 and 1, since: for k = 3 ⇒ 2k ≡ 2 ⋅1(mod 4)
and for k = 2 ⇒ 2k ≡ 2 ⋅ 6(mod 4) .
If one successively gives the values (0,0),(1,0),(2,0),(3,0),(0,1),(1,1),(2,1),(3,1) to the
couple ( y , k ) one obtains for x , respectively the values 1, 2,3,0,3,0,1, 2 . But we are not
interested in k . Therefore
⎪⎧⎪ x ≡ 1(mod 4)(2)(3)(0)(3)(0)(1)(2)

⎪⎪⎩ y ≡ 1(mod 4)(1)(2)(3)(0)(1)(2)(3)

98
This shows that the number of solutions of this congruence is equal to (2,6)4 = 8 .

7.101.
Let’s consider ai ∈ , i ∈ {1, 2,..., n} , n ≥ 2 and b ∈ . Show that the equation
n

∑a x
i =1
i i = b admits an infinite number of solutions in the set of natural numbers if and only if
2
(a1 ,..., an ) divides b and if there exists (i0 , j0 ) ∈ {1,..., n} such that ai ⋅ a j < 0 .
0 0

(One notes a1 ,..., an the greatest common divisor of a1 ,..., an ).

Solution:
If we put the coefficients of the equation with the same denominator we can eliminate the
denominators and therefore we can say that all the ai and b are integers.
Necessity.
Since the equation has solutions in n , then it would also have in n because n ⊂ n .
It results that (a1 ,..., an ) divides b , according to a known theorem.
Let’s suppose by absurd that all the terms of the equation have the same sign, for
example positive (in the opposite case one multiplies the equation by -1):
- If b < 0 then the equation does not have any natural solution: contradiction.
⎡b⎤
- If b ≥ 0 , each unknown xi cannot take values which are between 0 and ⎢ ⎥ (natural
⎢ ai ⎥
⎣ ⎦
values), therefore a finite number of solutions; also contradiction. From which the
supposition is false, therefore it is not true that the terms of the equation have the same
sign.
Sufficiency.
Because (a1 ,..., an ) divides b it results that the equation has solutions in n . By hypothesis, the
equation has l has terms positives non nulls 1 ≤ l < n and k = n − l terms negatives non nulls.
One has 1 ≤ k ≤ n −1 < n . Then one writes:
l n

∑ a h xh −
h =1
∑ax
j =l +1
'
j j = b , 0 < a 'j = −a j

j ∈ {l + 1,..., n}

(One has supposed the first l terms positives and the following k negatives. In the other cases
one reorders the terms and (implicitly) one re-numerates them.)
Let’s consider 0 < M = [ a1 ,..., an ] the smallest common multiple of a1 ,..., an . One notes
ci = M / ai and i ∈ {1,..., n}
If one notes 0 < p = [ l , k ] the smallest common multiple of l and k.
We note l1 = p / l and k1 = p / k . If we note:

99
⎧⎪ xh = l1ch t + xh0 , 1≤ h ≤ l


⎪⎪ x j = k1c j t + x 0j , l +1≤ j ≤ n

⎪⎧⎪⎡ −x 0 ⎤ ⎡ −x 0j ⎤ ⎪⎫⎪
with t ∈ , t ≥ max ⎨⎢ h ⎥ , ⎢⎢ ⎥⎬ + 1

h , j ⎪⎢ l c ⎥
⎪⎩⎣ 1 h ⎦ ⎢⎣ 1 j ⎥⎦ ⎪⎪⎭
k c

where [ x ] represents the integer part of x , and ( x10 ,..., xn0 ) is a particular solution of the equation
(it was shown at the beginning of this proof that there exist integer solutions), then one obtains
an infinite number of natural solutions for our equation.

7.102.
Let’s consider the linear equation with integer coefficients
n

∑a x
i =1
i i =b

2
a) If there exists (i0 , j0 ) ∈ {1,..., n} such that ai0 ≡ ±1(mod a j0 )
then the equation admits solutions in the set of integer numbers.
b) In this case, solve it.

Solution:
a) Let’s consider ai0 ≡ ±1(mod a j0 ) ⇔ ∃h0 ∈ : ai0 − h0 = ±1 . It results that
d ∼ (ai0 , a j0 ) , therefore d /| ai0 − h0a j0 , then d | ±1 , from where d ∼ 1 .
Because (ai0 , a j0 ) ∼ 1 one has (a1 ,..., an ) ∼ 1 , but 1| b . Therefore the equation admits integer
solutions.
n
b) ∑ a x = ∑ a x + (h a
i =1
i i
i ≠i0
i i 0 j0 ± 1) xi0 + a j0 x j0 = ∑ ai xi +a j0 ( x j0 + h0 xi0 ) ± xi0 =
i ≠i0
i ≠ j0 i ≠ j0

= ∑ ai xi +a j0 t ± xi0 = b
i ≠i0
i ≠ j0

where t = x j0 + h0 xi0 . It results:

100
⎧⎪ ⎛ ⎞⎟
⎪⎪ ⎜⎜ ⎟⎟
⎪⎪⎪ xi0 = ±⎜⎜⎜ ∑ ai xi +a j0 t − b⎟⎟⎟
⎪⎪ ⎜⎝ ii≠≠ij00 ⎟⎟⎠
⎪⎪
⎪ ⎛
⎪⎪⎪ ⎜⎜ ⎟⎟⎞
⎨ x j0 = t − h0 xi0 = ±⎜⎜h0 ∑ ai xi +a j0 (h0 xi0 ± 1) t − h0b⎟⎟⎟
⎪⎪ ⎜⎜ i≠i0 ⎟⎟
⎪⎪ ⎝ i≠ j0 ⎟⎠
⎪⎪
⎪⎪with xi ∈ , i ∉ {i0 , j0 } , and t ∈
⎪⎪
⎪⎪
⎪⎪

(parameters).

7.103.

It is given the system:


f j ( x1 ,..., xn ,[ x1 ],...,[ xn ], { x1 } ,..., { xn }) = b j ∈ , j = 1, n
where f j are linear functions which have their coefficients in and [ x ], { x} represent
respectively the integer part and the fractional part of x .
Find a method for solving this system.

Solution:
(1) Writing xi = [ xi ] + { xi } with [ xi ] ∈ , 0 ≤ { xi } < 1 , i = 1, n one obtains (after the
elimination of denominators) the equivalent system:
(2) g j ([ x1 ],...,[ xn ], { x1 } ,..., { xn }) = c j ∈ , j = 1, n
where the g j are now linear functions which have their coefficients in
One solves this system considering that { x1 } ,..., { xn } are the unknowns. Since:
n n

∑ α [ x ]+ ∑ β {x } = c ,
i =1
ij i
j =1
ij j i i = 1, n

and because αij [ xi ], ci ∈ , i, j = 1, n it results:


n

∑ β {x } ∈
j =1
ij j , ∀i ∈ 1, n

One applies the method of the substitution. One calculates {x }


j1 of an equation
1 ≤ j1 ≤ n , and replaces it in the other equations. It will remain a linear system of n-1 equations
with n-1 unknowns.

101
We proceed in the same way with this new system. (If one obtains during this procedure
an impossible equation then the system (2) does not admit solutions. Stop.). At the end one
obtains:
δ { x jn } = h ([ x1 ],...,[ xn ]) + k ∈ , then δ { x jn } ∈ . It results: { x jn } = 0 where
p p
{x } = ε ,
jn p∈ * but such that 0 ≤
δ
< 1; δ ,k ∈ .
These two cases can be written as a single case:
p p
{ x jn } = δ , p ∈ and 0 ≤ δ < 1
Let s jn be the number of solutions for { x jn } . Now one will follow the inverse procedure until
the determination of all the { x } . One replaces the value(s) of { x }, j = 1, n
jn j in the previous
system of 2 equations with 2 unknowns. One obtains the values of { x } . jn−1

Let s jn−1 be the number of these.


The inverse procedure continues until when one determines { x j1 } which has s j1 solutions:
({ j1 ,..., j1} = {1, 2,..n})
One remarks that { xi } ∈ , i = 1, n . If the system has solutions it results that these are in
.
n
Until now one has obtained ∏s
i =1
i solutions.

(3) Reporting all the values of { x1 } ,..., { xn } in (2) on obtains a linear system of n equations with
n unknowns: [ x1 ],...,[ xn ] which will be solved in integer numbers. One normally solves in n
,
and if the solution belongs to n then this solution is correct (one then performs the relation (1)
to obtain x1 ,..., xn ); otherwise, it is not convenient.
One will execute the paragraph (3) for all the values of { x1 } ,..., { xn } .
n
Thus, the system is well solved. The number of solutions of this is ≥ 0 and ≤ ∏ si .
i =1

7.104.
Solve in the equation: 3x − 7 y + 2 z = −18

Solution:
The general solution in is the following: x = k1 , y = k1 + 2k2 , z = 2k1 + 7k2 − 9
with: k1 , k2 ∈

102
⎡ k ⎤
Because x ≥ 0, y ≥ 0, z ≥ 0 it results that k1 ≥ 0 and also that k2 ≥ ⎢− 1 ⎥ + 1 and
⎣⎢ 2 ⎦⎥
⎡ 9 − 2k1 ⎤ ⎡ 2 − 2k1 ⎤
k2 ≥ ⎢ ⎥ + 1 , that is one has k2 ≥ ⎢ ⎥ + 2 . From which, the general solution in will
⎢⎣ 7 ⎥⎦ ⎢⎣ 7 ⎥⎦
⎡ 2 − 2k1 ⎤
be: x = k1 , y = k1 + 2k2 , z = 2k1 + 7k2 − 9 with k1 , k2 in and k2 ≥ ⎢ ⎥+2
⎢⎣ 7 ⎥⎦

7.105.
Solve in the equation: 2 x + 15 y + 9 z = 44

Solution:
⎡ 44 ⎤
One has 0 ≤ y ≤ ⎢ ⎥ = 2 .
⎢⎣ 15 ⎥⎦
⎡ 44 ⎤
A) y = 0 ⇒ 2 x + 9 z = 44 ⇒ 0 ≤ z ≤ ⎢ ⎥ = 4
⎢⎣ 9 ⎥⎦
a) z = 0 ⇒ x = 22
35
b) z = 1 ⇒ x = ∉
2
c) z = 2 ⇒ x = 13
d) z = 3 ⇒ x ∉
e) z = 4 ⇒ x = 4
B) y = 1 ⇒ 2 x + 9 z = 29 ⇒ 0 ≤ z ≤ 3
a) z = 1 ⇒ x = 10
b) z = 3 ⇒ x = 1
c) z ∈ {0, 2} ⇒ x ∉
C) y = 2 ⇒ 2 x + 9 z = 14 ⇒ 0 ≤ z ≤ 1
a) z = 0 ⇒ x = 7
b) z = 1 ⇒ x ∉
All the solutions are:

(22, 0, 0); (10,1,1); (7, 2, 0)


(13, 0, 2); (1,1,3)
(4,0, 4)
Therefore there is a limited number of solutions.

103
7.106.
Solve in the equation:
17 x1 + 20 x2 −18 x3 = −34

Solution:
One writes the equation thus:
20 x2 − 18 x3 + 17 ( x1 + 2) = 0
One notes x1 + 2 = t ∈ . It results: 20 x2 − 17 ( x3 − t ) − x3 = 0
That is: 20 x2 −17h − x3 = 0
One notes x3 − t = h ∈
It results:
⎧⎪⎪ x1 + 2 = t
⎨ ⇒ x1 = t − 2 = ( x3 − h ) − 2 = −h − 2 + 20 x2 − 17h = 20 x2 − 18h − 2
⎪⎪⎩ x3 − t = h
The general solution is:
⎧⎪ x = 20k − 18k − 2
⎪⎪ 1 1 2
⎪⎨ x = k
⎪⎪ 2 1

⎪⎪ x3 = 20k1 − 17k2 , (k1 , k2 ) ∈ 2


7.107.
Solve in the set of integer numbers the equation:
15x −17 y + 9 z = α
Where α is an integer parameter.

Solution:
The equation can be written:
15x + 9( z − 2 y ) + y = α
Or again:
15x + 9t + y = α (1)
where t = z − 2 y (2)
It results from (1) that: y = −15 x − 9t + α (3)
It results from (2) that: z = t + 2 y
It results from (3) that: z = −30 x − 17t + 2α
The integer general solution is:
⎧⎪ x = k1
⎪⎪
⎨ y = −15k1 − 9k2 + α (k1 , k2 ) ∈ 2
⎪⎪
⎪⎪⎩ z = −30k1 − 17k2 + 2α

104
(one has noted x = k1 and t = k2 )

7.108.
Solve in the equation 3x + 70 y − 35z + 6 = 76 .

Solution:
The equation can be written:
⎛ ⎞⎟ ⎛ ⎞⎟
⎜⎜ ⎟ ⎜⎜
3⎜ x + 23 y − 12 z + 2 w − 25⎟⎟ + ⎜ y + z ⎟⎟⎟ = 1
⎜⎝⎜ ⎟⎠ ⎜⎝⎜ t ⎠⎟
t1 2

With these notation one has: 3t1 + t2 = 1 , equation which admits the general solution:
⎧⎪⎪t1 = k
⎨ with k ∈
⎪⎪⎩t2 = −3k + 1
Therefore y + z = t2 = −3k + 1 from where y = −z − 3k + 1 with z ∈ (1)
In a similar way x + 23 y −12 z + 2 w − 25 = t1 = k , where
x − 23z − 69k + 23 −12 z + 2 w − 25 = k (one has used (1)).
Thus x = 35z − 2 w + 70k + 2
The general solution of the equation in 4 will be:
⎪⎧⎪ x = 35z − 2 w + 70k + 2
⎨ z , w, k ∈
⎪⎪⎩ y = −z − 3k

7.109.
Solve the equation xy + 5z − 2 = 0 in the set of integer numbers.

Solution:
∀x ∈ one can write: x = 5k1 + r1 with k1 ∈ and r1 ∈ {0,1, 2,3, 4} (1)
∀y ∈ , one can also write: y = 5k2 + r2 , with k 2 ∈ and r2 ∈ {0,1, 2,3, 4} (2)
Using (1) and (2) in the initial equation, one has
5(5k2 k2 + k1r2 + k2 r1 + z ) + r1r2 − 2 = 0
It results that 5 divides ( r1r2 − 2) . Therefore ( r1 , r2 ) ∈ {(1, 2), (2,1) , (3, 4), (4, 3)} (3)
From where, the general solution will be:

105
⎧⎪
⎪⎪
⎪⎪ x = 5k1 + r1
⎪ y = 5k + r

⎪ 2 2

⎪⎪⎪ 2 − r1r2
⎪⎪ z = −5k1k2 − k1r2 − k2 r1 +
⎩ 5

With (k1 , k2 ) ∈ 2
(arbitrary parameters), and ( r1 , r2 ) ∈ {(1, 2), (2,1) ,(3, 4), (4, 3)} .
The unknown z has been obtained by starting from the initial equation because there
were known the values of x and y .

7.110.
y
It is given the equation x 2 + 3k1 + 2 = (3k2 + 1) . Show that the equation does not admit
a natural solution, for any (k1 , k2 ) ∈ 2
. Generalize.

Solution:
y
One has: 3k1 + 2 ≡ 2(mod 3) and 3k 2 + 1 ≡ 1(mod 3) therefore (3k2 + 1) ≡ 1(mod 3)
where x 2 ≡ 1 − 2 ≡ 2(mod 3)
a) if x = M 3 + 1 ⇒ x 2 ≡ 1 ≡
/ 2(mod 3) .
b) if x = M 3 + 2 ⇒ x 2 ≡ 1 ≡
/ 2(mod 3)
c) if x = M 3 + 2 ⇒ x 2 ≡ 0 ≡
/ 2(mod 3)
From which ∀x ∈ , x 2 ≡ / 2(mod 3) , Thus the equation does not admit a natural solution.
Generalization:
w
The equation x 2 + 3 y + 2 = (3z + 1) does not admit an integer solution.
The proof is the same. First of all one shows that if k 2 ≠ 0 then y ≥ 0 , because if y < 0
it would result that an integer number (the left member of the equation) is equal to a non-integer
number (the right member).

7.111.
Solve the equation xy + 4t − 7 w + 14 = 0 in the set of integer numbers.

Solution:
One writes: xy + 4t − 8w + 12 + w + 2 = 0 we note t − 2 w + 3 = v which will be a new
unknown.

106
The equation becomes: xy + 4v − w + 2 = 0 where w = −xy − 4v − 2 . And
t = v + 2 w − 3 = v + 2 (− xy − 4v − 2) − 3 − 2 xy − 7v − 7 . If one changes the notations (for the
sake of mathematical esthetic) one has the integer general solution:
⎪⎧⎪ x = k1
⎪⎪
⎪ y = k2

⎪⎪t = −2k1k2 − 7k3 − 7
⎪⎪
⎪⎪⎩w = −k1k2 − 4k3 − 2
with (k1 , k2 , k3 ) ∈ 3 (parameters).

7.112.
Show that the equation:
2 x 2 + 5 xy − 12 y 2 − 2 x + 3 y − 1 = 0
does not have a solution in the set of integer numbers.

Solution:
The equation can be written:
2 x 2 + 8 xy − 3xy −12 y 2 − 2 x + 3 y = 1
Or again x (2 x − 3 y ) + 4 y (2 x − 3 y ) − 1(2 x − 3 y ) = 1
Therefore (2 x − 3 y )( x + 4 y − 1) = 1
As x, y ∈ it results that one has the following possibilities:
a) either
(2 x − 3 y ) = 1 (1)
x + 4 y −1 = 1 (2)
(2) implies x = −4 y + 2 , and substituting in (1) it comes
3
−11 y = −3 ⇒ y = ∉
11
b) or
2 x − 3 y = −1 (3)
x + 4 y −1 = −1 (4)
(4) ⇒ x = −4 y and substituting in (3) it comes
1
−8 y − 3 y = −1 ⇒ y = ∉
11
Therefore the equation does not have a solution in .

107
7.113.
Prove that the equation: 5 x 2 + 50 y 2 − 26 xy − 8 x − 46 y + 15 = 0
does not admit a solution in the set of natural numbers.

Solution I:
The equation can be written:
(4 x 2 + 49 y 2 + 9 − 28 xy + 12 x − 42 y ) + ( x 2 + y 2 + 4 + 2 xy − 4 x − 4 y ) + 2 = 0
Or
2 2
(2 x − 7 y + 3) + ( x + y − 2) + 2 = 0
But this equation does not admit a solution in , because
2 2
(2 x − 7 y + 3) + ( x + y − 2) + 2 > 0 . Therefore it does not admit even more a solution in .

Solution II:
The equation can be written:
5 x 2 + 2 (−13 y + 4) x + (50 y 2 − 46 y + 15) = 0
Δ = b12 − ac = 169 y 2 + 16 − 104 y − 250 y 2 + 230 y − 75 = − ⎡⎢(9 y − 7) + 10⎤⎥ < 0
2

⎣ ⎦
It results that the equation does not admit a solution in . Therefore it does not admit
even more a solution in .

7.114.
Solve in the set of integer numbers the equation:
x3 − 3 y = 2

Solution:
The equation can be written: x 3 − 2 = 3 y .
Therefore x 3 − 2 is divisible by 3, that is
x3 = M 3 + 2
x = 3k + r, r = 0,1, 2, k ∈
x = 3k ⇒ x 3 = M 3 ≠ M 3 + 2
x = 3k + 1 ⇒ x 3 = M 3 + 1 ≠ M 3 + 2
x = 3k + 2 ⇒ x 3 = M 3 + 8 = M 3 + 2
Let’s consider x = 3k + 2, k ∈
3
x 3 − 2 (3k + 2) − 2
y= = = 9k 3 + 18k 2 + 12k + 2
3 3
The solution of the equation is:
108
⎧⎪ x = 3k + 2
⎪⎨
⎪⎪⎩ y = 9k 3 + 18k 2 + 12k + 2
k∈

7.115.
Prove that the equation x 4 − 7 y1... yn = 14 z + 10, n ≥ 1 does not admit any integer
solution.

Solution:
One can write: x 4 − 7 ( y1... yn − 2 z − 1) = 3 , or x 4 − 3 = 7 ( y1... yn − 2 z − 1)
From which 7 divides x 4 − 3 , that is x 4 = M 7 + 3 .
Let’s consider x = M 7 + r with r ∈ {0, ±1, ±2, ±3} . Then x 4 = M 7 + r 4 ; but r 4 ∈ {04 ,14 , 24 ,34 }
But one can see that r 4 ≠ 3(mod 7) , therefore x 4 ≠ M 7 + 3 . It results that the equation does not
admit any integer solution.

7.116.
Solve in the set of integer numbers the equation:
5 x 4 − 6 y = 20

Solution:
5 x 4 − 6 y = 20 ⇔ 6 y = 5( x 4 − 4)
Therefore y is divisible by 5.
Let’s consider y = 5z , z ∈
The equation becomes:
x4 − 4
5 x 4 − 30 z = 20 ⇔ x 4 − 6 z = 4 ⇔ z = ∈
6
Therefore x 4 ≡ 4(mod 6)
It results:
x = 6k + 2 , x = 6k + 4 , k ∈
The integer solutions of the equation are:
⎪⎧⎪ x = 6k + 2
⎪⎪ 4
⎪⎪⎨ y = 5 (6k + 2)
⎪⎪ 6
⎪⎪
⎪⎪⎩k ∈

109
⎪⎧⎪ x = 6k + 4
⎪⎪ 4
⎪⎪ (6k + 2) − 4
⎨ y = 5
⎪⎪ 6
⎪⎪
⎪⎪⎩k ∈

7.117.
Solve in the set of integer numbers the equation:
4x y − 7z = 5

Solution:

one has 4 x y ∈ .
4 x y = 7 z + 5 ; since z ∈
y ⎧⎪ 1 1 ⎫⎪ 4x y −5 1
Therefore x = α ∈ where x ∈ ⎨± , ± ⎬ but z =
y
, from where x y = or
⎪⎪⎩ 2 4 ⎪⎪⎭ 7 2
1
x y = α ∈ A solution is: x = −2, y = −1, z = −1 , since x y = admits the solution in integer
2
numbers x = −2, y = −1 .
If x y = α ∈ , the initial equation becomes 4α − 7 z − 5 = 0 which admits the integer
general solution.
⎧⎪⎪α = 7k + 3

⎪⎪⎩ z = 4k + 1
with k ∈ (parameter).
xy −3
Therefore x y − 7k − 3 = 0 , with x, y, z of . It results k = . But x = M 7 + r ,
7
with r ∈ {0,1, 2,..,6} .
y
(7 s + r ) − 3
One writes x = 7 s + r, s ∈ , and then ∈ if and only if r = 3 or 5
7
x = 7 s + 3 ⇒ y = 6t + 1, t ∈
x = 7 s + 5 ⇒ y = 6t + 5, t ∈
Therefore the integer solutions of the equation are:
⎧⎪ x = −2
⎪⎪
⎨ y = −1
⎪⎪
⎪⎪⎩ z = −1

110




⎪ x = 7 s + 3, s ∈


⎨ y = 6t + 1, t ∈



⎪ 4(7 s + 3)6 t+1 − 5
⎪ z=


⎩ 7




⎪ x = 7 s + 5, s ∈


⎨ y = 6t + 5, t ∈



⎪ 4(7 s + 5)6 t+1 − 5
⎪ z=


⎩ 7

7.118.
Solve in the equation:
y
x + 5z − 2 = 0

Solution:

The equation can be written: x y + 5 z − 2 = 0


x has the form 5k1 + r1 , k1 ∈ , r1 ∈ {0,1, 2,3, 4} and it must have x y ≡ 2 ( mod 5) or
y
(5k1 + r1 ) ≡ r1y ≡ 2 (mod 5)
Thus r1 ≠ 0, r1 ≠ 1, r1 ≠ 4
For r1 = 2 one has 2 M 4 +1 ≡ 2(mod 5) and
For r1 = 3 one has 3M 4 +3 ≡ 2 ( mod 5)
Therefore





⎪ x = 5k1 + 2 : k1 ∈


⎨ y = 4k2 + 1: k2 ∈ (1)




4 k +1
2 − (5k1 + 2) 2

⎪ z=

⎩ 5
and
x = 5k1 + 3, k1 Є Z;

y = 4k2+3, k2 Є Z; (2)

z = {2 – (5k1 + 3)4k2+3} / 5.

One can observe that z ∈ in (1) and also in (2). It results that:
111
2 4 k2 +1 ≡ 2 (mod 5) and 34 k2 +3 ≡ 2 (mod 5)
The integer general solution is obtained by bringing together (1) and (2).

7.119.
Solve in the equation: x ! = y z

Solution:
⎧⎪ ⎧
⎪⎪ x = 0 ⇒ ⎪⎪⎨ y = 1 and z ∈
⎪⎪ ⎪⎪⎩ y ∈ * and z=0
⎪⎪
⎪⎪ ⎧
⎪⎨ x = 1 ⇒ ⎪⎪⎨ y = 1 and z ∈
⎪⎪ ⎪⎪ y ∈ * and z=0

⎪⎪
⎪⎪ x ≥ a ⇒ y = x ! and z = 1
⎪⎪
⎪⎪⎩

We prove the last affirmation, because the first two are banal. One supposes (by absurd)
that z = k ≥ 2 . One has x ! = y k . One excludes the case x = 2 which implies y = 2 and z = 1 .
Therefore x ≥ 3 . From which x ! contains at least two prime divisors (2 and 3). Therefore y k
also admits at least two prime divisors (2 and 3), from which y admits at least two prime divisors
(2 and 3).
Let’s consider x1 ,..., x p to be all the prime numbers inferior or equal to x (one has
α α
proved that p ≥ 2 ) x ! = x1α1 ... x p p , α p ∈ *
, 1 ≤ i ≤ p . Thus x1α1 ... x p p = y z . From which it
results that α1 : z, α2 : z....α p : z
One considers x p , the greatest prime number inferior or equal to x . In accordance to the
x
theorem of Chebyshev, for ≤ x p ≤ x there exists at least a prime number. Therefore
2
x ⎡ x ⎤ ⎡x⎤
< x p ≤ x . Then α p = ⎢⎢ ⎥⎥ + ⎢⎢ 2 ⎥⎥ + ... = 1 . It results that z = 1 . Thus ( s ) represents all the
2 ⎢⎣ x p ⎥⎦ ⎢⎣ x p ⎥⎦
solutions of the equation.

7.120.
Determine the general form of the solution in the set of integer numbers of the equation:

112
n

∑x i
pi

i =1
m
=1
∑x
rj
j
j =1

where m, n, pi , rj ∈ *
, m ≤ n, rj < pi , j ∈ {1,..., m} , where all the pi are even numbers and all
the rs are odd numbers.

Solution:

One has

n m m m rj

∑ xipi = ∑ x jj ≤ ∑ x jj = ∑ x j
r r

i =1 j =1 j =1 j =1

and
n pi m rj m rj

∑ xi = ∑ xj ≥ ∑ xj
i =1 j =1 j=1
m pi m rj

From which ∑xi=1


i = ∑ x j , it results:
j=1
m pj n pj
rj
∑x
j =1
j − xj + ∑
j =m+1
xj =0
pj rj pj
Since xj x j ≥ 0 and x j ≥ 0 , ∀j ∈ {1,..., n} one has x j = 0 for j ∈ {m + 1,..., n} and
pj rj
xj − xj = 0 for j ∈ {1,..., m} . Therefore x j ∈ {0,1, −1} for j ∈ {1,..., m} .
The general form of the solution in integer numbers of the equation is:

⎪ x1 = ε1 ,..., xm = εm , xm+1 = ... = xn = 0; with ε j = 0 or 1



⎪ m



∑ εj ≠ 0


1


⎪ x1 = ξ1 ,..., xm = ξm , xm+1 = ... = xn = 0; with ξ j = 0 or 1




m


⎪ ∑ξj ≠ 0

⎩ 1
The number of solutions is 2 (Cm1 + Cm2 + ... + Cmm ) = 2 (2m − Cm0 ) = 2m+1 − 2

7.121.
Determine the linear equation which admits the following solution in the set of integer numbers:

113

⎪ x1 = 3k1 − 7k2 + 5


⎨ x2 = k1 + 2k2


⎩ x3 = 4k1 + 13k2 − 71


where k1 and k2 are parameters in .

Solution:
The equation has three unknowns x1 , x2 , x3 .
a2' a3' b'
Its general form is: a1' x1 + a2' x2 + a3' x3 = b , with ai' , bi' ∈ , i = 1, 2,3 . Or x1 +
'
x 2 + x = '
' 3
a1 a1 a1
By differently noting the coefficients, we obtain: x1 + a2 x2 + a3 x3 = b with a2 , a3 , b ∈ .
One can write for k1 , k2 arbitraries in :
3k1 − 7k2 + 5 + a2 k1 + 2a2k2 + 4a3k1 + 13a3k2 − 71a3 = b

For k1 = k2 = 0 ⇒ 5 − 71a3 = b (1)


For k1 = 0, k2 = 1 ⇒ −7 + 2a2 + 13a3 = 0 (2) (4)
For k1 = 0, k2 = 1 ⇒ −7 + 2a2 + 13a3 = 0 (3)
(One has used (1) to obtain (2) and (3).)
But (4) is a system of three equations with three unknowns which will be normally
solved.
From (3) it results a2 = −4a3 − 3 . From (2) it results now −7 + 13a3 − 8a3 − 6 = 0 . Or
13 67
a3 = , therefore a2 = .
5 5
25 − 923 898
From (1) it results b = =− .
5 5
Therefore the equation is:
5 x1 − 67 x2 + 13x3 = −898

7.122.
One considers a natural n ≥ 3 and a ∈ . Solve the inequality:
⎡ x + a⎤ ⎡ x−a⎤
⎢ ⎥+⎢ ⎥ ≥ [ x ] ; Discussion.
⎢⎣ n ⎥⎦ ⎢⎣ n ⎥⎦

Solution:
Let’s consider x = ng + r, 0 ≤ r < n, r ∈ , q ∈

114
⎡ x⎤
I) If a = nqa , qa ∈ , then the inequality of the problem becomes: 2 ⎢ ⎥ ≥ [ x ] (1) which is
⎣⎢ n ⎦⎥
equivalent to (2 − n ) q ≥ 0 .
Therefore x ∈ M 1 = { y | y = nq + r : 0 ≤ r < n, r ∈ , q ≤ 0, q ∈ }
II) If a ≠ nqa : qa ∈ , then a can be written
a ≠ nqa + ra : 0 < ra < n, ra ∈ , qa ∈ (2)
One can suppose 0 < a < n , since, with (2), the inequality of the problem becomes
equivalent to
⎡ x + ra ⎤ ⎡ x − ra ⎤
qa + ⎢ ⎥ − ra + ⎢ ⎥ ≥[ x]
⎣⎢ n ⎦⎥ ⎣⎢ n ⎦⎥
⎡ x + a⎤ ⎡ x−a⎤
One has ⎢ ⎥+⎢ ⎥ ≥[ x] (3)
⎢⎣ n ⎥⎦ ⎢⎣ n ⎥⎦
which is equivalent to (2 − n ) q + E ( r ) ≥ [ r ] (4)
⎧⎪−1 if r < min {a, n − a}
⎡ r + a ⎤ ⎡ r − a ⎤ ⎪⎪⎪
where E ( r ) = ⎢ ⎥+⎢ ⎥ = ⎨0 if min {a, n − a} ≤ r < max {a, n − a}
⎢⎣ n ⎥⎦ ⎢⎣ n ⎥⎦ ⎪⎪
⎪⎪+1 if r ≥ max {a, n − a}

1) 0 ≤ r < min {a, n − a} then (4) ⇔ (2 − n ) q − 1 ≥ [ r ] .
where q ≤ −1 .
α) ( q = −1) ⇒ (4) ⇔ (n − 3 ≤ [ r ]) ⇒ x ∈ M 2 = { y | y = qn + r,0 ≤ r < min {a, n − a} ,[ r ] ≤ n − 3}
β ) ( q = −2) ⇒ (4) ⇔ (2n − 5 ≥ [ r ]) ⇒ x ∈ M 3 = { y | y = qn + r,0 ≤ r < min {a, n − a} ,[ r ] ≤ 2n − 5}
γ ) ( q ≤ −3) ⇒ (4) is true
⇒ x ∈ M 4 = { y | y = qn + r,0 ≤ r < min {a, n − a} , q ≤ −3, q ∈ }
2)min {a, n − a} ≤ r < max {a, n − a} .Then (4) ⇔ (2 − n ) q ≥ [ r ]
α) ( q = −1) ⇒ x ∈ M 5 = { y | y = −n + r : min {a, n − a} ≤ r < max {a, n − a} ,[ r ] ≤ n − 2}
β ) (−q ≤ −2) ⇒ x ∈ M 6 = { y | y = qn + r, min {a, n − a} ≤ r <, max {a, n − a} , q ≤ −2, q ∈ }
3)max {a, n − a} ≤ r < n ⇒ (4) ⇔ ((2 − n ) q _1 ≥ [ r ]) ⇒ x ∈ M 7 =
= { y | y = qn + r, max {a, n − a} ≤ r < n, q ≤ −1, q ∈ }
And we analyzed all the cases.

7.123.
Find a method for solving in natural numbers the equation:
m n

∑ x + a = b⋅
i =1
i
p c ⋅∏ y j
j =1

115
*
with a, b, c ∈ , p ∈ .

Solution:
The equation will have an infinity of natural solutions when b ⋅ c ≠ 0 .
∀h ∈ {1,..., n −1} one takes yh ∈ arbitrary. We construct yn such that cy1... yn = k p with
k ∈ . (1)
Let’s consider yn' the smallest natural number which has the property (1). (There exists a
such yn' because, if one writes each yh = ∏ piαih with αih ∈ and pi being the i-th prime
i

(positive) number h ∈ {1,..., n − 1} , one takes yn' = ∏ piβin , with βin ∈ and the βin are
i
n−1
selected such that γi + ∑ αih + βim = M p , where one has written c = ∏ piγi with γi ∈ , and
h=1 i

βin (for each i) is the smallest natural number which verifies this property.)
We construct yn= yn' ⋅ t p , with t ∈ *
, t being a parameter. The equation becomes
m m

∑ xi + a = bkt where the unknowns are x1,... xn , t . One has:


i =1
∑ x + a = αt , where we
i =1
i noted

bk = α ∈ .

m
A) If bc = 0 , then the equation ∑ x + ε = 0 does not admit a solution in natural numbers.
i =1
i

B) If bc ≠ 0 , then α ≠ 0 . The equation admits an infinity of natural solutions:


∀s ∈ {1,..., m −1} , xs = αws + rs where bc ≠ 0 ≤ rs ≤ α − 1 , a rs ∈ and ws is a natural
m
parameter, and xm = αwm + rm , where 0 ≤ rm ≤ α − 1 but rm is chosen such that ∑r +a = M
i =1
i α

(one has noted M α a multiple of α ) and also rm ∈ , wm = natural parameter.

7.124.
It is given the equation P ( x1 ,.., xn ) = 0 with P ( x1 ,.., xn ) a second degree polynomial in
x1 ,.., xn with real coefficients. Show that Δxi is perfect square if and only if Δ x j is a perfect
square. (By Δ xh one has noted the determinant of the initial second degree equation relative to
the unknown xh .)

Solution:
Necessity.
(The reciprocal proof will be similar.)

116
The equation can be written: Ai xi2 + Bi xi + C xi = 0 (1)
Where A is a constant and Bi a first degree linear function in x1 ,.., xi−1 , xi+1 ,..., xn . ( A is a
constant, because otherwise it would result that P has a degree strictly superior to 2.)
Δxi being a perfect square, it implies that Δ xi = ki2 ( x1 ,.., xi−1 , xi+1 ,..., xn ) with ki a first degree
linear function in x1 ,.., xi−1 , xi+1 ,..., xn .
⎛ B + ki ⎞⎛ ⎟⎟⎜⎜ x − Bi − ki ⎞⎟⎟ = 0 , where A 2 Axi + Bi − ki 2 Axi + Bi + ki = 0 (2)
(1) becomes A⎜⎜⎜ xi − i
2 A ⎟⎠⎝⎜ 2 A ⎠⎟
i
⎝ 2A 2A
Since 2 Axi + Bi + ki are first degree functions in x1 ,.., xn one can compute x j in function of
x1 ,.., x j−1 , x j+1 ,..., xn .

⎜ x j − f1 ( x1 ,.., x j−1 , x j+1 ,..., xn )⎟⎟ ⎜⎜ x j − f 2 ( x1 ,.., x j−1 , x j+1 ,..., xn )⎟⎟
⎛ ⎞⎛ ⎞
(2) becomes A⎜⎜ ⋅
⎟⎟ ⎜ ⎟⎟ by notation.
⎜⎝ 2A ⎟⎠ ⎜⎝ 2A ⎟⎠

By notation B ( x j − g1 ( x1 ,.., x j−1 , x j+1 ,..., xn )) ⋅ ( x j − g 2 ( x1 ,.., x j−1 , x j+1 ,..., xn )) = 0


where g1 , g 2 are also linear functions in x1 ,.., x j−1 , x j+1 ,..., xn . Therefore Δ x j is a perfect square.

7.125.
Having aσ ( n ) x n + ... + aσ (1) x + a = 0 all the equations obtained by circular permutations
of coefficients, on the set {an ,...., a1 , a0 } , ai ∈ * , 0 ≤ i ≤ n, n even.
a) Show that these equations admit a real common root if and only if an + .... + a1 + a0 = 0
b) Let x0 be the real common root, S the sum of all the roots of the equations, P the product of
all the roots of the equations. Then:
P ⎛ n−2 a a a ⎞
S − x0 + (n + 1) = −⎜⎜ ∑ n−i + 0 + 1 ⎟⎟⎟
x0 ⎜⎝ i=0 an−i−2 an−1 an ⎟⎠

Solution:
In total one has n+1 equations.
a) One has on aσk ( n ) x 0n + ... + aσk (1) x0 + aσk (0) = 0, 0 ≤ k ≤ n .
Therefore let x0 be the real common root. One does the sum of all the relations (1), and it comes:
S1 ( x 0n + x 0n−1 + ... + x10 + 1) = 0
It results S1 = 0 or x 0n + x 0n−1 + ... + x10 + 1 = 0 , but it does not exists a x0 ∈ which annulets
the equation x n + ... + x1 + 1 = 0 , with n even. From which S1 = 0 ; but S1 = an + ... + a1 + a0
Reciprocal: S1 = 0 . This implies that aσk ( n )1n + ... + aσk (1)1 + aσk (0) = 0, 0 ≤ k ≤ n .
Therefore all the equations admit the real common root x0 = 1 .

117
⎛ a ⎞ ⎛ a ⎞ ⎛ a ⎞ ⎛ a ⎞
b) S − x0 = ⎜⎜− 1 − 1⎟⎟⎟ + ⎜⎜− 0 − 1⎟⎟⎟ + ⎜⎜− n − 1⎟⎟⎟ + ... + ⎜⎜− 2 − 1⎟⎟⎟
⎜⎝ an ⎟⎠ ⎜⎝ an−1 ⎟⎠ ⎜⎝ an−2 ⎟⎠ ⎜⎝ a0 ⎟⎠
a a a a
P = 0 n n−1 ⋅⋅⋅ 1 = 1
an an−1 an−2 a0
P ⎛ a a n−2
a ⎞
Therefore S − x0 + (n + 1) = −⎜⎜ 0 + 1 + ∑ n−i ⎟⎟⎟ .
x0 ⎜⎝ an−1 an i=0 an−i−2 ⎟⎠

7.126.
x x x
Solve inthe equation ( x + 1) + ( x + 2) = ( x + 3)
(Amer. Math. Monthly, 1985)

Solution:
Obviously x > −1 , since the bases of the powers must be non-negative, and that for
x = −1 the operation 0−1 does not have sense.
x
If one has ( x + 3) ≠ 0 , we divide the equation by this. It results that
x x
⎛ x + 1 ⎞⎟ ⎛ x + 2 ⎞⎟
⎜⎜ ⎜
⎜⎝ x + 3 ⎟⎟⎠ + ⎝⎜⎜ x + 3 ⎠⎟⎟ = 1
x x
⎛ x + 1 ⎞⎟ ⎛ x + 2 ⎞⎟
Let’s consider g1 ( x ) = ⎜⎜ and g 2 ( x ) = ⎜⎜ and f ( x ) = g1 ( x ) + g 2 ( x ) which have
⎜⎝ x + 3 ⎟⎟⎠ ⎜⎝ x + 3 ⎟⎟⎠
the same domain of definition ]−1, +∞[
We show that g1 and g2 are strictly declining, from which it results that f is also strictly
decreasing.
On constructs the graphic representations of g1 and g2 .
For
The line with the equation is a horizontal asymptote when x tends toward
The line with the equation is a vertical asymptote when x tends toward
The graph of is found in the figure (1).
For g1
−2
x
⎡ x +3 ⎤ x + 3
⎢⎛⎜ −2 ⎞⎟ −2 ⎥ 1 1
lim g1 ( x ) = lim ⎢⎜⎜1 + ⎟⎟ ⎥ = x−2 = 2 <
x→∞ x →∞ ⎢⎝ x + 3⎠ ⎥ x 4
⎣⎢ ⎦⎥
x = 0 ⇒ g1 (0) = 1
1
From which the line with the equation y = 2 is a horizontal asymptote when x tends toward
e
+∞ .
The graph of g1 is found in the figure (2).

118
For g 2 : x = 0 ⇒ g 2 (0) = 1; x = −1 ⇒ g 2 (−1) = 2
−1
1+ ⋅x
⎡ x +3 ⎤ x +3
⎢⎛⎜ −1 ⎞⎟ −1 ⎥ 1 1
lim g 2 ( x ) = lim ⎢⎜⎜1 + ⎟ ⎥ = e−1 = <
x→∞ x→∞ ⎢⎝ x + 3 ⎟⎠ ⎥ e 2
⎣⎢ ⎦⎥
1
From where the line from equation y = is a horizontal asymptote when x tends toward +∞
e2
The graphic of g2 can be seen in figure (2)
From (1) and (2) it results that g1 and g2 are strictly declining on ]−1, +∞[ , therefore one has
the same property for f . Because sf (2) = 1 it results that x = 2 is the only real solution of the
equation.

Y y

(1) (2)

1
1
g1 g2
1 1
e2 e

-1 O x -1 O x

7.127.
Knowing that b2 − 4ac is a perfect square, find a method for solving in the set of integer
numbers the equation ax 2 + bxy + cy 2 + dx + fy + e = 0 , with a, b, c, d , f , e integers.

Solution:
We try to write the equation using the form
ax + bxy + cy 2 + dx + fy + e = (α1 x + β1 y + γ1 )(α2 x + β2 y + γ 2 ) + δ
2

where α1 , β1 , γ1 , δ ∈ , i ∈ {1, 2} . One has


α1α2 x 2 + α1β2 xy + α1γ 2 x + β1α2 xy + β1β2 y 2 + β1γ 2 y + γ1α2 x + γ1β2 y + γ1γ 2 + δ =

119
ax 2 + bxy + cy 2 + dx + fy + e .
By identification it results:

⎪ α1α2 = a


(1) ⎨β1β2 = c


⎩α1β2 + β1α2 = b


and
⎧γ1α2 + α1γ 2 = d



(2) ⎨β1γ 2 + = eγ1β2 = f


⎩γ1γ 2 = e


which is a second degree system of 6 equations with 7 unknowns α1 , α2 , β1 , β2 , γ1 , γ 2 , δ . From (1)
a c c a
one obtains α2 = , β2 = and α1 + β1 = b
α1 β1 β1 α1
a α
It results that cz + = b where z = 1 ∈ . From which cz 2 − bz + a = 0; but it is necessary
z β1
that Δ z to be a perfect square, that is b 2 − 4ac = k 2 , k ∈ , what it is satisfied by hypothesis.
α b±k 2c b±k
Therefore z = 1 = . Then β1 = α1 ⋅ and β2 =
β1 2c b±k 2α1
One replaces in (2), and one obtains:
a 2c b±k 1 γ
γ1 + α1γ 2 − d ; α1γ 2 + α1 = f from which one finds 1 and α1γ 2 as
α1 b±k 2 α1 α1
rationales. (3) since one has a linear system of 2 equations with two unknowns
⎛ γ ⎞
⎜⎜⎜ w = 1 , t = α1γ 2 ⎟⎟⎟
⎝ α1 ⎠⎟
From (3) it results that one can express γ1 and γ 2 in function of α1 . From the equation
γ1γ 2 + δ = e one can take δ in function of α1 . One gives a convenient value to α1 and one thus
determinates all the unknowns. One has: (α '1 x + β '1 y + γ '1 )(α '2 x + β '2 y + γ '2 ) = δ ' . One puts
the coefficients to have the same denominator and one eliminates this. Then one finds
α '1 , α '2 , β '1 , β '2 , γ '1 , γ '2 , δ ' ∈ .
Now one decomposes δ ' an integer factors and one tries all the possibilities, which will give a
system of Diophantine equations: αi' x + βi' y + γi' = di with d1d 2 = −δ ' and i ∈ {1, 2}

7.128.
n
One considers the equation ∑a x
i =0
i
i
= 0 with all the real coefficients an ≠ 0 .and n ≥ 0

natural. Show that if (n − 1) an2−1 − 2nan an−2 < 0 then the equation does not have all its roots in
.

120
Solution:
2 2 2 2 2 2
S = ( x1 − x2 ) + ( x1 − x3 ) + ... + ( x1 − xn ) + ( x2 − x3 ) + ... + ( x2 − xn ) + ... + ( xn−1 − xn ) =
= (n −1) ⎡⎣⎢ x12 + ... + xn2 ⎤⎦⎥ − 2 [ x1 x2 + x1 x3 + ... + x1 xn + x2 x3 + ... + x2 xn + ... + xn−1 xn ] =
= (n − 1)[ x1 + ... + x2 ] + ⎡⎣−2 − 2 (n − 1)⎤⎦ [ x1 x2 + ... + x1 xn + x2 x3 + ... + x2 xn + ... + xn−1 xn ] =
2

a a 1
= (n −1) n−2 1 + n−2 (−2n ) = 2 ⎡⎢⎣(n −1) an2−1 − 2nan an−2 ⎤⎥⎦ < 0
an an an
(One has noted x1 , x2 ,..., xn the roots of the equation.)
It results that the given equation does not have all its roots in , since otherwise it would result
that S ≥ 0 .
Remark: for n=2 one obtains the well-known result that if the determinant of a second
degree equation, Δ < 0 , then the equation has complex roots.

7.129.
Solve the following system:
n

∑x
i =1
i = αi , 1 ≤ j ≤ n with n ≥ 2
i≠ j

Solution:
One explicitly writes the system:

⎪ x2 + x3 + ... + xn−1 + xn = α1



⎪ x1 + x3 + .... + xn−1 + xn =α2


⎪...................................................


⎩ x1 + x2 + x3 + .... + xn−1
⎪ =αn

One does the subtraction between the first equation and each other equation. One has:
−x1 + xk = α1 − αk , 2 ≤ k ≤ n
One replaces xk = α1 − αk , 2 ≤ k ≤ n in the first equation and one obtains:
(n − 1) x1 + (n − 1) α1 = α1 + α2 + ... + αn
1 ⎡
From which x1 = −(n − 2) α1 + α2 + ... + αn ⎤⎦
n −1 ⎣
One determines in a similar way the unknowns x2 ,..., xn . The solution of the system is:
1 ⎡
xi = α1 + ... + αi−1 − (n − 2) αi + αi+1 + ... + αn ⎤⎦ , 1 ≤ i ≤ n .
n −1 ⎣

121
7.130.
Solve in the set of integer numbers the system:
⎧⎪⎪−17 x + 52 y = 130

⎪⎪⎩35 x − 27 y + 26 z = 84

Solution:
We solve in integer numbers the first equation of the system, which is a Diophantine
equation, and its general solution will be:

⎪⎧ x = 52t − 26
(1) ⎪⎨ , with t ∈ .
⎪⎪⎩ y = 17t − 6
By replacing the values of x and y in the second equation, one has:
(2) 1361t + 26 z = 832
with (t , z ) ∈ 2
One solves this equation in integer numbers; its general solution will be
⎧⎪⎪t = 26k
⎨ , with k ∈
⎪⎪⎩ z = −1361k + 32

This is used in (1):


⎧⎪⎪ x = 52 ⋅ 26k − 26
⎨ , with k ∈
⎪⎪⎩ y = 17 ⋅ 26k − 6

Therefore, the general solution of the initial system is:



⎪ x = 1352k − 26


⎨ y = 442k − 6 , with k ∈


⎩ z = −1361k + 32

Observation: The method which was used is the normal substitution, which is also utilized to
solve in real numbers.

7.131.
Let’s consider a linear homogeneous system having as associated matrix A ∈ M (m, n, )
, which admits the rank r ( A) < n . (The rank of a matrix is the order of the greatest non-null
determinant which can be extracted from this matrix.) Show that the system admits non-banal
integer solutions

Solution:

122
n
One considers the initial system ∑a x
j =1
ij j = 0 , 1 ≤ i ≤ m , with all the aij ∈ . One

brings the coefficients to the same denominator and eliminates it. One obtains a system which
has all the coefficients integers. One notes r (Δ) = r < n (according to the hypothesis). If one
eliminates the secondary equations then r principal equations remain. We normally solve in n ,
applying the method of Cramer. Without diminishing the generality one supposes that x1 ,..., xr
are the principal variables.
Thus xr+1 ,..., xn will be the secondary variables. Because r < n , there exists at least a secondary
variable. The real solutions of the system are:
1 n
xh = ∑ bht xt , 1 ≤ h ≤ r , with all the bht , Δ integers, where Δ is the determinant which
Δ t=r+1
contains the columns 1,..., r and the lines 1,..., r .
If one notes xt = Δkt , r + 1 ≤ t < n , with kt ∈ (parameters) from which
n
xh = ∑b
t = r +1
k , 1≤ h ≤ r
ht t

It results an integer solution r undetermined for our system. If we give non null values to parameters
k r+1 ,..., kn we obtain a particular integer solution non trivial .

7.132.
Determine the matrices A and B of order n such that:
(1 x x 2 ... x n−1 ) ⋅ A = (1 1+x 1+x 2 ... 1+x n−1 ) and
(1 1+x 1+x 2 ... 1+x n−1 ) ⋅ B = (1 x x 2 ... x n−1 )
for any x real.
a) Compute Am and B m , for m ∈ N * .
p p
b) Show that Ak B e = B e Ak and ( AB ) = ( BA) ∀p, e, k ∈ N * .
s s s
c) Show that if ∑k = ∑e
i =1
i
i =1
i then ∏A
i =1
ki
B ei = I n , where I n is the unitary matrix of order n ;
*
and ki , ei ∈ N .

Solution:
Let’s consider the matrix
⎛a11...a1 j ...a1n ⎞⎟
⎜⎜ ⎟
⎜⎜a ...a ...a ⎟⎟
2n ⎟
A = ⎜⎜ ⎟
21 2j
⎜⎜.................... ⎟⎟⎟
⎜⎜ ⎟⎟
⎜⎝an1...anj ....ann ⎟⎟⎠

123
We note v = (1 x x 2 ... x n−1 ) and u = (1 1+x 1+x 2 ... 1+x n−1 ) .
Multiplying v by the first column of A we obtain:
a11 + a21 x + ... + an1 x n−1 = 1 , ∀x ∈ . One does x = 0 and it results a11 = 1 , therefore
a21 x + ... + an1 x n−1 = 0 , ∀x ∈ ; that is this is the null polynomial because it has more than
n − 1 roots; from which a21 = ... = an1 = 0 .
One multiplies v by the column j of A ( 2 ≤ j ≤ n ) and one obtains:
a1 j + a2 j + ... + a j−1 j x j−2 + a jj x j−1 + ... + anj x n−1 = 1 + x j−1 + ... + anj x n−1 = 0 ∀x ∈ .
For x = 0 one finds a1 j = 1 . Therefore
a2 j + a2 j + ... + a j−1 j x j−1 + (a jj −1) x j−1 + ... + anj x n−1 = 0 ∀x ∈ . This polynomial, also, is null,
thus a2 j = a j−1, j = ... = anj = 0 and a jj −1 = 0 , or a jj = 1 , with 2 ≤ j ≤ n . It results that
⎛1 1 1 ... 1⎟⎞
⎜⎜ ⎟
⎜⎜0 1 0 ... 0 ⎟⎟
A = ⎜⎜ ⎟⎟
⎜⎜.................0 ⎟⎟⎟
⎜ ⎟
⎜⎝0 ........0 1 ⎟⎟⎠
⎛b11...b1 j ...b1n ⎞⎟
⎜⎜ ⎟
⎜⎜b ...b ...b ⎟⎟
2n ⎟
Let’s consider B = ⎜⎜ ⎟ . One multiplies u by the first column of B , and one finds:
21 2j
⎜⎜....................⎟⎟⎟
⎜⎜ ⎟⎟
⎜⎝bn1...bnj ....bnn ⎟⎟⎠
b11 + b21 + ... + bn1 + b21 x + ... + bn1 x n−1 = 1 , ∀x ∈ .
For x = 0 , that implies b11 + b21 + ... + bn1 = 1 . One has also b21 x + ... + bn1 x n−1 = 0 , ∀x ∈
From which b21 = .... = bn1 = 0
Multiplying u by the column j of B , we obtain:
b1 j + b2 j + ... + bnj + b2 j x + ... + b jj x j + ... + bnj x n−1 = x j−1 , ∀x ∈ , 2 ≤ j ≤ n . If x = 0 ,
one finds b1 j + b2 j + ... + bnj = 0 . Therefore,
b2 j x + ... + b jj x j + ... + bnj x n−1 = x j−1 ⇔ b2 j x + ... + b j−1 j x j−2 + (b jj −1) x j−1 + ... + bnj x n−1 = 0
∀x ∈ .
The same thing: b2 j x = ... = b j−1 j = b j+1 j = ... = bnj = 0 and b jj −1 = 0 , or b jj = 1 . Because
b1 j + b2 j + ... + bnj = b1 j + 1 = 0 ⇒ b1 j = −1 . From which
⎛ ⎞
⎜⎜1 − 1... − 1 ⎟⎟
⎜⎜0 1 0 ... 0⎟⎟
⎜⎜ ⎟⎟
B = ⎜⎜.................0 ⎟⎟⎟
⎜⎜ ⎟⎟
⎜⎜.................0 ⎟⎟⎟
⎜⎜0.............01 ⎟⎟
⎝ ⎠
a) One shows by recurrence that

124
⎛1m ...... m⎞⎟
⎜⎜ ⎟
⎜⎜0 1 0 ... 0⎟⎟
⎜⎜ ⎟⎟
A = ⎜⎜.................0 ⎟⎟⎟
m

⎜⎜ ⎟⎟
⎜⎜.................0 ⎟⎟⎟
⎜⎜0.............01 ⎟⎟
⎝ ⎠
The case m = 1 is obvious.
One supposes the property true for m , it must be shown for m + 1 :
⎛1 m + 1 ...... m + 1⎞⎟
⎜⎜ ⎟
⎜⎜0 1 0 ...... 0 ⎟⎟
⎜ ⎟⎟
` A = A A⎜⎜.........................0 ⎟⎟⎟
m+1 m ⎜

⎜⎜ ⎟⎟
⎜⎜.........................0 ⎟⎟⎟
⎜⎜0....................0.1 ⎟⎟
⎝ ⎠
In a similar mode one proves that
⎛1 − m ..... − m ⎞⎟
⎜⎜ ⎟⎟

⎜ 0 1 ....0. 0 ⎟⎟
B m = ⎜⎜ ⎟
⎜⎜........................ ⎟⎟⎟
⎜ ⎟
⎜⎝0 0 ....0. 1⎟⎟⎠
b) One sees that AB = BA − I n .
Ak B e = A... A B...B = A... A BA B... B ... = BAk B e−1 = ... =
k e k −1 e−1
p p
= B A ⇒ AB = BA ⇒ ( AB) = ( BA) .
2e k

t
c) Ak1 B e1 Ak2 B e2 .... Aks B es = Ak1 Ak2 B e1 B e2 ... Aks B es = At B t = ( AB) = I nt = I n ,
s s
where one has noted t = ∑k = ∑e
i =1
i
i =1
i

7.133.
⎛ab ⎞
Let’s consider the matrix: A = ⎜⎜ ⎟⎟⎟ with a, b ∈ .
⎜⎝ba ⎟⎠
1) Compute the matrix An , n ∈ * .
2) Discuss the limit:
n

∑ det ( A )
k =1
k

lim ⎛ n ⎞
det ⎜⎜ ∑ Ak ⎟⎟⎟
n→∞

⎜⎝ k =1 ⎠
Solution
I) One will prove by the recurrence method that

125
⎡n⎤
⎢ ⎥
⎛αn βn ⎟⎞ ⎣⎢ 2 ⎦⎥
An = ⎜⎜ ⎟⎟ , with αn = ∑ Cn2i a n−2i b 2i (1)
⎜⎝βn αn ⎟⎠ i =0

and
⎡ n+1⎤
⎢ ⎥
⎣⎢ 2 ⎦⎥
βn = ∑ Cn2i−1a n−2i+1b 2i−1 (2)
i =0

where [ x ] represents the integer part of x .


The case n =1 is evident. We suppose the property true for n , it must be proved that the
property is also true for n + 1
⎛αn βn ⎞⎟⎛ab ⎞⎟ ⎛aαn + bβn bαn + aβn ⎞⎟ ⎛⎜αn+1....βn+1 ⎞⎟
An+1 = An ⋅ A = ⎜⎜ ⎟⎜⎜ ⎟ = ⎜⎜ ⎟=⎜ ⎟,
⎜⎝βn αn ⎟⎟⎠⎜⎝ba ⎟⎟⎠ ⎜⎝bαn + aβn aαn + bβn ⎟⎟⎠ ⎜⎝βn+1....αn+1 ⎟⎟⎠
⎡n⎤ ⎡ n+1⎤
⎢ ⎥ ⎢ ⎥
⎢⎣ 2 ⎥⎦ ⎢⎣ 2 ⎥⎦
it must be proved that αn+1 = aαn + bβn = ∑ Cn2i a n+1−2i b 2i + ∑ Cn2i−1a n−2i+1b 2i =
i =0 i =1
⎡n⎤
⎢ ⎥
⎢⎣ 2 ⎥⎦
= a n+1 + ∑ (Cn2i + Cn2i−1 ) a n+1−2 i b2 i + ∑C 2 i−1 n+1−2 i 2 i
n a b (3)
i =1 n+1
i=
2

n +1
if ∈ .
2
n +1 ⎡ n ⎤ ⎡ n + 1⎤
i) If n = 2k , then ∉ , then the second sum from (3) is equal to zero, and ⎢ ⎥ = ⎢ ⎥
2 ⎢⎣ 2 ⎥⎦ ⎢⎣ 2 ⎥⎦
it results that the expression of αn+1 from (1).
n +1
ii) If n = 2k + 1 , then ∈ and the second sum of (3) is equal to Cnn a 0bn+1 = bn+1 , from
2
where it results the expression of αn+1 from (1) because the first sum from (3) will have the same
⎡n⎤
terms of i = 0 until i = ⎢ ⎥ .
⎣⎢ 2 ⎦⎥
Also, it must be shown that
⎡n⎤ ⎡ n+1⎤
⎢ ⎥ ⎢ ⎥
⎣⎢ 2 ⎦⎥ ⎣⎢ 2 ⎦⎥
βn+1 = bαn + aβn = ∑ Cn2i a n−2i b 2i+1 + ∑ Cn2i−1a n−2i+2b 2i−1 =
i =0 i =1
⎡ n+2 ⎤ ⎡ n+1⎤
⎢ ⎥ ⎢ ⎥
⎢⎣ 2 ⎥⎦ ⎢⎣ 2 ⎥⎦
= ∑C i =1
2 i−2
n a n−2 i+2b 2 i−1 + ∑ Cn2 i−1a n−2 i+2b 2 i−1 =
i =1

⎡ n+1⎤
⎢ ⎥
⎣⎢ 2 ⎦⎥
∑ (C
i =1
2 i−2
n + Cn2i−1 ) a n+1−2i+1b 2i−1 + ∑ (C
n+2
2 i−2
n a n−2i+2b 2i−1 ) ; (4)
i=
2
n+2
if ∈
2

126
n+2
if ∈
2
n+2
i) If n = 2k + 1 , then ∉ and therefore the second sum from (4) is null. Also
2
⎡ n + 1⎤ ⎡ n + 2 ⎤
⎢ ⎥=⎢ ⎥ , from which it results the expression of βn+1 from (2).
⎢⎣ 2 ⎥⎦ ⎢⎣ 2 ⎥⎦
n+2
ii) If n = 2 k , then ∈ , from which the second sum of (4) is equal to
2
⎡n⎤ ⎡n⎤ ⎡n⎤
⎢ ⎥ +2−2 n−2 ⎢ ⎥ +2−2 2 ⎢ ⎥ +2−1
⎣⎢ 2 ⎦⎥ ⎣⎢ 2 ⎦⎥ ⎢⎣ 2 ⎦⎥
C n a b = b n+1 .
Similar like in (4), if one does the addition between the two sums, it results that for the first sum
n + 2 ⎡n + 2⎤
i takes the values from 1 until =⎢ ⎥ , from which it results the expression of βn+1
2 ⎢⎣ 2 ⎥⎦
from (2)/
n n n
2) det Ak = αk2 − βk2 ⇒ ∑ det ( Ak ) = ∑ (αk2 − βk2 ) = ∑ (αk − βk )(αk + βk ) =
k =1 k =1 1
n n
= ∑ ( a − b ) ( a + b) = ∑ ( a 2 − b 2 )
k k k

1 1

(a 2 − b2 )
n
n −1
2 2
C1) if a − b ≠ 1 then ∑ (a
1
2
−b )
2 k
= (a − b
2 2
) a 2 − b2 −1
;

⎛ n k n k ⎞⎟
⎜⎜∑ α ∑ β ⎟
⎜ 1 ⎟⎟⎟
2 2
n
⎜ ⎛ n k ⎞⎟ ⎛ n k ⎞⎟ ⎛ n k ⎟⎞
∑ A = ⎜⎜ n
k 1
⎟⎟ ⇔ det ⎜∑ A ⎟⎟ = ⎜∑ α ⎟⎟ − ⎜∑ β ⎟⎟ =

⎝⎜ 1

⎠ ⎝⎜ 1

⎠ ⎝⎜ 1
⎜⎜ k⎟ ⎠
n

⎜⎜∑ β ∑ α ⎟⎟
1 k

⎝ 1 1 ⎠
⎛ n n ⎞⎛ n n ⎞ ⎛ n ⎞⎛ n ⎞
= ⎜⎜∑ α k − ∑ β k ⎟⎟⎟⎜⎜∑ α k + ∑ β k ⎟⎟⎟ = ⎜⎜∑ (α k − β k )⎟⎟⎟⎜⎜∑ (α k + β k )⎟⎟⎟ =
⎜⎝ 1 1 ⎠⎝⎜ 1 1 ⎠ ⎜⎝ 1 ⎠⎝⎜ 1 ⎠
⎛ n k ⎞⎛
n
k⎞
= ⎜⎜∑ (a − b) ⎟⎟⎟⎜⎜∑ (a + b) ⎟⎟⎟
⎜⎝ 1 ⎠⎝⎜ 1 ⎠
n
n
( a − b) − 1
∑ ( a − b)
k
C2) If a − b ≠ 1 , then = ( a − b)
1 a − b −1
n
n
( a + b) − 1
∑ ( a + b)
k
C3) If a + b ≠ 1 , then = ( a + b) ;
1 a + b −1
Discussion
The cases:
A) The conditions C1, C2, C3 are satisfied.
B) It exists at least a condition from these which is not satisfied.

127
⎛ n ⎞
⎜⎜ det Ak ⎟⎟
⎜⎜ ∑ ( )⎟⎟ (a − b − 1)(a + b − 1) ( a 2 − b2 ) −1
n

A) lim ⎜⎜ 1 ⎟
⎟= ⋅ lim =L
⎞⎟⎟
⎛ n k ⎟⎟
( )
2 n
2 2
n→∞ ⎜
⎜⎜ det ⎜⎜∑ A ⎟⎟ a − b − 1 n→∞
a 2
− b − ( a − b )
n
− ( a + b )
n
+ 1
⎜⎝ ⎜⎝ 1 ⎟⎟
⎟⎠⎟⎠
One has the sub-cases:
A.I. a 2 − b2 < 1
A.II. a 2 − b2 > 1
A.III. a 2 − b2 = 1 ⇒ a 2 − b2 = −1 because one has C1.
A.I. It admits the situations:
(a − b −1)(a + b −1)
A.I.1. a − b < 1 and a + b < 1 ⇒ L = −
a 2 − b2 − 1
A.I.2. a − b < 1 and a + b > 1 ⇒ L = 0
A.I.3. a − b < 1 and a + b = 1 ⇔ a − b < 1 and a + b = −1 , because one has C3. It results that
the limit does not exist.
A.I.4. a − b > 1 and a + b < 1 ⇒ L = 0
A.I.5. a − b > 1 and a + b > 1 ( this case is not possible, because it would result a 2 − b2 > 1 ,
which is in contradiction with A.I.)
A.I.6. a − b > 1 and a + b = 1 (the same this case is not possible).
A.I.7. a − b = 1 and a + b < 1 ⇔ a − b = −1 and a + b < 1 , because one has C2. It results that
the limit does not exist.
A.I.8. a − b = 1 and a + b = 1
A.I.9. a − b = 1 and a + b > 1 these two cases don’t exist because of A.I.

A.II. admits the situations:


A.II.1. a − b > 1 and
1
1−
(a
− b2 )
2 n
(a − b −1)(a + b −1)
a −b >1 ⇒ L = ⋅ lim =
a 2 − b2 −1 n→∞ 1 1 1
1− n − n +
( a + b) ( a − b) ( a 2 − b 2 )
n

(a − b −1)(a + b −1)
=
a 2 − b2 −1
A.II.2. a − b > 1 and a + b < 1 ⇒ L = 0
A.II.3. a − b > 1 and a + b = 1 ⇔ a − b > 1 and a + b = −1 , because one has C3. It results
that the limit does not exist.
A.II.4. a − b = 1 and a + b > 1 ⇔ a − b = −1 and a + b > 1 , because one has C2. It results
that the limit does not exist.

128
A.II.5. a − b = 1 and a + b = 1
A.II.6. a − b = 1 and a + b < 1
A.II.7. a − b < 1 and a + b = 1 These 4 cases don’t exist because of A.I.
A.II.8. a − b < 1 and a + b < 1
A.II.9. a − b < 1 and a + b > 1 ⇒ L = 0
n
(a − b −1)(a + b −1) (−1) −1
A.III. It results: L = − lim
(−1) + 1− (a − b) − (a 2 + b2 )
n
a 2 − b2 −1 n→∞
n n

a 2 − b2 1
But a − b ≠ 0 and a + b = − , therefore
a −b a+b
n
(a − b −1)(a + b + 1) (−1) −1
L= lim n
a 2 − b2 −1 n→∞ n ⎛ 1 ⎟⎞ n
(−1) + 1− (a − b) − ⎜⎜⎜ ⎟
⎝ a − b ⎠⎟
a − b ∉ {−1,1} because of C2 and because if a − b = −1 it would result
−1 = a 2 − b 2 = (a − b)(a + b) = −(a + b) , that is a + b = 1 , contradiction with C3.
A.III. admits the situations:
1
A.III.1. a − b < 1 ⇒ − >1 ⇒ L = 0
a+b
1
A.III.2. a − b > 1 ⇒ − <1 ⇒ L = 0
a −b
A.III.3. a − b = 1 ⇔ a − b = 1 or a − b = −1 , which is not possible.
B) It exists at least a condition between C1, C2 or C3 which is not satisfied.
n
- if the condition C1 is not satisfied, then a 2 − b2 = 1 ⇒ ∑ det ( Ak ) = n
1
n
- if the condition C2 is not satisfied, then a − b = 1 ⇒ ∑ (a − b) = n
k

1
n
- if the condition C3 is not satisfied, then a + b = 1 ⇒ ∑ (a + b) = n
k

1
We analyze all the possibilities for this case.
B.I. a − b = 1 and a + b ≠ 1 . It results a 2 − b2 ≠ 1 , a = b + 1 and
(a 2 − b2 )(a 2 − b2 )
n
−1 1 a + b −1
L = lim ⋅ ⋅ =
n→∞ a 2 − b2 −1 n (a + b) ⎡(a + b)n −1⎤
⎣⎢ ⎦⎥
(a 2 − b2 )(a + b −1) 1 (2b + 1)n −1
= lim =0
(a + b)(a 2 − b2 −1) n→∞ n (2b + 1)n −1
n
B.II. a − b = 1 and a + b = 1 ⇒ a 2 − b 2 = 1 ⇒ a = 1 and b = 0 ⇒ L = lim =0
n→∞ n : n

129
B.III. a − b ≠ 1 and a + b = 1 ⇒ a 2 − b 2 ≠ 1 ,
(a 2 − b2 ) ⎡⎢⎣(a 2 − b2 ) −1⎤⎥⎦ 1
n

a − b −1
a = 1 − b ⇒ L = lim ⋅ ⋅ =
n→∞
2
a − b −1 2
n (a − b) ⎡(a − b)n − 1⎤
⎢⎣ ⎥⎦
(a 2 − b2 )(a − b −1) 1 (1 − 2b) − 1
n

= 2 ⋅ lim ⋅ =0
(a − b2 −1)(a − b) n→∞ n (1− 2b)n −1
B.IV. a − b ≠ 1 and a + b ≠ 1 ⇒ a 2 − b 2 = 1 because of B).
a − b −1 a + b −1
⇒ L = lim ⋅ ⋅n =
n→∞ a − b ⎡ a − b
) − 1⎤⎥⎦ (a + b) ⎡⎢⎣(a + b) − 1⎤⎥⎦
n n
( ) ⎢⎣(
(a − b − 1)(a + b − 1) n
= lim 2 =
(a − b2 ) − (a − b)n − (a + b)n + 1
2 2 n
a −b n→∞

n n
= (a − b −1)(a + b −1) lim n n
= 2 (1− a ) lim n n
n→∞ 2 − ( a − b) − ( a + b) n→∞ 2 − ( a − b ) − ( a + b)
B.IV.1. a − b < 1 and a + b > 1 . Applying the theorem of Stolz-Césaro, one obtains:
n
L = 2 (1− a ) lim =0
n→∞ ⎡
n ⎢ 2 ⎛ a − b ⎞⎟
n ⎤
( a + b) ⎢ ⎜ ⎥
n −⎜
⎜⎝ a + b ⎟⎟⎠ −1⎥
⎣⎢ ( a + b ) ⎦⎥
B.IV.2. a − b < 1 and a + b = 1 ⇔ a − b < 1 and a + b = −1 .
This case does not exist because it would result a 2 − b2 < 1 , therefore a 2 − b2 ≠ 1 .
Contradiction with B.IV.
B.IV.3. a − b < 1 and a + b < 1
B.IV.4. a − b = 1 and a + b < 1
B.IV.5. a − b = 1 and a + b > 1 these 5 cases don’t exist because of B).
B.IV.6. a − b > 1 and a + b > 1
B.IV.7. a − b > 1 and a + b = 1
B.IV.8. a − b > 1 and a − b < 1 ⇒ L = 0
B.IV.9. a − b = 1 and a + b = 1 ⇔ a − b = −1 and a + b = −1 ⇒ a = −1 and b = 0 , implies
that the limit does not exist.
And, look, all the cases are analyzed.
The discussion about limit is very long, but it necessitate a good arrangement of cases
(which depend of the real parameters a and b).

7.134.
Let a0 , a1 ,..., an be real numbers, an > 0, n ∈

130
One considers the polynomial
P( x) = an x n + ... + a1 x + a0
such that: if there exist ai < 0, 1 ≤ i ≤ n − 1 then the first non null coefficient
ai+k : 1 ≤ k ≤ n − 1 , before ai verifies ai ≤ ai+k . If x1 ,..., xm ∈ + determine the minimum of
the expression:
m ⎛ ⎛ 1 ⎞⎟⎞⎟
⎜⎜
E ( x1 ,..., xm ) = ∑ ⎜ P( x j ) + P ⎜⎜⎜ ⎟⎟⎟⎟⎟
j =1 ⎜
⎝ ⎜⎝ x j ⎟⎠⎟⎠
and the point ( x1 ,..., xn ) where this minimum is obtained.

Solution:
1 1
If x > 0 and i > j , then x i + i
≥ x j + j ≥ 2 , the two equalities taking place when
x x
x = 1.
m ⎛ ⎞⎟
⎜ ⎛ 1 ⎞⎟ ⎛ 1 ⎞⎟
E ( x1 ,..., xm ) = ∑ ⎜⎜an ⎜⎜⎜ x nj + n ⎟⎟ + ... + a1 ⎜⎜⎜ x j + ⎟⎟ + 2a0 ⎟⎟⎟
j =1 ⎜
⎝ ⎜⎝ x j ⎟⎠ ⎝⎜ x j ⎠⎟ ⎠⎟
Let’s consider
⎛ 1 ⎞⎟ m ⎛ 1 ⎞⎟ ⎛ 1 ⎞⎟
F ( x j ) = P ( x j ) + P ⎜⎜⎜ ⎟⎟ = ∑ an ⎜⎜⎜ x nj + n ⎟⎟ + ...a1 ⎜⎜⎜ x j + ⎟⎟ + 2a0
⎜⎝ x j ⎠⎟ j=1 ⎜⎝ x j ⎠⎟ ⎜⎝ x j ⎠⎟
If ai ≥ 0, 1 ≤ i ≤ n − 1 , then
min* F ( x j ) = 2 (a0 + a1 + ... + an )
x j∈ +

and this is only realized for x j = 1


If there exist ai < 0, 1 ≤ i ≤ n − 1 , then the hypothesis of the problem will give
ai ≤ ai+k , 1 ≤ k ≤ n − 1 , and for the other coefficients one has ai+1 = ... = ai+k−1 = 0
Therefore:
⎛ 1 ⎟⎞ ⎛ 1 ⎟⎞ ⎛ 1 ⎟⎞ ⎛ ⎞
ai+k ⎜⎜⎜ x ij+k + i+k ⎟⎟ + ai ⎜⎜⎜ x ij + i ⎟⎟ = (ai+k ai )⎜⎜⎜ x ij+k + i+k ⎟⎟ + ai ⎜⎜ x i+k + 1 − x i − 1 ⎟⎟ ≥
⎜⎜ j ⎟
x j ⎟⎠ x j ⎟⎠ x j ⎟⎠ x ij ⎟⎠
j
⎝⎜ ⎝⎜ ⎝⎜ ⎝ x ij+k
≥ (ai+k ai ) ⋅ 2 + 0 = 2 (ai + ai+k )
the equality taking place only for x j = 1 , and the same it results that
min* F ( x j ) = 2 (a0 + a1 + ... + an )
x j∈ +

which it is only realized for x j = 1


One finds that min E ( x1 ,..., xm ) = 2m (a0 + a1 + ... + an )
*
x j∈ +
j∈{1,...,m}

and this is only realized for ( x1 ,..., xm ) = (1,...,1)


m

131
7.135.
Show that
a) The sum of the power of order 2 p + 1 of 2k + 1 natural consecutive numbers is
divisible by 2k + 1 .
b) The sum of the power of order 2 p + 1 of 2k natural consecutive numbers is divisible by
2k if and only if p ≥ 1 and k is divisible by 2.

Solution
a) Let S 1 the sum of the powers of order 2 p + 1 of 2k + 1 natural consecutive numbers.
The 2k + 1 natural consecutive numbers constitute a complete system of residues modulo 2k + 1
That means:
2 p+1 2 p+1 2 p+1
S 1 = 02 p+1 + 12 p+1 + 22 p+1 + ... + k 2 p+1 + (k + 1) + ... + (2k −1) + ( 2k ) (mod 2k + 1)
2k − i ≡ −(i + 1)( mod 2 k + 1) for 0 ≤ i ≤ k − 1
2 p+1 2 p+1
( 2k − i ) ≡ −(i + 1) (mod 2k + 1)
0 ≤ i ≤ k −1
Therefore:
2 p+1
(2k ) ≡ −12 p+1 (mod 2k + 1)
2 p+1
(2k −1) ≡ −k 2 p+1 (mod 2k + 1)
…………………………………….
2 p+1
(k + 1) ≡ −k 2 p+1 (mod 2k + 1)

2 p+1 2 p+1 2 p+1 2 p+1
S 1 = 02 p+1 + 12 p+1 + 22 p+1 + ... + (k −1) + k 2 p+1 + (k + 1) + ... + (2k − 2) + (2k −1) ( mod 2k )
Therefore
S1 : 2k + 1
b) Similarly, let S2 the sum of the powers of order 2 p + 1
of 2k consecutive natural numbers. The 2k natural consecutive numbers constitute a complete
system of residues modulo 2k . That is:
2 p+1 2 p+1
S 2 = 02 p+1 + 12 p+1 + 22 p+1 + ... + (k −1) + k 2 p+1 + (k + 1) +.
2 p+1 2 p+1
.. + (2k − 2) + (2k −1) (mod 2k ) . But 2k − i ≡ −i (mod 2k ) for 1 ≤ i ≤ k − 1 ⇒
2 p+1
(2k − i ) ≡ −i 2 p+1 (mod 2k ) :
for 1 ≤ i ≤ k − 1
Then:
2 p+1
(2k −1) ≡ −12 p+1 (mod 2k )

132
2 p+1
( 2k − 2) ≡ −22 p+1 (mod 2k )
…………………………………
2 p+1 2 p+1
(k + 1) ≡ −(k −1) (mod 2k )
2 p+1 2 p+1
S 2 = 0 + 12 p+1 + 22 p+1 + ... + (k −1) + k 2 p+1 −(k −1) − ... − 22 p+1 −12 p+1 ≡ k 2 p+1 ( mod 2k )
Then
( S2 : 2k ) ⇔ (k 2 p+1 : 2k ) ⇔ (k 2 p : 2) ⇔ ( p ≥ 1 and k : 2)
i.e. k is divisible by 2.

7.136.
( )
Prove that if a and m are integers, m ≠ 0 , then a m − a ( m −1) is divisible by m .

Solution
I) m is prime.
a) a = M m (multiple of m) then a m − a = M m , and we find the conclusion.
b) a ≠ M m we have, using the Fermat theorem a m − a = M m
II) m is not prime, m ≠ 0
a) |m| = 4. Then
( ) ( )
E = a m − a (m −1)! ≡ 2a a m −1 −1 ≡ 2a (a 3 −1)( mod 4)
If a = M 2 (multiple of 2), it results that E ≡ 0 ( mod 4)
If a = M 2 + 1 , it results that a 3 −1 = M 2 , from where E ≡ 0 ( mod 4)
b) m ≠ 4 . Then ∃a, b ∈ − {0, −1, +1} such that m = a ⋅ b
If a ≠ b , because a < m − 1 , b < m − 1 , it is clear that a and b are among the factors of
( m −1)! , then ( m −1)! ≡ 0(mod m) , from where we have the conclusion.
If a = b , because m ≠ 4 and a < ( m − 1) , b < m − 1 , we have 2 b < m − 1 , therefore a
and 2 b are found among the factors of ( m −1)! , then ( m −1)! ≡ 0(mod m) , then
E ≡ 0 ( mod m )
.

Remark:
In II we proved the following assertion: if m ∈ − {0, ±2} , then ( m −1)! ≡ 0(mod m) .

133
REFERENCES
[1] Gazeta Matematică, Gazette de la Société de Mathématique de Roumanie;
[2] Revista Matematică a Elevilor din Timişoara;
[3] Ştiinţă şi Tehnică, Bucharest;
[4] Licăriri, Craiova;
[5] Matematikai Lapok, Cluj-Napoca;
[6] Les Annales de l’Université de Timişoara;
[7] Buletinul Ştiinţific şi Tehnic al Institultului Politehnic “Traian Vuia”, Timişoara
[8] Les Annals de l’Université de Craiova;
[9] W. Sierpinski – 250 problèmes de théorie élémentaire des nombres – Classiques
Hachette, Paris, 1972;
[10] D. Gorll, G. Girard – Les olympides internationals de mathématique - Classiques
Hachette, Paris, 1976.

134
This book is addressed to College honor students, researchers, and professors.
It contains 136 original problems published by the author in various
scientific journals around the world.
The problems could be used to preparing for courses, exams, and
Olympiads of Mathematics.
Many of these have a generalized form.
For each problem we provide a detailed solution.

I was a professeur coopérant between 1982-1984, teaching mathematics in


French language, at Lycée Sidi EL Hassan Lyoussi in Sefrou, Province de Fès,
Morocco.

I used many of these problems for selecting and training, together with other
Moroccan professors, in Rabat city, of the Moroccan student team for the
International Olympiad of Mathematics in Paris, France, 1983.

You might also like